首页 > 其他分享 >高考真题2

高考真题2

时间:2024-09-02 22:52:23浏览次数:1  
标签:right frac 真题 高考 sqrt cdots leqslant left

%\documentclass[11pt,a4paper]{ctexbook}
\documentclass[11pt,a4paper]{ctexart}
%\documentclass{exam-zh}%搞不清楚还有哪个宏包有冲突
%\usepackage[utf8]{inputenc}
%\usepackage{varwidth}
\usepackage{geometry}
\geometry{top=2cm,bottom=2cm,right=1.8cm,left=1.8cm}
\usepackage{float}
\usepackage[explicit,clearempty,pagestyles]{titlesec}%设置section外观,要放在fancyhdr宏包之前避免报错
\usepackage{fancyhdr}
\usepackage{lastpage}
\usepackage{graphicx}
\usepackage{wrapfig}
\usepackage{pgf}
\usepackage{exam-zh-choices}%exam-zh与有些宏包冲突,只选了其中的选项宏包
\usepackage{amsmath}
\usepackage{amssymb}%与exam-zh冲突
%\usepackage{physics}%向量相关宏包
\usepackage{microtype}
\usepackage{xcolor}
\usepackage{tcolorbox}
\usepackage{fontawesome5}%用来输出辣椒符号
\usepackage{enumitem}%设置有序列表格式,导入后enumerate环境就可以更改起始序号
\usepackage{tikz}
% \usepackage{showframe}
\usepackage{ifthen}
%%%%%%%%%%%%%%%%%%%%%%%%%%%%%%%%%%%%%%%%%%%
%自定义颜色
\definecolor{mycolor}{HTML}{4e9b86}%可以通过改变代码来改变标题颜色
%16进制颜色命令的参数是[HTML],
%%%%%%%%%%%%%%%%%%%%%%%%%%%%%%%%%%%%%%%%%%%%%%%%%%%%
%页眉与页脚设置%
\pagestyle{fancy}
\fancyhf{} % 清空页眉页脚
\fancypagestyle{plain}{% 重定义plain页面样式
\fancyfoot[C]{\kaishu\mytitle\quad 第\thepage 页\quad 共\pageref{LastPage}页%\quad\textcolor{mycolor}{\faWeixin vfmath}
} % 首页页脚,设置页脚居中
}
\fancyfoot[C]{\kaishu\mytitle\quad 第\thepage 页\quad 共\pageref{LastPage}页%\quad\textcolor{mycolor}{\faWeixin vfmath}
} % 设置页脚居中
\renewcommand{\headrulewidth}{0pt} % 去掉页眉的横线
%\renewcommand{\footrulewidth}{0pt}
%%%%%%%%%%%%%%%%%%%%%%%%%
%设置编号格式
\renewcommand{\thesection}{\chinese{section}} % 去掉章节编号前缀
\titleformat{\section}%设置section的外观,修改自https://www.latexstudio.net/index/details/index/mid/871
{\normalfont\zihao{-4}\bfseries\raggedright}
{}
{-0.1em}
{
\begin{tikzpicture}%[remember picture]
\fill[xshift=-4mm,yshift=-4.5mm,mycolor] (0,0) rectangle (1.5,.02);
\fill[xshift=-4mm,yshift=-4.5mm,mycolor!30] (0,.2) rectangle (1.5,.75);
\fill [thick,mycolor!30,xshift=-4mm,yshift=-4.5mm] (16mm,0) -- (16mm,0.7mm) -- (16.7mm,1.2mm) -- (\textwidth-1mm,1.5mm) -- (\textwidth-1mm,0) -- cycle;
\node[right] at (12mm,0) {\color{mycolor}#1};
\node at (.35,0) {\color{mycolor}类型\thesection};
\end{tikzpicture}
\vspace*{-1cm}
}
%%%%%%%%%%%%%%%%%%%%%%%%%%%%%%%%%%%%%%%%%%%%%%%%%%%%%%%%%
%设置题干、答案解析以及试题来源的格式、显示与隐藏等
%以下是自动显示或隐藏答案、解析的代码
%在这个示例代码中,我们定义了一个布尔变量 showjiexi,用于控制是否显示解析。我们还定义了一个jiexi 命令,用于在需要的地方显示解析。在选择题中,我们使用了 enumerate 环境来列出选项,并在需要的地方调用 jiexi 命令来显示解析。
\newboolean{showjiexi}
\newboolean{showdaan}
\newboolean{showtigan}
\newboolean{shownandu}
\newboolean{showtiyuan}
\setboolean{showtiyuan}{true}%题源开关
\setboolean{shownandu}{true}%难度开关
\setboolean{showtigan}{true}%题干开关,等将难度,题源关掉才能关掉
\setboolean{showdaan}{true} %答案开关。设置是否显示答案,false为不显示,true为显示
\setboolean{showjiexi}{true} %解析开关设置是否显示解析
\newcommand{\daan}[1]{\ifthenelse{\boolean{showdaan}}{\tikz[baseline]{\node[anchor=base,font=\bfseries\kaishu\color{white},draw=mycolor!90!white,fill=mycolor!90!white,rounded corners=0.8em,inner xsep=0.5em,ultra thick,minimum height=1.2ex](T)at(0,0){答案};}{\kaishu #1}}{}\par} % 定义答案命令,可以通过修改颜色值来改变颜色
\newcommand{\jiexi}[1]{\ifthenelse{\boolean{showjiexi}}{\tikz[baseline]{\node[anchor=base,font=\bfseries\kaishu\color{white},draw=mycolor!90!white,fill=mycolor!90!white,rounded corners=0.8em,inner xsep=0.5em,ultra thick,minimum height=1.2ex](T)at(0,0){解析};}{\kaishu #1}}{}\par} % 定义解析命令,字体是楷书
\newcommand{\tigan}[1]{\ifthenelse{\boolean{showtigan}} {#1}{}\par}
\newcommand{\tiyuan}[1]{\ifthenelse{\boolean{showtiyuan}\AND \boolean{showtigan}}{\textcolor{mycolor}{\textbf{\kaishu {(#1)}}}}{}}%试题来源,蓝色显示。题源和题干同时为真则显示,否则不显示
\newcommand{\lajiao}{\color[HTML]{f47a55}{\faPepperHot}}%定义一个新的命令辣椒用来存放红色辣椒图标
\newcommand{\nandu}[1]{\ifthenelse{\boolean{shownandu}\AND \boolean{showtigan}}{%难度和题干同时为真则显示难度,否则不显示
\ifcase#1%
\or {\lajiao}%
\or {\lajiao\lajiao}%
\or {\lajiao\lajiao\lajiao}%
\or {\lajiao\lajiao\lajiao\lajiao}%
\or {\lajiao\lajiao\lajiao\lajiao\lajiao}%
\fi%
}{}}%难易度显示,根据数字的个数来显示辣椒的个数。
%%%%%%%%%%%%%%%%%%%%%%%%%%%%%%%
%由于amssymb与exam-zh冲突需要重新定义空集符号,本模板为使用exam-zh文档类,可不用理会
\newcommand{\kongji}{%
\mathord{\text{%
\tikz[baseline,line width=0.05ex,line join=round]
\draw (0,0.6ex) circle (0.35em) (-0.35em,-0.25ex) -- (0.35em,1.35ex);}}}
%定义平行且等于符号
\newcommand\pxqdy{%
\mathrel{\text{%
\tikz[baseline,line width=0.05ex,line cap=round]
\draw (0,0) -- (.7em,0)
(0,.45ex) -- (.7em,.45ex)
(.25em,.55ex) -- (.35em,1.6ex)
(.45em,.55ex) -- (.55em,1.6ex);}}}

%定义平行四边形符号
\newcommand\pxsbx{%
\mathord{\text{%
\tikz[baseline,line width=0.05ex,line join=round]
\draw (0,.1ex) -- (.8em,.1ex) -- (1em,1.6ex) -- (.2em,1.6ex) -- cycle;}}}
%%%%%%%%%%%%%%%%%%%%%%%%%%%%%%%%%%%%%%%%%%%%%%
%设置选择题选项,针对高中数学试题设置为4个选项。
\newcommand{\xuanxiang}[4]{\begin{choices}
\item #1
\item #2
\item #3
\item #4
\end{choices}}
%%%%%%%%%%%%%%%%%%%%%%%%%%%%%%%%%%%%%%%%%%%%%%%
\newcommand*\cir[1]{\tikz[baseline=(char.base)]{%来源于testbook宏包内容
\node[circle,fill=mycolor,draw=mycolor,minimum size=0.5cm,align=center,inner sep=-1.0pt,font=\bfseries\sffamily\footnotesize\color{white}] (char){#1};}
}
%题目环境%
\newenvironment{QsNum}{\begin{enumerate}[label=\protect\cir{\arabic*},itemindent=0em]\setlength{\columnsep}{4.5em}}
{\end{enumerate}}%格式为mycolor色的圆圈
%根据是否显示题干修改标题,显示题干为XXX试题,不显示题干显示答案为XX试题答案
\newcommand{\mytitle}{}
\ifthenelse{\boolean{showtigan}}{\renewcommand{\mytitle}{新定义问题}} {\renewcommand{\mytitle}{新定义问题答案}}
\newcommand{\mytoday}{\number\year.\number\month.\number\day}%按照纯数字2023.5.26的格式输出,其它设置不能得到想要的结果。
\title{\fontsize{22}{22}\selectfont\bfseries\textcolor{mycolor}{\mytitle}}%设置标题
\author{}
\date{}

\usepackage{pifont}%带圆圈数字

\everymath{\displaystyle}
\allowdisplaybreaks[4]%公式跨页

\usepackage{tasks}
\settasks{ %设置
label=\Alph*., %编号
label-format={\bfseries}, %加粗
%item-indent={-0.1em},
%label-offset={-0.05em}
}

\begin{document}

\maketitle
\vspace{-4em} %将空隙缩小4个字符大小
\begin{flushright}%日期居右
%\author{\kaishu{命题人:潍坊高中数学}}\par
\date{\mytoday}
\end{flushright}

\begin{QsNum}
\item
\nandu{5}
\tiyuan{微分中值定理}
\tigan{(17分)罗尔中值定理是微分学中一条重要的定理.罗尔定理描述如下:设函数$f$在$[a,b]$上连续,在$(a,b)$上可导,且$f(a)=f(b)$,则存在一点$\xi\in (a,b)$,使得$f'(\xi)=0$.

{\heiti 注:}这里“连续”是指函数的图象是连续不断的曲线, “可导”是指函数在给定区间内每个点处的导数都存在.

(1) (4分)设实数$a,b,c,d$满足$\frac{a}{4}+\frac{b}{3}+\frac{c}{2}+d=0$.
证明:函数$g(x)=ax^3+bx^2+cx+d$在$(0,1)$内存在零点;

(2)选择下面\ding{172}, \ding{173}中的一个定理进行证明,如果两个都选,则按第一个解答计分.

\ding{172} (5分)证明拉格朗日中值定理:设函数$f$在$[a,b]$上连续,在$(a,b)$上可导,则存在一点$\xi\in (a,b)$,使得$\frac{f(b)-f(a)}{b-a}=f'(\xi)$;

\ding{173} (6分)证明柯西中值定理:设函数$f,g$在$[a,b]$上连续,在$(a,b)$上可导,且$g'(x)\neq 0,\forall x\in (a,b)$,则存在一点$\xi\in (a,b)$,使得$\frac{f(b)-f(a)}{g(b)-g(a)}=\frac{f'(\xi)}{g'(\xi)}$;

(3) (7分)设$n\in\mathbf{N}$,且 $f(x)=\sum_{k=1}^nc_k\mathrm{e}^{\lambda_kx}$,其中 $\lambda_1,\lambda_2,\cdots,\lambda_n$为互不相等的实数, $c_1,c_2,\cdots,c_n$是不同时为$0$的实数.证明:函数$f$至多有 $n-1$个实零点.}
\daan{ }
\jiexi{(1)考虑$G\left( x \right) =\frac{a}{4}x^4+\frac{b}{3}x^3+\frac{c}{2}x^2+dx$,
由于$G(0)=G(1)=0$,则由罗尔中值定理可知在$(0,1)$内存在一个点$x_0$,使得$G'(x_0)=ax_0^3+bx_0^2+cx_0+d=0$,即函数$g(x)=ax^3+bx^2+cx+d$在$(0,1)$内存在零点.

(2)若选择\ding{172},令$k=\frac{f(b)-f(a)}{b-a}$,则$f(b)-f(a)=kb-ka$,故$f(b)-kb=f(a)-ka$.

考虑函数$F(x)=f(x)-kx$,则$F(a)=F(b)$,由罗尔中值定理可知存在一点$\xi\in (a,b)$,使得$F'(\xi)=f'(\xi)-k=0$,即存在一点$\xi\in (a,b)$,使得$\frac{f(b)-f(a)}{b-a}=f'(\xi)$.

若选择\ding{173},令$k=\frac{f(b)-f(a)} {g(b)-g(a)}$,则$f(b)-f(a)=kg(b)-kg(a)$,
故$f(b)-kg(b)=f(a)-kg(a)$.

考虑函数$G(x)=f(x)-kg(x)$,则$G(a)=G(b)$,由罗尔中值定理可知存在一点$\xi\in (a,b)$,使得$G'(\xi)=f'(\xi)-kg'(\xi)=0$,即存在一点$\xi\in (a,b)$,使得$\frac{f(b)-f(a)} {g(b)-g(a)}=\frac{f'(\xi)}{g'(\xi)}$.

(3)当$n=1$时, $f(x)=c_1\mathrm{e} ^{\lambda_1x}$,由于$c_1\neq 0,$故此时$f$没有零点.假设当$n=m$时,
$f(x)=\sum_{k=1}^mc_k\mathrm{e}^{\lambda_kx}$至多有 $m-1$个实零点的命题成立.则当$n=m+1$时,
$c_ {m+1}\neq 0$, $f\left( x \right) =c_1\text{e}^{\lambda _1x}+c_2\text{e}^{\lambda _2x}+\cdots +c_m\text{e}^{\lambda _mx}+c_{m+1}\text{e}^{\lambda _{m+1}x}$,将它写成
$$
f\left( x \right) =\text{e}^{\lambda _1x}\left( c_1+c_2\text{e}^{\left( \lambda _2-\lambda _1 \right) x}+\cdots +c_m\text{e}^{\left( \lambda _m-\lambda _1 \right) x}+c_{m+1}\text{e}^{\left( \lambda _{m+1}-\lambda _1 \right) x} \right).
$$

而$f(x)=0$的实根就是$c_1+c_2\text{e}^{\left( \lambda _2-\lambda _1 \right) x}+\cdots +c_m\text{e}^{\left( \lambda _m-\lambda _1 \right) x}+c_{m+1}\text{e}^{\left( \lambda _{m+1}-\lambda _1 \right) x}=0$的实根.但$g(x)=c_1+\sum_{k=2}^{m+1}c_k\mathrm{e}^{(\lambda_k-\lambda_1)x}$至多只有$m$个零点. (利用反证法)否则,若 $g(x)=0$有 $m+1$个零点,由 罗尔中值定理可知, $g'(x)=0$至少有$m$个零点.但

$$
g'(x)=\sum_{k=2}^{m+1}c_{k}(\lambda_{k}-\lambda_{1})
\mathrm{e}^{(\lambda_{k}-\lambda_{1})x}
=\sum_{i=1}^{m}b_{i}\mathrm{e}^{\mu_{i}x},
$$
其中$b_i=c_{i+1}(\lambda_{i+1}-\lambda_1),
\mu_i=\lambda_{i+1}$-$\lambda_1$.由$\lambda_i$互不相同, $c_{m+1}\neq 0$,满足假设条件,故$g'$至多只有$m-1$个零点,矛盾.这就证明了$f(x)$至多只有$n-1$个不同零点.

恰有$n-1$个不同零点的例子是存在的,例如
$$
f(x)=(\mathrm{e}^x-\mathrm{e})(\mathrm{e}^x-\mathrm{e}^2)
\cdots(\mathrm{e}^x-\mathrm{e}^{n-1})
=\sum_{k=1}^nc_k\mathrm{e}^{(k-1)x}.
$$
其中$c_n=1,c_1=(-1)^{n-1}\mathrm{e}^{\frac{n(n-1)}2}$都不为零.它恰有$n-1$个不同的根: $1,2,\cdots,n-1$.}

 

\item
\nandu{5}
\tiyuan{內积}
\tigan{(17分)对于任意正整数$n$,记集合$\mathbf{R}^n=\left\{ \boldsymbol{x}\left| \boldsymbol{x}=\left( x_1,x_2,\cdots ,x_n \right) ,x_i\in \mathbf{R},i=1,2,3,\cdots ,n \right. \right\}$,其中元素$\boldsymbol{x}$称为一个$n$维向量.
设$k\in \mathbf{R}$, $\boldsymbol{x}=\left( x_1,x_2,\cdots ,x_n \right)\in \mathbf{R}^n,\boldsymbol{y}=\left( y_1,y_2,\cdots ,y_n \right)\in \mathbf{R}^n$,定义加法和数乘运算: $\boldsymbol{x}+\boldsymbol{y}=\left( x_1+y_1,x_2+y_2,\cdots ,x_n+y_n \right)$,
$k\boldsymbol{x}=\left( kx_1,kx_2,\cdots ,kx_n \right)$.
如果对任何两个向量$\boldsymbol{x},\boldsymbol{y}$,
有一实数$\left< \boldsymbol{x},\boldsymbol{y} \right>$与之对应,并且满足下列条件:

(i) $\left< \boldsymbol{x},\boldsymbol{x} \right>\geqslant 0$,而$\left< \boldsymbol{x},\boldsymbol{x} \right>=0$当且仅当$\boldsymbol{x}=\left( 0,0,\cdots ,0 \right)\in \mathbf{R}^n$;

(ii) $\left< a\boldsymbol{x}+b\boldsymbol{y},\boldsymbol{z} \right> =a\left< \boldsymbol{x},\boldsymbol{z} \right> +b\left< \boldsymbol{y},\boldsymbol{z} \right>$,其中$\boldsymbol{x},\boldsymbol{y},\boldsymbol{z}\in \mathbf{R}^n$, $a,b\in \mathbf{R}$;

(iii) $\left< \boldsymbol{x},\boldsymbol{y} \right>=\left< \boldsymbol{y},\boldsymbol{x} \right>$, $\boldsymbol{x},\boldsymbol{y}\in \mathbf{R}^n$,

则称$\left< \boldsymbol{x},\boldsymbol{y} \right>$为$\boldsymbol{x}$和$\boldsymbol{y}$的內积,
称$\|\boldsymbol{x}\|=\sqrt{\left< \boldsymbol{x},\boldsymbol{x} \right>}$为$\mathbf{R}^n$上的范数.

(1) (4分)判断下列$\left< \boldsymbol{x},\boldsymbol{y} \right>$是否为內积,并说明理由.

\ding{172} $\left< \boldsymbol{x},\boldsymbol{y} \right>=x_1y_1+x_2y_2+\cdots+x_ny_n$;

\ding{173} $\left< \boldsymbol{x},\boldsymbol{y} \right>=x_1+y_1+x_2+y_2+\cdots+x_n+y_n$.

%\ding{174} $\left< \boldsymbol{x},\boldsymbol{y} \right>=x_1^2y_1^2+x_2^2y_2^2+\cdots+x_n^2y_n^2$.


(2) (5分)证明:$\|\boldsymbol{x}+\boldsymbol{y}\|^2+\|\boldsymbol{x}-\boldsymbol{y}\|^2=2\left(\|\boldsymbol{x}\|^2+\|\boldsymbol{y}\|^2\right)$.

%(2)证明: $| \|\boldsymbol{x}\|-\|\boldsymbol{y}\| | \leqslant \|\boldsymbol{x}+\boldsymbol{y}\|\leqslant \|\boldsymbol{x}\|+\|\boldsymbol{y}\|$,这里$| \|\boldsymbol{x}\|-\|\boldsymbol{y}\| |$表示$\|\boldsymbol{x}\|-\|\boldsymbol{y}\|$的绝对值.

(3) (8分)证明:对于任意实数$p,q>1$, $\frac{1}{p}+\frac{1}{q}=1$,成立
$$
\left( 2-p \right) \lVert \boldsymbol{x} \rVert ^2+\left( 2-q \right) \lVert \boldsymbol{y} \rVert ^2\leqslant \lVert \boldsymbol{x}+\boldsymbol{y} \rVert ^2\leqslant p\lVert \boldsymbol{x} \rVert ^2+q\lVert \boldsymbol{y} \rVert ^2.
$$
}
\daan{ }
\jiexi{(1) \ding{172}中$\left< \boldsymbol{x},\boldsymbol{y} \right>$是內积,而\ding{173}不是.

对于\ding{172}, $\left< \boldsymbol{x},\boldsymbol{x} \right>=x_1^2+x_2^2+\cdots+x_n^2\geqslant 0$,
而$\left< \boldsymbol{x},\boldsymbol{x} \right>=0$当且仅当$x_1=x_2=\cdots=x_n=0$,
此时$\boldsymbol{x}=\left( 0,0,0,\cdots ,0 \right)\in \mathbf{R}^n$成立,故条件(i)满足;

设$\boldsymbol{z}=\left( z_1,z_2,z_3,\cdots ,z_n \right)\in \mathbf{R}^n$, $a,b\in \mathbf{R}$,则
\begin{align*}
\left< a\boldsymbol{x}+b\boldsymbol{y,z} \right> &=\sum_{i=1}^n{\left( ax_i+by_i \right) z_i}=a\sum_{i=1}^n{x_iz_i}+b\sum_{i=1}^n{y_iz_i}\\
&=a\left< \boldsymbol{x,z} \right> +b\left< \boldsymbol{y,z} \right>,
\end{align*}
故条件(ii)满足;

又$\left< \boldsymbol{x,y} \right> =\sum_{i=1}^n{x_iy_i}=\sum_{i=1}^n{y_ix_i}=\left< \boldsymbol{y,x} \right>$,故条件(iii)也满足.

故\ding{172}中$\left< \boldsymbol{x},\boldsymbol{y} \right>$是內积.

 

对于\ding{173}, 取$\boldsymbol{x}=\left( -1,-1,-1,\cdots ,-1 \right)\in \mathbf{R}^n$,
可知
$\left< \boldsymbol{x},\boldsymbol{x} \right>=-2n<0$,故条件(i)不满足.

故\ding{173}中$\left< \boldsymbol{x},\boldsymbol{y} \right>$是內积.

(2)由题意可知
\begin{align*}
\lVert \boldsymbol{x}+\boldsymbol{y} \rVert ^2 &=\left< \boldsymbol{x}+\boldsymbol{y,x}+\boldsymbol{y} \right> =\left< \boldsymbol{x,x} \right> +\left< \boldsymbol{y,x} \right> +\left< \boldsymbol{x,y} \right> +\left< \boldsymbol{y,y} \right>\\
&=\lVert \boldsymbol{x} \rVert ^2+2\left< \boldsymbol{x,y} \right> +\lVert \boldsymbol{y} \rVert ^2,
\end{align*}

\begin{align*}
\lVert \boldsymbol{x}-\boldsymbol{y} \rVert ^2 &=\left< \boldsymbol{x}-\boldsymbol{y,x}-\boldsymbol{y} \right> =\left< \boldsymbol{x,x} \right> -\left< \boldsymbol{y,x} \right> -\left< \boldsymbol{x,y} \right> +\left< \boldsymbol{y,y} \right>\\
&=\lVert \boldsymbol{x} \rVert ^2-2\left< \boldsymbol{x,y} \right> +\lVert \boldsymbol{y} \rVert ^2,
\end{align*}
于是$\|\boldsymbol{x}+\boldsymbol{y}\|^2+\|\boldsymbol{x}-\boldsymbol{y}\|^2
=2\left(\|\boldsymbol{x}\|^2+\|\boldsymbol{y}\|^2\right)$.

(3) {\heiti 引理:} $| \|\boldsymbol{x}\|-\|\boldsymbol{y}\| | \leqslant \|\boldsymbol{x}+\boldsymbol{y}\|\leqslant \|\boldsymbol{x}\|+\|\boldsymbol{y}\|$,这里$| \|\boldsymbol{x}\|-\|\boldsymbol{y}\| |$表示$\|\boldsymbol{x}\|-\|\boldsymbol{y}\|$的绝对值.


{\heiti 引理的证明.}对任意实数$t$,成立
$$
0\leqslant \left< \boldsymbol{x}-t\boldsymbol{y,x}-t\boldsymbol{y} \right> =\left< \boldsymbol{x,x} \right> -t\left< \boldsymbol{y,x} \right> -t\left[ \left< \boldsymbol{x,y} \right> -t\left< \boldsymbol{y,y} \right> \right].
$$
令$t=\frac{\left< \boldsymbol{x,y} \right>}{\left< \boldsymbol{y,y} \right>}$,则有
$$
0\leqslant \left< \boldsymbol{x,x} \right> -\frac{\left< \boldsymbol{x,y} \right>}{\left< \boldsymbol{y,y} \right>}\left< \boldsymbol{y,x} \right> =\lVert \boldsymbol{x} \rVert ^2-\frac{\left| \left< \boldsymbol{x,y} \right> \right|^2}{\lVert \boldsymbol{y} \rVert ^2},
$$
整理得$\left| \left< \boldsymbol{x,y} \right> \right|\leqslant \lVert \boldsymbol{x} \rVert \cdot \lVert \boldsymbol{y} \rVert$.

因此
\begin{align*}
\lVert \boldsymbol{x}+\boldsymbol{y} \rVert ^2 &=\lVert \boldsymbol{x} \rVert ^2+2\left< \boldsymbol{x,y} \right> +\lVert \boldsymbol{y} \rVert ^2\\
&\leqslant \lVert \boldsymbol{x} \rVert ^2+2\lVert \boldsymbol{x} \rVert \cdot \lVert \boldsymbol{y} \rVert +\lVert \boldsymbol{y} \rVert ^2=\left( \lVert \boldsymbol{x} \rVert +\lVert \boldsymbol{y} \rVert \right) ^2,
\end{align*}
整理得$\lVert \boldsymbol{x}+\boldsymbol{y} \rVert\leqslant \lVert \boldsymbol{x} \rVert +\lVert \boldsymbol{y} \rVert$.

由于$\lVert \boldsymbol{x}+\boldsymbol{y} \rVert+ \lVert \boldsymbol{x} \rVert\geqslant \lVert (\boldsymbol{x}+\boldsymbol{y})+(-\boldsymbol{x}) \rVert=\lVert \boldsymbol{y} \rVert$,
于是$\lVert \boldsymbol{x}+\boldsymbol{y} \rVert\geqslant \lVert \boldsymbol{y} \rVert-\lVert \boldsymbol{x} \rVert$.

同理可得$\lVert \boldsymbol{x}+\boldsymbol{y} \rVert\geqslant \lVert \boldsymbol{x} \rVert-\lVert \boldsymbol{y} \rVert$,
因此$|\ \|\boldsymbol{x}\|-\|\boldsymbol{y}\|\ | \leqslant \|\boldsymbol{x}+\boldsymbol{y}\|\leqslant \|\boldsymbol{x}\|+\|\boldsymbol{y}\|$.

 


回到原题.要证$$\lVert \boldsymbol{x}+\boldsymbol{y} \rVert ^2\leqslant p\lVert \boldsymbol{x} \rVert ^2+q\lVert \boldsymbol{y} \rVert ^2.
$$
只需证$\left( \lVert \boldsymbol{x} \rVert +\lVert \boldsymbol{y} \rVert \right) ^2\le p\lVert \boldsymbol{x} \rVert ^2+q\lVert \boldsymbol{y} \rVert ^2$,即$\left( p-1 \right) \lVert \boldsymbol{x} \rVert ^2+\left( q-1 \right) \lVert \boldsymbol{y} \rVert ^2\geqslant 2\lVert \boldsymbol{x} \rVert \cdot \lVert \boldsymbol{y} \rVert$.

由基本不等式可得
\begin{align*}
\left( p-1 \right) \lVert \boldsymbol{x} \rVert ^2+\left( q-1 \right) \lVert \boldsymbol{y} \rVert ^2 &\geqslant 2\sqrt{\left( p-1 \right) \left( q-1 \right)}\lVert \boldsymbol{x} \rVert \cdot \lVert \boldsymbol{y} \rVert\\
&=2\sqrt{pq-p-q+1}\lVert \boldsymbol{x} \rVert \cdot \lVert \boldsymbol{y} \rVert =2\lVert \boldsymbol{x} \rVert \cdot \lVert \boldsymbol{y} \rVert,
\end{align*}
再由
\begin{align*}
\lVert \boldsymbol{x}+\boldsymbol{y} \rVert ^2 &\geqslant \left( \lVert \boldsymbol{x} \rVert -\lVert \boldsymbol{y} \rVert \right) ^2=\lVert \boldsymbol{x} \rVert ^2+\lVert \boldsymbol{y} \rVert ^2-2\lVert \boldsymbol{x} \rVert \cdot \lVert \boldsymbol{y} \rVert\\
&\geqslant \lVert \boldsymbol{x} \rVert ^2+\lVert \boldsymbol{y} \rVert ^2-\left[ \left( p-1 \right) \lVert \boldsymbol{x} \rVert ^2+\left( q-1 \right) \lVert \boldsymbol{y} \rVert ^2 \right]\\
&=\left( 2-p \right) \lVert \boldsymbol{x} \rVert ^2+\left( 2-q \right) \lVert \boldsymbol{y} \rVert ^2.
\end{align*}
因此
$$
\left( 2-p \right) \lVert \boldsymbol{x} \rVert ^2+\left( 2-q \right) \lVert \boldsymbol{y} \rVert ^2\leqslant \lVert \boldsymbol{x}+\boldsymbol{y} \rVert ^2\leqslant p\lVert \boldsymbol{x} \rVert ^2+q\lVert \boldsymbol{y} \rVert ^2.
$$
}


\item
\nandu{5}
\tiyuan{集合的势}
\tigan{集合的势是一个用来度量集合所含元素多少的量.对于两个集合$A$和$B$,如果对任意$b\in B$,存在函数$f(x)$和唯一的$a\in A$,满足$f(a)=b$,则称$A$和$B$等势,记为$A\sim B$,并称满足条件的函数$f(x)$为从$A$到$B$的一一对应.

(1)判断下列集合是否等势,并说明理由.

\ding{172} 偶数集和整数集;\qquad \ding{173} $(0,1)$和$(2,4)$;


%\ding{174} $[-1,1]$和$\mathbf{R}$. $y=\sin x$

%\ding{175} $(0,+\infty)$和$\mathbf{R}$. $y=\ln x$

\ding{174} $(-1,1)$和$\mathbf{R}$; %$y=\tan\left( \frac{\pi}{2} x\right)$.
\qquad \ding{175} 集合$\{(i,j)|i,j\in \mathbf{N}^\ast\}$与$\mathbf{N}^\ast$.

(2)证明: $\mathbf{N}^\ast\sim (0,1)\cap \mathbf{Q}$;

(3)证明: $(0,1)\sim [0,1]$以及$(0,1)$中的无理数集$\sim (0,1)$.}
\daan{ }
\jiexi{
(1)
\ding{172} 考虑$f(n)=2n,n\in \mathbf{Z}$;


\ding{173} 考虑$f(x)=2x+2,x\in (0,1)$;

\ding{174} 考虑$f(x)=\tan\left( \frac{\pi}{2} x\right),x\in (-1,1)$.

\ding{175} 将正整数$n=2^{i-1}(2j-1)$与点$(i,j)$一一对应即可.

(2)将区间$(0,1)$内的有理数按照相同的分母依次排成数列,并剔除掉相等的数,可得$\frac{1}{2},\frac{1}{3},\frac{2}{3},\frac{1}{4},\frac{3}{4},\cdots$,由此建立两集合$\mathbf{N}^\ast$与$(0,1)\cap \mathbf{Q}$之间的一一对应.

(3)设$n\in \mathbf{N}^\ast$,记$(0,1)\cap \mathbf{Q}=\{r_1,r_2,\cdots,r_n,\cdots\}$,
考虑从$(0,1)$到$[0,1]$的一一对应
$$f(x)=\begin{cases}
x, & \mbox{若$x$为无理数}, \\
0, & \mbox{若$x=r_1$},\\
1, & \mbox{若$x=r_2$},\\
r_{n-2}, & \mbox{若$x=r_n$},
\end{cases}$$
可知$(0,1)\sim [0,1]$.

设$n\in \mathbf{N}^\ast$,考虑从$(0,1)$到$(0,1)$中的无理数集的一一对应
$$f(x)=\begin{cases}
x, & \mbox{若$x$为无理数且$x\neq \frac{\sqrt{2}}{n+1}$}, \\
\frac{\sqrt{2}}{2n}, & \mbox{若$x=r_n$},\\
\frac{\sqrt{2}}{2n+1}, & \mbox{若$x=\frac{\sqrt{2}}{n+1}$},
\end{cases}$$
可知$(0,1)$中的无理数集$\sim (0,1)$.

}


\item
\nandu{5}
\tiyuan{米勒---拉宾素性检验}
\tigan{米勒---拉宾素性检验被广泛应用于大数的素性判断中.设$n$是正整数且$n\geqslant 2$,集合$X=\{1,2,\cdots,n-1\}$.设$n-1=2^km$,其中$k$是非负整数, $m$是正奇数.
若对任意正整数$a$,均有$a^n-a$是$n$的倍数,则称$n$为卡迈尔数;若对任意$a\in X$,满足下列两个条件中的一个:

(i) $a^m-1$是$n$的倍数;

(ii) 存在$i\in\{0,\cdots,k-1\}$,使得$a^{2^im}+1$是$n$的倍数,

则称$n$能通过以$a$为底的米勒检验,否则称$n$不能通过以$a$为底的米勒检验.

(1)判断$9$是否通过以$4$为底的米勒检验,说明理由;

(2)证明:合数$1281$能通过以$41$为底的米勒检验;

(3)若$n$是素数,证明: $n$必定能通过以$a$为底的米勒检验.}
\daan{}
\jiexi{(1)当$n=9,a=4$时, $9-1=2^3$,此时$k=3,m=1$,由于$a-1=3$不是$9$的倍数, $a+1=5,a^2+1=17,a^4+1=257$均不是$9$的倍数,故$9$不能通过以$4$为底的米勒检验.

(2)若$a$与$b$除以$p$的余数相同,则记为$a\equiv b\ (\bmod\ p)$.

因为$41^{5}+1\equiv (-1)^5+1=0\ (\bmod\ 3)$, $41^{5}+1\equiv (-1)^5+1=0\ (\bmod\ 7)$,
$41^{5}+1\equiv (41^{2})^2\cdot 41+1\equiv 34^2\cdot 41+1\equiv 58\cdot 41+1\equiv -3\cdot (-20)+1\equiv 0\ (\bmod\ 61)$.故$41^{5}+1$是$1281$的倍数,因此$1281$能通过以$41$为底的米勒检验.

 


(3) {\heiti 引理. (费马小定理)} 如果$p$是一个素数,而整数$a$不是$p$的倍数,则$a^{p-1}\equiv 1\ (\bmod\ p)$.

{\heiti 引理的证明.}由于$a,2a,\cdots, (p-1)a$除以$p$的余数两两不同,且跑遍$1,2,3,\cdots,p-1$,则$1\cdot 2\cdot 3\cdots\cdot p-1\equiv a\cdot 2a \cdots (p-1)a$,
即$(p-1)!\equiv (p-1)!\cdot a^{p-1}\ (\bmod\ p)$.
由于$(p-1)!$与$p$互素,则$a^{p-1}\equiv 1\ (\bmod\ p)$.

回到原题.注意到
\begin{align*}
a^{2^km}-1 &=\left( a^{2^{k-1}m} \right) ^2-1=\left( a^{2^{k-1}m}-1 \right) \left( a^{2^{k-1}m}+1 \right)\\
&=\left( a^{2^{k-2}m}-1 \right) \left( a^{2^{k-2}m}+1 \right) \left( a^{2^{k-1}m}+1 \right)\\
&=\cdots =\left( a^m-1 \right) \left( a^m+1 \right) \left( a^{2m}+1 \right) \left( a^{4m}+1 \right) \cdots \left( a^{2^{k-1}m}+1 \right),
\end{align*}
若$n$是素数,则由费马小定理可得$a^{n-1}\equiv 1\ (\bmod\ n)$,即$a^{2^{k}m}-1\equiv 0\ (\bmod\ n)$,

$$\left( a^m-1 \right) \left( a^m+1 \right) \left( a^{2m}+1 \right) \left( a^{4m}+1 \right) \cdots \left( a^{2^{k-1}m}+1 \right)\equiv 0\ (\bmod\ n),$$
则$a^m-1\equiv 0\ (\bmod\ n)$,或$a^{2^im}+1\equiv 0\ (\bmod\ n),\exists i\in\{0,\cdots,k-1\}$.则$n$必定能通过以$a$为基的米勒检验.}

 

\item
\nandu{5}
\tiyuan{杰卡德距离}
\tigan{杰卡德距离经常用来度量两个有限集合的相似程度,在机器学习、数据科学等领域有着广泛的应用.设$A$和$B$为有限集,定义杰卡德距离为:
$$d\left( A,B \right)
=\begin{cases}
1-\frac{\left| A\cap B \right|}{\left| A\cup B \right|}, & \mbox{若$A$、$B$不全为$\emptyset$} \\
0, & \mbox{若$A=B=\emptyset$}.
\end{cases}$$
其中$|A|$表示$A$中的元素个数.设$S_1,S_2,S_3$均为有限集.%$A\triangle B=(A\cup B)\backslash (A\cap B)$为对称差.

(1)若$S_1=\emptyset,S_2=\{1,2,3\},S_3=\{1,2,3,4\}$,
求$d\left( S_1,S_2 \right)$和$d\left( S_2,S_3 \right)$;

(2)记$U=S_1\cup S_2\cup S_3$, $V=S_1\cap S_2\cap S_3$,证明: $\frac{\left| S_1\cup S_2 \right|-\left| S_1\cap S_2 \right|}{\left| U \right|}\leqslant d\left( S_1,S_2 \right) \leqslant 1-\frac{\left| V \right|}{\left| U \right|}$;


(3)设$x_i\in \mathbf{N}\ (i=1,2,\cdots,7)$证明:
\begin{align*}
1+\frac{x_5+x_7}{x_1+x_3+x_4+x_5+x_6+x_7} \geqslant \frac{x_4+x_7}{x_2+x_3+x_4+x_5+x_6+x_7}
+\frac{x_6+x_7}{x_1+x_2+x_4+x_5+x_6+x_7}.
\end{align*}
}
\daan{ }
\jiexi{(1)由题意可得$d\left( S_1,S_2 \right)=1-\frac{\left| S_1\cap S_2 \right|}{\left| S_1\cup S_2 \right|} =1$,
$d\left( S_2,S_3 \right) =1-\frac{\left| S_2\cap S_3 \right|}{\left| S_2\cup S_3 \right|}=1-\frac{3}{4}=\frac{1}{4}$.


(2)一方面,由$\left| U \right|\geqslant \left| S_1\cup S_2 \right|$
可知
$$
d\left( S_1,S_2 \right) =1-\frac{\left| S_1\cap S_2 \right|}{\left| S_1\cup S_2 \right|}=\frac{\left| S_1\cup S_2 \right|-\left| S_1\cap S_2 \right|}{\left| S_1\cup S_2 \right|}\geqslant \frac{\left| S_1\cup S_2 \right|-\left| S_1\cap S_2 \right|}{\left| U \right|}.
$$

另一方面,由$\left| U \right|\geqslant \left| S_1\cup S_2 \right|$和$\left| V \right|\leqslant \left| S_1\cap S_2 \right|$可知
$$
\frac{\left| V \right|}{\left| U \right|}\leqslant \frac{\left| S_1\cap S_2 \right|}{\left| S_1\cup S_2 \right|},
$$
于是
$$
d\left( S_1,S_2 \right) =1-\frac{\left| S_1\cap S_2 \right|}{\left| S_1\cup S_2 \right|}\leqslant 1-\frac{\left| V \right|}{\left| U \right|}.
$$
因此$\frac{\left| S_1\cup S_2 \right|-\left| S_1\cap S_2 \right|}{\left| U \right|}\leqslant d\left( S_1,S_2 \right) \leqslant 1-\frac{\left| V \right|}{\left| U \right|}$.


(3)将$U$分成如图所示的四部分,则$U=T_1\cup T_2\cup T_3\cup V$.
\begin{figure}[H]
\centering
\includegraphics[width=6cm]{fig/jkdjh1.png}
\end{figure}
%保存png,用windows裁剪图片.


$\frac{\left| S_1\cup S_2 \right|-\left| S_1\cap S_2 \right|}{\left| U \right|}\leqslant d\left( S_1,S_2 \right) \leqslant 1-\frac{\left| V \right|}{\left| U \right|}$可知
$$
\frac{\left| T_1 \right|+\left| T_2 \right|}{\left| U \right|}\leqslant d\left( S_1,S_2 \right) \leqslant \frac{\left| T_1 \right|+\left| T_2 \right|+\left| T_3 \right|}{\left| U \right|}.
$$
类似地有
$\frac{\left| T_2 \right|+\left| T_3 \right|}{\left| U \right|}\leqslant d\left( S_2,S_3 \right) \leqslant \frac{\left| T_1 \right|+\left| T_2 \right|+\left| T_3 \right|}{\left| U \right|}$和$\frac{\left| T_1 \right|+\left| T_3 \right|}{\left| U \right|}\leqslant d\left( S_1,S_3 \right) \leqslant \frac{\left| T_1 \right|+\left| T_2 \right|+\left| T_3 \right|}{\left| U \right|}$,

\begin{align*}
d\left( S_1,S_2 \right) +d\left( S_2,S_3 \right) &\geqslant \frac{\left| T_1 \right|+\left| T_2 \right|}{\left| U \right|}+\frac{\left| T_2 \right|+\left| T_3 \right|}{\left| U \right|}=\frac{\left| T_1 \right|+2\left| T_2 \right|+\left| T_3 \right|}{\left| U \right|}\\
&\geqslant \frac{\left| T_1 \right|+\left| T_2 \right|+\left| T_3 \right|}{\left| U \right|}\geqslant d\left( S_1,S_3 \right).
\end{align*}

如图,将七个不相交集合的元素个数分别记作$x_i\in \mathbf{N}\ (i=1,2,\cdots,7)$,
\begin{figure}[H]
\centering
\includegraphics[width=6cm]{fig/jkdjh2.png}
\end{figure}
则由
$d\left( S_1,S_2 \right)+d\left( S_2,S_3 \right)\geqslant d\left( S_1,S_3 \right)$
可知
\begin{align*}
& 1-\frac{x_6+x_7}{x_1+x_2+x_4+x_5+x_6+x_7}
+1-\frac{x_4+x_7}{x_2+x_3+x_4+x_5+x_6+x_7}\\
\geqslant & 1-\frac{x_5+x_7}{x_1+x_3+x_4+x_5+x_6+x_7},
\end{align*}
因此
\begin{align*}
1+\frac{x_5+x_7}{x_1+x_3+x_4+x_5+x_6+x_7} \geqslant \frac{x_4+x_7}{x_2+x_3+x_4+x_5+x_6+x_7}
+\frac{x_6+x_7}{x_1+x_2+x_4+x_5+x_6+x_7}.
\end{align*}
}


\item
\nandu{5}
\tiyuan{反正切函数}
\tigan{对于任意$y\in \left( -\frac{\pi}{2},\frac{\pi}{2} \right)$,若$x=\tan y$,则记为$y=\arctan x$.

(1)求$\arctan 0$和$\arctan 1$;

%(2)证明: $\arctan A-\arctan B=\arctan \frac{A-B}{1+AB},AB>-1$;

(2)若$A>B>0$,证明: $\arctan A-\arctan B=\arctan \frac{A-B}{1+AB}$;

(3)证明: $\sum_{k=1}^{n}\arctan \frac{1}{2k^2}<\frac{\pi}{4}$.}
\daan{ }
\jiexi{(1)因为$\tan 0=0$,则$\arctan 0=0$.

又$\tan\frac{\pi}{4}=1$,则$\arctan 1=\frac{\pi}{4}$.

(2)因为$A>B>0$,则$\arctan A,\arctan B\in \left( 0,\frac{\pi}{2} \right)$且$\arctan A>\arctan B$,
则$\arctan A-\arctan B\in \left( 0,\frac{\pi}{2} \right)$.

又$\frac{A-B}{1+AB}>0$,故$\arctan \frac{A-B}{1+AB}\in \left( 0,\frac{\pi}{2} \right)$.


$$
\tan \left( \arctan A-\arctan B \right) =\frac{\tan \left( \arctan A \right) -\tan \left( \arctan B \right)}{1+\tan \left( \arctan A \right) \tan \left( \arctan B \right)}=\frac{A-B}{1+AB}
$$
可知$\arctan A-\arctan B=\arctan \frac{A-B}{1+AB}$.

(3)对于$n\in \mathbf{N}^\ast$,利用
$$
\arctan \frac{1}{2n-1}-\arctan \frac{1}{2n+1}=\arctan \frac{\frac{1}{2n-1}-\frac{1}{2n+1}}{1+\frac{1}{4n^2-1}}=\arctan \frac{1}{2n^2}
$$
可知
\begin{align*}
\sum_{k=1}^n{\arctan \frac{1}{2k^2}} &<\sum_{k=1}^n{\left( \arctan \frac{1}{2k-1}-\arctan \frac{1}{2k+1} \right)}\\
&=\arctan 1-\arctan \frac{1}{2n+1}<\frac{\pi}{4}.
\end{align*}
}
\end{QsNum}

 

(2023年全国新高考I卷22题)在直角坐标系$xOy$中,点$P$到$x$轴的距离等于点$P$到点$\left(0,\frac12\right)$的距离,记动点$P$的轨迹为$W$.

(1)求$W$的方程;

(2)已知矩形$ABCD$有三个顶点在$W$上,证明:矩形$ABCD$的周长大于$3\sqrt{3}$.

(1)解法 1.设点$P(x,y)$,则$|y|=\sqrt{x^2+\left(y-\frac{1}{2}\right)^2}$,两边平方并化简,可得$y=x^{2}+\frac{1}{4}$, 即$W$的方程为$y=x^{2}+\frac{1}{4}$.

解法2.令$X=x,Y=y-\frac{1}{4}$,这相当于将抛物线向下平移$\frac{1}{4}$个单位.由题意可知点$P$到直线$Y=-\frac{1}{4}$的距离等于点$P$到点$\left(0,\frac14\right)$的距离,则点$P$的轨迹为焦点在$\left(0,\frac14\right)$,准线为$Y=-\frac{1}{4}$的抛物线,故$X^2=Y$,则$x^2=y-\frac{1}{4}$,即$W$的方程为$y=x^{2}+\frac{1}{4}$.

 

(2)解法 1.设矩形的顶点$A,B,C,D$按顺时针排列,设顶点
$A\left(a,a^2+\frac{1}{4}\right)$, $B\left(b,b^2+\frac{1}{4}\right)$, $C\left(c,c^2+\frac{1}{4}\right)$在轨迹$W$上.
设向量$\overrightarrow{BA}$的方向向量为$(\cos\theta,\sin\theta)$,向量$\overrightarrow{BC}$的方向向量为$(\cos(\theta+90^{\circ}),\sin(\theta+90^{\circ}))$,
设边长$|BA|=u>0$, $|BC|=v>0$.

向量$\overrightarrow{BA}=(u\cos\theta,u\sin\theta)=(a-b,a^2-b^2)$,这样
$$
b+a=\frac{a^{2}-b^{2}}{a-b}=\frac{u\sin\theta}{u\cos\theta}
=\tan\theta,\quad 2b=(b+a)-(a-b)=\tan\theta-u\cos\theta.
$$

同理可得 $2b=\tan{(\theta+90^{\circ})-v\cos(\theta+90^{\circ})}
=-\cot\theta+v\sin\theta$, 两式相减得
$u\cos\theta+v\sin\theta=\tan\theta+\cot\theta
=\frac{\sin^{2}\theta+\cos^{2}\theta}{\sin\theta\cos\theta}$,
即$F=(\cos^{2}\theta\sin\theta)\times u+(\sin^{2}\theta\cos\theta)\times v=1$.

由于$\cos^{2}\theta\sin\theta=\sin\theta-\sin^{3}\theta
\leqslant\frac{2\sqrt{3}}{9}$,等号成立当且仅当$\sin\theta=\frac{\sqrt{3}}{3}$,这是因为
$$\sin\theta-\sin^{3}\theta-\frac{2\sqrt{3}}{9}
=-\left(\sin\theta-\frac{\sqrt{3}}{3}\right)^{2}\left(\sin\theta
+\frac{2\sqrt{3}}{3}\right)\leqslant 0.$$


同理 $\sin^{2}\theta\cos\theta=\cos\theta-\cos^{3}\theta
\leqslant\frac{2\sqrt{3}}{9}$, 等号成立当且仅当 $\cos\theta=\frac{\sqrt{3}}{3}$.

因此$F= ( \cos ^2\theta\sin \theta) \times u+ ( \sin ^{2}\theta\cos \theta) \times v\leqslant \frac {2\sqrt {3}}9u+ \frac {2\sqrt {3}}9v$,而$\sin\theta= \frac {\sqrt {3}}3$与$\cos\theta=\frac{\sqrt{3}}{3}$不能同时成立,不等式不能取等号,即$\frac{2\sqrt{3}}{9}u+\frac{2\sqrt{3}}{9}v>F=1$.

因此矩形周长$2(u+v)>2\times\frac{9}{2\sqrt{3}}=3\sqrt{3}$.

注:也可以利用均值不等式
$$\cos^2\theta\times\cos^2\theta\times2\sin^2\theta\leqslant\left(\frac{\cos^2\theta+\cos^2\theta+2\sin^2\theta}{3}\right)^3
=\left(\frac{2}{3}\right)^3=\frac{8}{27}$$
得到$\cos^{2}\theta\sin\theta\leqslant\frac{2\sqrt{3}}{9}$.

 

解法2.设点$A,B,C$的坐标分别为$\left(a,a^2+\frac14\right)$, $\left(b,b^2+\frac14\right)$, $\left(c,c^{2}+\frac{1}{4}\right)$.由于轨迹$W$关于$y$轴对称,不妨设 $b\geqslant 0$.由于$AB\perp BC$,直线$AB$, $BC$中恰有一条的斜率大于$0$,不妨设$AB$的斜率$k>0$,则直线$AB$的方程为$y=k(x-b)+b^{2}+\frac{1}{4}$,直线$BC$的方程为 $y=-\frac{1}{k}(x-b)+b^{2}+\frac{1}{4}$,代人$W$的方程可得$a=k-b$, $c=-\frac1k-b$ (由$a\neq b$知$k\neq 2b$).


则$|AB|=|k-2b|\sqrt{1+k^2}$, $|BC|=\left(2b+\frac{1}{k}\right)\sqrt{1+\frac{1}{k^2}}$,
矩形$ABCD$的周长为
$$L=2(|AB|+|BC|)=2\left(|k-2b|+\frac{2b}{k}
+\frac{1}{k^{2}}\right)\sqrt{1+k^{2}}.$$

(i)当$k<2b$时, $L>2\left(1+\frac{1}{k^{2}}\right)\sqrt{1+k^{2}}$, 记$x=\sqrt{1+k^{2}}$,
则$\left(1+\frac{1}{k^{2}}\right)\sqrt{1+k^{2}}=
\frac{x^3}{x^2-1}$.考虑函数$f(x)=\frac{x^3}{x^2-1}\ (x>1)$,由$f'(x)=\frac{x^4-3x^2}{(x^2-1)^2}$,可得$f(x)$在$\left(1,\sqrt{3}\right)$单调递减,在$\left(\sqrt{3},+\infty\right)$单调递增,在$x=\sqrt{3}$处取得最小值$\frac{3\sqrt{3}}{2}$.故
$L>2f\left(\sqrt{1{+}k^{2}}\right)\geqslant
2f\left(\sqrt{3}\right)=3\sqrt{3}$.

(ii)当$k>2b$时, $L=2\left(k-2b+\frac{2b} {k}+\frac{1}{k^{2}}\right)\sqrt{1+k^{2}}$.

若$k=1$,则$L=4\sqrt{2}>3\sqrt{3}$;

若$k>1$,则$L=2\left[1+\frac{1}{k^{2}}+(k-2b)\left(1-\frac{1}{k}\right)\right]
\sqrt{1+k^{2}}
>2\left(\:1+\frac{1}{k^{2}}\right)\sqrt{1+k^{2}}
\geqslant 3\sqrt{3}$;

若$k<1$,则$L=2\left(\frac{1}{k}+2b-2bk+k^{2}\right)\sqrt{1+\frac{1}{k^{2}}}
>2\left(1+k^{2}\right)\sqrt{1+\frac{1}{k^{2}}}
=2f\left(\sqrt{1+\frac{1}{k^2}}\right)\geqslant 3\sqrt{3}$.


综上,矩形$ABCD$的周长大于$3\sqrt{3}$.


注:也可以利用均值不等式得到
\begin{align*}
\left( 1+\frac{1}{k^2} \right) \sqrt{1+k^2} =\frac{\left( 1+k^2 \right) ^{3/2}}{k^2}=\frac{\left( 1+\frac{k^2}{2}+\frac{k^2}{2} \right) ^{3/2}}{k^2}
\geqslant \frac{\left[ 3\left( 1\cdot \frac{k^2}{2}\cdot \frac{k^2}{2} \right) ^{1/3} \right] ^{3/2}}{k^2}=\frac{3\sqrt{3}}{2}.
\end{align*}


解法3. 周长$L$的表达式及对$k<2b$情形、$k=1$情形的分析同解法2.

当$k>2b$时, $L=2\left(k-2b+\frac{2b} {k}+\frac{1}{k^{2}}\right)\sqrt{1+k^{2}}$.

若$k>1$,令$g(k)=k+\frac{2b}{k}-2b$.当$b\geqslant\frac{1}{2}$时, $g(k)$在区间$(2b,+\infty)$单调递增,所以 $g(k)>g(2b)=1$; 当$0<b<\frac{1}{2}$时, $g(k)$在区间$\left(2b,\sqrt{2b}\right)$单调递减,在区间$\left(\sqrt{2b},+\infty\right)$单调递增,所以$g(k)>g(1)=1$.故$L=2\left(k-2b+\frac{2b}{k}+\frac{1}{k^{2}}\right)\sqrt{1+k^{2}}
>2\left(1+\frac{1}{k^{2}}\right)
\sqrt{1+k^{2}}\geqslant 3\sqrt{3}$.


若$k<1$,令$h(k)=\frac{1}{k}-2bk+2b$, $h(k)$在区间$(2b,1)$单调递减,所以$h(k)>h(1)=1$.
故$L=2\left(k-2b+\frac{2b}{k}+\frac{1}{k^{2}}\right)\sqrt{1+k^{2}}
=2\left(\frac{1}{k}+2b-2bk+k^{2}\right)
\sqrt{1+\frac{1}{k^{2}}}>2(1+k^{2})\sqrt{1+\frac{1}{k^{2}}}
\geqslant3\sqrt{3}$.


解法4.设$A\left(a,a^{2}+\frac{1}{4}\right)$, $B\left(b,b^{2}+\frac{1}{4}\right)$, $D\left(d,d^{2}+\frac{1}{4}\right)$,
其中$a,b,d$互不相等,不妨设$a\geqslant 0$.

由$AB\perp AD$可知$(a-b,a^2-b^2)\cdot (a-d,a^2-d^2)=0$,化简得$(a+b)(a+d)=-1$.

因此$ABCD$的周长$L=2\left[ \sqrt{\left( a-b \right) ^2+\left( a^2-b^2 \right) ^2}+\sqrt{\left( a-d \right) ^2+\left( a^2-d^2 \right) ^2} \right]$,即
$$L=2\left[ \left| a-b \right|\sqrt{1+\left( a+b \right) ^2}+\left| a-d \right|\sqrt{1+\left( a+d \right) ^2} \right].$$

设$a+b=m,a+d=-\frac1m$,由对称性不妨设$0<m\leqslant 1$,则
\begin{align*}
L &=2\left[ \left| a-b \right|\sqrt{1+m^2}+\left| a-d \right|\sqrt{1+\frac{1}{m^2}} \right]\\
&\geqslant 2\sqrt{1+m^2}\left[ \left| a-b \right|+\left| a-d \right| \right] \geqslant 2\sqrt{1+m^2}\left| d-b \right|\\
&\geqslant 2\sqrt{1+m^2}\left( m+\frac{1}{m} \right),
\end{align*}
当且仅当$m=1$时取等号成立.又
$$
2\sqrt{1+m^2}\left( m+\frac{1}{m} \right) =2\frac{\left( 1+m^2 \right) ^{3/2}}{m}=2\frac{\left( \frac{1}{2}+\frac{1}{2}+m^2 \right) ^{3/2}}{m}\geqslant 2\frac{\left[ 3\left( \frac{1}{2}\cdot \frac{1}{2}\cdot m^2 \right) ^{1/3} \right] ^{3/2}}{m}=3\sqrt{3},
$$
当且仅当$m=\frac{\sqrt{2}}{2}$时取等号成立.由于两次取等条件不一致,因此 $ABCD$ 的周长$L$大于$3\sqrt{3}$.

已知$f(x)$是定义域为$\{x|x\neq0\}$的非常数函数,若对定义域内的任意实数$x,y$均有$f(x)f(y)=f(xy)+f\left(\frac{x}{y}\right)$,则下列结论正确的是

A. $f(1)=2$

B. $f\left(x\right)$的值域为$[2,+\infty)$

C. $f(x)=f(\frac 1x)$

D. $f(x)$是奇函数

取$f(x)=ax+\frac{a}{x}$.


已知关于$x$的不等式$\ln x+1\leqslant a\sqrt{x}e^{\frac{ax-1}2}$恒成立,则实数$a$的取值范围是\underline{\hspace{2cm}}.

 

 

 

 


若$(x-y)f(x+y)-(x+y)f(x-y)=4xy(x^2-y^2)$,求$f(x)$.

$\frac{f(x+y)}{x+y} -\frac{f(x-y)}{x-y} =4xy=(x+y)^2-(x-y)^2$,则
$\frac{f(x+y)}{x+y}-(x+y)^{2}=\frac{f(x-y)}{x-y}-(x-y)^{2}$, 令$x\to\frac{x+y}2$, $y\to\frac{x-y}2$,因此
$\frac{f(x)}x-x^2=\frac{f(y)}y-y^2=k$,
故$f(x)=x^{3}+kx$.

 

\begin{QsNum}
\item
\nandu{5}
\tiyuan{济南三模第2题:集合与简易逻辑}
\tigan{设集合$A=\{x|\ |x-1|\leqslant a\},B=\{x|x\leqslant 2a\}$,则“$a\geqslant 1$”是“$A\subseteq B$”的
\begin{tasks}(2)
\task 充分不必要条件 \task 必要不充分条件 \task 充要条件 \task 既不充分也不必要条件
\end{tasks}%D
}
\daan{}
\jiexi{选A.
$A\subseteq B$等价于$a<0$或$a\geqslant 1$.
}

\item
\nandu{5}
\tiyuan{济南三模第6题:抽象函数}
\tigan{已知函数$f(x)$的定义域为$\mathbf{R}$,且$yf(x)-xf(y)=xy(x-y)$,则下列结论一定成立的是\hfill (\quad)
\begin{tasks}(2)
\task $f(1)=1$
\task $f(x)$为偶函数
\task $f(x)$有最小值
\task $f(x)$在$[0,1]$上单调递增
\end{tasks}%C, $f(x)=x^2+kx$
}
\daan{}
\jiexi{
选C.
}

16.已知双曲线$C:\frac{x^2}{a^2}-\frac{y^2}{b^2}=1\ (a>0,b>0)$的左、右焦点分别为$F_1,F_2$.点$A$在$C$上,点$B$在$y$轴上, $\overrightarrow{F_1A}\perp\overrightarrow{F_1C},
\overrightarrow{F_2A}=-\frac23\overrightarrow{F_2B}$,
则$C$的离心率为\underline{\hspace{2cm}}.


\item
\nandu{5}
\tiyuan{济南三模第7题:双曲线与解三角形}
\tigan{已知双曲线$C:\frac{x^2}{a^2}-\frac{y^2}{b^2}=1\ (a>0,b>0)$的左、右焦点分别为$F_1,F_2$.点$P$在$C$上, $O$为原点,且$\overrightarrow{PF_1}\cdot \overrightarrow{PF_2}=2a^2,\left| \overrightarrow{PO} \right|=\sqrt{2}b$,则$C$的离心率为\hfill (\quad)
\begin{tasks}(4)
\task $\sqrt{2}$
\task $\sqrt{3}$
\task $2$
\task $3$
\end{tasks}%C, $e=2$
}
\daan{}
\jiexi{
选C.
}

 


\item
\nandu{5}
\tiyuan{济南三模第10题:函数综合}
\tigan{已知函数$f(x)=\sin (x)\cdot \ln x$, $M$为函数图象上一点, $O$为原点,则下列结论正确的是\hfill (\quad)
\begin{tasks}(1)
\task 曲线$y=f(x)$在$x=\pi$处的切线斜率为$\ln \pi$
\task 直线$OM$的斜率小于$\frac{1}{e}$
\task 方程$f(x)=100$有无数个实数根
% \task 过$O$能作曲线$y=f(x)$的无数条切线
\task 导函数$y=f'(x)$的图象在直线$y=x$下方
\end{tasks}%BCD, $e=2\mathrm{\pi}$
}
\daan{}
\jiexi{
选BCD.
}


(2009年全国卷高考)如图,已知抛物线$E:y^2=x$与圆$M:(x-4)^2+y^2=r^2\ (r>0)$相交于$A,B,C,D$四个点.

(l)求$r$的取值范围;

(II)当四边形$ABCD$的面积最大时,求对角线 $AC,BD$的交点$P$的坐标.


已知双曲线$xy=a\ (a>0)$与圆$x^2+y^2=4$相交于四个不同的点$A,B,C,D$,则$a$的取值范围为\underline{\hspace{2cm}},四边形$ABCD$面积的最大值为\underline{\hspace{2cm}}.


\item
\nandu{5}
\tiyuan{济南三模第14题:圆与抛物线}
\tigan{已知抛物线$x^2=2y$与圆$x^2+(y-4)^2=r^2\ (r>0)$相交于四个不同的点$A,B,C,D$,则$r$的取值范围为\underline{\hspace{2cm}},四边形$ABCD$面积的最大值为\underline{\hspace{2cm}}.
}
\daan{}
\jiexi{联立$x^2=2y$和$x^2+(y-4)^2=r^2$可得
$y^2-6y+16-r^2=0$,由$\Delta =36-4\left( 16-r^2 \right) >0$和$16-r^2>0$可得$\sqrt{7}<r<4$.

设$A(x_1,y_1),B(x_2,y_2)$为第一象限上两点,则$y_1+y_2=6,y_1y_2=16-r^2$,
因此
$$\left( \sqrt{y_1}+\sqrt{y_2} \right) ^2=y_1+y_2+2\sqrt{y_1y_2}=6+2\sqrt{16-r^2},$$
%$$\left( \sqrt{y_1}-\sqrt{y_2} \right) ^2=y_1+y_2-2\sqrt{y_1y_2}=6-2\sqrt{16-r^2},$$

$$
\left( y_2-y_1 \right) ^2=\left( y_1+y_2 \right) ^2-4y_1y_2=36-4\left( 16-r^2 \right) =4r^2-28,
$$

 

故四边形$ABCD$的面积为
\begin{align*}
S &=\frac{1}{2}\left( 2x_1+2x_2 \right) \left| y_2-y_1 \right|=\frac{1}{2}\left( 2\sqrt{2y_1}+2\sqrt{2y_2} \right) \left| y_2-y_1 \right|\\
&=\sqrt{2}\sqrt{6+2\sqrt{16-r^2}}\cdot \sqrt{4r^2-28}\\
&=4\sqrt{\left( 3+\sqrt{16-r^2} \right) \left( r^2-7 \right)}.
\end{align*}

令$u=\sqrt{16-r^2}$,由三元均值不等式可知
$$
\left( 3+\sqrt{16-r^2} \right) \left( r^2-7 \right) =\frac{1}{2}\left( 3+u \right) ^2\cdot 2\left( 3-u \right)\leqslant \frac{1}{2}\left( \frac{\left( 3+u \right) +\left( 3+u \right) +2\left( 3-u \right)}{3} \right) ^3=32,
$$
故$S\leqslant 4\sqrt{32}=16\sqrt{2}$.

当且仅当$r=\sqrt{15}$时,四边形$ABCD$面积取得最大值为$16\sqrt{2}$.
}

 

已知函数$f\left( x \right) =\left( a-x^2 \right) \ln \left( 1+x^2 \right) -ax^2\cos x$.

(1)求$f'(0)$和$f''(0)$;


(2)证明: $x=0$恒为$f(x)$的极大值点.

 


已知函数$f\left( x \right) =\left( a-x \right) \ln \left( 1+x \right) -ax\cos x$.

(1)求$f'(0)$和$f''(0)$;


(2)若$x=0$为$f(x)$的极大值点,求$a$的取值范围;
%$a>-2$


\item
\nandu{5}
\tiyuan{济南三模第18题:导数与微积分}
\tigan{极值的第三充分条件是利用函数的高阶导数对极值进行判定,在微积分中有重要的应用.记$f^{(n)}(x)$为函数$f(x)$的$n$阶导函数, $f'(x)=f^{(1)}(x)$且有$f^{(n)}(x)=\left[f^{(n-1)}(x)\right]'\ (n\in\mathbf{N}^\ast)$,若$f^{(n)}(x)$存在,则称$f(x)$是$n$阶可导的.设$f(x)$在定义域内$n$阶可导,且$f^{(k)}(x_0)=0\ (k=1,2,\cdots,n-1),f^{(n)}(x_0)\neq 0$,则

(i)当$n$为偶数时, $f$在$x_0$取得极值,且当$f^{(n)}(x_0)<0$时取极大值, $f^ {(n)}(x_0)>0$时取极小值.

(ii)当$n$为奇数时, $f$在$x_0$处不取极值.


已知函数$f\left( x \right) =ax\cos x-\left( x+a \right) \ln \left( x+1 \right)$.

(1)求$f'(0)$;


(2)若$x=0$为$f(x)$的极大值点,求$a$的取值范围;

(3)若$a=2$,证明:当$x>0$时, $f(x)<0$.
}
\daan{}
\jiexi{
(1)求导得$f'\left( x \right) =a\cos x-ax\sin x-\ln \left( x+1 \right) -\frac{x+a}{x+1}$,则$f'\left( 0 \right) =0$.

(2)又$f''\left( x \right) =-2a\sin x-ax\cos x-\frac{1}{x+1}-\frac{1-a}{\left( x+1 \right) ^2}$,
故$f''\left( 0 \right) =a-2$.

当$a<2$时, $x=0$为$f(x)$的极大值点.

当$a=2$时,由$f'''\left( x \right) =2x\sin x-6\cos x+\frac{x-1}{\left( x+1 \right) ^3}$可得$f'''\left( 0 \right) =-7$,此时$x=0$不是$f(x)$的极值点.


(3)若$a=2$,等价于证明$\ln \left( 1+x \right) >\frac{2x\cos x}{x+2},x>0$,只需证明$\ln \left( 1+x \right) >\frac{2x}{x+2},x>0$.

}

已知椭圆$E:\frac{x^2}{a^2}+\frac{y^2}{b^2}=1\ (a>0,b>0)$和抛物线$C:x^2=2p(y+m)\ (m>b,p>0)$,若从$C$上任一点$P$向椭圆$E$作两条切线分别交抛物线$C$于点$Q,R$,均有直线$QR$与$E$也相切,则称$C$为完美抛物线.若$C$为完美抛物线,求$a,b,p,m$满足的关系式.

$a^2=2p(m-b)$.

 

已知椭圆$E:\frac{x^2}{a^2}+\frac{y^2}{b^2}=1\ (a>0,b>0)$和圆$C:x^2+y^2=r^2\ (r>0)$,若从$C$上任一点$P$向椭圆$E$作两条切线分别交圆$C$于点$Q,R$,均有直线$QR$与$E$也相切,则称$C$为完美抛物线.若$C$为完美圆,求$a,b,r$满足的关系式.

$r=a+b$.

 

 


\item
\nandu{5}
\tiyuan{济南三模第19题:解析几何(抛物线与椭圆,开放性,充要条件)}
\tigan{已知椭圆$E:\frac{x^2}{2}+y^2=1$和抛物线$C:y=x^2-m\ (m>1)$.

(1)若$m=2$, $A$为抛物线的顶点,过点$A$作椭圆$E$的两条切线,求切线方程.

(2)若从$C$上任一点$P$向椭圆$E$作两条切线分别交抛物线$C$于另外两点$Q,R$,均有直线$QR$与$E$也相切,则称$C$为完美抛物线.

(i)是否存在$m$使得$C$为完美抛物线,若存在,求出$m$的值并进行证明;若不存在,请说明理由;

(ii)若将题设条件改为椭圆$E:\frac{x^2}{a^2}+\frac{y^2}{b^2}=1\ (a>0,b>0)$和抛物线$C:x^2=2p(y+m)\ (m>b,p>0)$,其它条件不变.是否存在$m$使得$C$为完美抛物线,若存在,试给出$a,b,p,m$满足的关系式,不需要证明;若不存在,请说明理由.
}
\daan{}
\jiexi{
(1)由题意可知$A(0,-2)$,设切线方程为$y=kx-2$,联立椭圆方程$\frac{x^2}{2}+y^2=1$可得$\left( 2k^2+1 \right) x^2-8kx+6=0$.
由$\Delta =\left( 8k \right) ^2-24\left( 2k^2+1 \right) =8\left( 2k^2-3 \right)$可得$k=\pm \frac{\sqrt{6}}{2}$.故切线方程为$y=\pm \frac{\sqrt{6}}{2}x-2$.

(2)取抛物线的顶点$A(0,-m)$,设此时切线方程为$y=kx-m$,联立椭圆方程$\frac{x^2}{2}+y^2=1$可得$\left( 2k^2+1 \right) x^2-4kmx+2m^2-2=0$.
由$\Delta =\left( 4km \right) ^2-4\left( 2k^2+1 \right) \left( 2m^2-2 \right) =8\left( 2k^2-m^2+1 \right)$
可得$2k^2+1=m^2$.显然此时切线与$C$交点的纵坐标为$1$,即一个交点为$M\left( \sqrt{m+1},1 \right)$,此时
$k_{AM}=\frac{1+m}{\sqrt{m+1}}=\sqrt{m+1}$.

于是$m^2=2k^2+1=2(m+1)+1=2m+3$,解得$m=3$或$-1$ (舍去).

设$P(x_1,y_1),Q(x_2,y_2),R(x_3,y_3)$,则$y_1=x_1^2-3$, $y_2=x_2^2-3$, $y_3=x_3^2-3$.

直线$PQ$的方程为$y-y_1=\frac{y_2-y_1}{x_2-x_1}\left( x-x_1 \right)$,
即$y=\left( x_1+x_2 \right) x-x_1x_2-3$.

联立椭圆方程$\frac{x^2}{2}+y^2=1$可得
$$
\left[ 2\left( x_1+x_2 \right) ^2+1 \right] x^2-4\left( x_1+x_2 \right) \left( x_1x_2+3 \right) x+2\left( x_1x_2+3 \right) ^2-2=0.
$$

\begin{align*}
\Delta &=\left[ 4\left( x_1+x_2 \right) \left( x_1x_2+3 \right) \right] ^2-4\left[ 2\left( x_1+x_2 \right) ^2+1 \right] \left[ 2\left( x_1x_2+3 \right) ^2-2 \right]\\
&=8\left( 2x_{1}^{2}+2x_{2}^{2}-x_{1}^{2}x_{2}^{2}-2x_1x_2-8 \right)=0
\end{align*}
可得$2x_{1}^{2}+2x_{2}^{2}-x_{1}^{2}x_{2}^{2}-2x_1x_2-8=0$,

$$
2x_1x_2+y_1y_2+y_1+y_2+5=0.
$$
同理可得
$$
2x_1x_3+y_1y_3+y_1+y_3+5=0.
$$
因为直线方程$2x_1x+(y_1+1)y+y_1+5=0$同时经过点$Q,R$,所以$QR$的直线方程为$2x_1x+(y_1+1)y+y_1+5=0$.

联立椭圆方程$\frac{x^2}{2}+y^2=1$可得
$$
\left[ 8x_{1}^{2}+\left( y_1+1 \right) ^2 \right] x^2+8x_1\left( y_1+5 \right) x+16y_1+48=0.
$$

于是
\begin{align*}
\Delta &=\left[ 8x_1\left( y_1+5 \right) \right] ^2-4\left[ 8x_{1}^{2}+\left( y_1+1 \right) ^2 \right] \left( 16y_1+48 \right)\\
&=64\left( y_1+1 \right) ^2\left( x_{1}^{2}-y_1-3 \right) =0.
\end{align*}
故直线$QR$与椭圆相切,因此$m=3$.

(3) $a^2=2p(m-b)$.
}
注:点$(a,-b)$需要落在抛物线上.

 


(深圳中学高三二轮二阶测试)已知曲线 $y=\ln x+a$ ($a\in\mathbb{R})$ 和圆 $O:x^2+y^2=r^2\ \left(r\in\mathbb{R},r>0\right)$相交于$A,B$两个不同点,记直线$AB$的斜率为 $k$.

(1)当$r=\sqrt 2$时,证明: $a>-1$;

(2)当$r=\frac{\sqrt {3}}2$时,证明: $k>\sqrt{2}$.

 

(1)当$r=\sqrt{2}$时,由题意得, $x^2+\left(\ln x+a\right)^2=2$有两个不同零点.

设函数$g(x)=x^2+(\ln x+a)^2-2$,
$g'(x)=2x+{\frac{2(\ln x+a)}{x}}={\frac{2}{x}}(x^{2}+\ln x+a)$.


易知$\varphi(x)=x^2+\ln x+a$在$x>0$时单调递增且$\lim_{x\to 0}\varphi(x)=-\infty$, $\lim_{x\to+\infty}\varphi(x)=+\infty$.

由零点存在定理知,存在$x_0>0$,使得$x_0^2+\ln x_0+a=0$.

当$x\in(0,x_0)$时, $g'(x)<0$; 当$x\in(x_0,+\infty)$ 时, $g'(x)>0$.所以函数$g(x)$在$(0,x_0)$递减,在$(x_0,+\infty)$递增.

又因为$\lim_{x\to 0^+}g(x)=\lim_{x\to +\infty}g(x)=+\infty$,所以$g(x)$有两个不同零点等价于 $g(x_0)<0$,即$x_0^2+(\ln x_0+a)^2-2=x_0^2+x_0^4-2<0$,解得$x_0^2<1$,
所以$a=-x_{0}^{2}-\ln x_{0}>-1$.

(2)取线段$AB$中点$E$,要证明$k>\sqrt{2}$,只需证明$k_{OE}>-\frac{\sqrt{2}}{2}$.设$OA$顿斜角为$\theta_{1}$, $OB$倾斜角为$\theta_{2}$, $\theta_{1},\theta_{2}\in(-\frac{\pi}{2},\frac{\pi}{2})$
则$OE$倾斜角为$\frac{\theta_{1}+\theta_{2}}{2}$,只需证明$\tan\frac{\theta_{1}+\theta_{2}}{2}>-\frac{\sqrt{2}}{2}$.

因为$A\left(\frac{\sqrt{3}}{2}\cos\theta_1,\frac{\sqrt{3}}{2}\sin\theta_1\right)$, $B\left(\frac{\sqrt{3}}{2}\cos\theta_2,
\frac{\sqrt{3}}{2}\sin\theta_2\right)$在曲线 $y=\ln x+a$上.

所以$\theta_{1},\theta_{2}$为方程$\frac{\sqrt{3}}{2}\sin\theta-\ln\left(\frac{\sqrt{3}}{2}
\cos\theta\right)-a=0$在$\left(-\frac{\pi}{2},\frac{\pi}{2}\right)$的两根.

令$h(\theta)={\frac{\sqrt{3}}{2}}\sin\theta-\ln\left({\frac{\sqrt{3}}{2}}
\cos\theta\right)-a$,
则$h'(\theta)=\frac{\sqrt{3}}{2}\cos\theta+\frac{\sin\theta}{\cos\theta}
=\frac{(1+\sqrt{3}\sin\theta)(\sqrt{3}-\sin\theta)}
{2\cos\theta}$.

记$\theta_{0}\in(-\frac{\pi}{2},\frac{\pi}{2})$满足$\sin\theta_{0}=-\frac{\sqrt{3}}{3}$,
则$\tan\theta_{0}=-\frac{\sqrt{2}}{2},
\cos\theta_{0}=\frac{\sqrt{6}}{3}$.

易知$h(\theta)$在$\left(-\frac{\pi}{2},\theta_{0}\right)$递减, $\left(\theta_ {0},\frac{\pi}{2}\right)$递增,因此$-\frac{\pi}{2}<\theta_{1}<\theta_{0}<\theta_{2}<\frac{\pi}{2}$.

要证$\tan\frac{\theta_{1}+\theta_{2}}{2}>-\frac{\sqrt{2}}{2}=\tan\theta_{0}$,只需证$\frac{\theta_{1}+\theta_{2}}{2}>\theta_{0}$,
即$\theta_2>2\theta_0-\theta_1$,因为$\theta_2,2\theta_0-\theta_1\in \left(\theta_0,\frac\pi2\right)$,
所以只需证明 $h(\theta_1)=h(\theta_2)>h(2\theta_0-\theta_1)$.

构造函数$H(\theta)=h(\theta)-h(2\theta_0-\theta)$, $\theta\in\left(-\frac{\pi} {2},\theta_0\right)$,

\begin{align*}
H'(\theta) &=h'(\theta)+h'(2\theta_{0}-\theta)
=\frac{\sqrt{3}}{2}\cos\theta
+\frac{\sin\theta}{\cos\theta}
+\frac{\sqrt{3}}{2}\cos(2\theta_{0}-\theta)
+\frac{\sin(2\theta_{0}-\theta)}
{\cos(2\theta_{0}-\theta)}\\ &=\sqrt{3}\cos\theta_0\cos(\theta_0-\theta)
+\frac{\sin2\theta_0}{\cos\theta
\cos(2\theta_0-\theta)}\quad \text{(和差化积公式)}\\&=\sqrt{3}\cos\theta_{0}\cos(\theta_{0}-\theta)
+\frac{2\sin2\theta_{0}}{\cos2\theta_{0}
+\cos(2\theta_{0}-2\theta)}\quad \text{(积化和差公式)}\\
&=\sqrt{2}\cos(\theta_{0}-\theta)
-\frac{2\sqrt{2}}{3\cos^{2}(\theta_{0}-\theta)-1}
=\sqrt{2}\frac{3\cos^{3}(\theta_{0}-\theta)
-\cos(\theta_{0}-\theta)-2}{3\cos^{2}
(\theta_{0}-\theta)-1}.
\end{align*}

令$t=\cos(\theta_{0}-\theta)\in \left(\frac{\sqrt{3}}{3},1\right)$,
则$H'(\theta)=\sqrt{2}\frac{3t^{3}-t-2}{3t^{2}-1}
=\sqrt{2}\frac{(t-1)(3t^{2}+2t+2)}{3t^{2}-1}<0$,
所以$H(\theta)$在$\left(-\frac{\pi}{2},\theta_{0}\right)$单调递减,因此$H(\theta)>H(\theta_{0})=0$.

(1)令$f(x)=\ln x+a+\sqrt{r^{2}-x^{2}}$,则
$f(x)$在$(0,r)$上有两个不相等的零点,
$$f'(x)=\frac{1}{x}-\frac{x}{\sqrt{r^{2}-x^{2}}}
=\frac{\sqrt{r^{2}-x^{2}}-x^{2}}{x\sqrt{r^{2}-x^{2}}},$$
令$f'(x)=0$,
则$x=\sqrt{\sqrt{r^2+\frac14}-\frac12}$,
记$x_0=\sqrt{\sqrt{r^2+\frac14}-\frac12}$,则
$\forall x\in(0,x_0)$, $f'(x)>0$;
$\forall x\in(x_0,r),f'(x)<0$,
所以$f(x)$在$(0,x_0)$上单调递增,在$(x_0,r)$上单调递减,
所以$f(x)$在$(0,r)$上的最大值为$\ln x_0+x_0^2+a$,所以$a>-x_0^2+\ln x_0$,
故$a>-\sqrt{r^2+\frac14}
+\frac12\ln\left(\sqrt{r^2+\frac14}-\frac12\right)
+\frac12$.

(2)记$AB$中点为$M(x_0,y_0)$,由$AB$为圆$O$的弦可得
$$
y_0=-\frac{x_0}k,\quad (1)
$$
$$
x_0^2+y_0^2<r^2,\quad (2)
$$

过$M$引$x$轴的垂线与曲线$y=\ln x+a$交于点$N$,则
$$
k=k_{AB}>k_N=\frac1{x_0},\quad (3)
$$

由(1)、(2)、(3)可得
$$\left(1+\frac{1}{k^{2}}\right)\frac{1}{k^{2}}<r^{2},
\quad (4),$$
由(4)可得
$$k>\frac{1} {\sqrt{\sqrt{r^{2}+\frac{1}{4}}-\frac{1}{2}}}.$$

 


已知曲线 $E:y=e^x+a$ ($a\in\mathbf{R})$ 和圆 $C:x^2+y^2=4$, $O$为原点.

(1)若$E$与$C$相交于$A,B$两个不同点,证明: $<a<\sqrt{2}$;

精确值为$a=1.3838,x=-0.81173$或$-7.52502,x=1.9813$.

(2)若$E$与$C$恰有一个公共点$P$,记直线$OP$的斜率为 $k$,证明: $k>\sqrt{2}$或.

考虑函数$f(x)=\sqrt{4-x^2}-e^x$,则$f'\left( x \right) =-\frac{x}{\sqrt{4-x^2}}-e^x$.

又$f'(x)$单调递减,且$f'\left( -1 \right) =\frac{1}{\sqrt{3}}-\frac{1}{e}>0,f'\left( -\frac{1}{2} \right) =\frac{1}{\sqrt{15}}-\sqrt{e}>0$,故存在$m\in \left(-1, -\frac{1}{2} \right)$,使得
$$
f\left( m \right) =\sqrt{4-m^2}-e^m=\sqrt{4-m^2}+\frac{m}{\sqrt{4-m^2}}\in \left( \frac{2\sqrt{3}}{3},\frac{13\sqrt{15}}{30} \right).
$$

考虑函数$g(x)=-\sqrt{4-x^2}-e^x$,则$g'\left( x \right) =\frac{x}{\sqrt{4-x^2}}-e^x$.

故$g'\left( \frac{3}{2} \right) =\frac{3}{\sqrt{7}}-e^{3/2}<0$,则$n\in \left(\frac{3}{2},2 \right)$,
于是
$g\left( n \right) =-\sqrt{4-n^2}-e^n\in \left( -\frac{\sqrt{7}}{2}-e^{3/2},-e^2 \right)$.

 


$k=-2.25182$或$-0.137716$.

3. 设$a,b,c,d$是4个不等于1的正数,满足$abcd=1$,问:
$$
a^{2010}+b^{2010}+c^{2010}+d^{2010}\quad\text{和}\quad a^{2011}+b^{2011}+c^{2011}+d^{2011}
$$
哪个数大?为什么?

 

\item
\nandu{5}
\tiyuan{济南三模第18题:导数与微积分}
\tigan{已知函数$f\left( x \right) =a^x+2^x+3^x+\cdots +n^x-n$,其中$a>0$且$a\neq 1$, $n\geqslant 2, n\in \mathbf{N}^\ast$.

(1)求$f(0)$;

(2)若$f(x)>0$在$(0,+\infty)$上恒成立,求$a$的最小值;

(3)记(2)中$a$的最小值为$a_n$且$a_1=1$,数列$\{a_n\}$的前$n$项和为$S_n$,证明: $1+S_n<\mathrm{e}$.
}
\daan{}
\jiexi{(1) $f(0)=0$;

(2)由题意可知$f'\left( x \right) =a^x\ln a+2^x\ln 2+3^x\ln 3+\cdots +n^x\ln n$.

记$g\left( x \right) =a^x\ln a+2^x\ln 2+3^x\ln 3+\cdots +n^x\ln n$,则$g'\left( x \right) =a^x\ln^2 a+2^x\ln^2 2+3^x\ln^2 3+\cdots +n^x\ln^2 n>0$,
故$f'\left( x \right)=g(x)$在$(0,+\infty)$上单调递增,则$f'(x)>f'(0)=\ln (n!a)$.


要使$f(x)>0$在$(0,+\infty)$上恒成立,因为$f(0)=0$,
只需保证$f'(0)=\ln (n!a)\geqslant 0$,即$a\geqslant \frac{1}{n!}$.


(3)由(2)可知$a_n=\frac{1}{n!},n\geqslant 2$,又$a_1=1$,则$a_n=\frac{1}{n!},n\in \mathbf{N}^\ast$.

考虑函数$F\left( x \right) =\mathrm{e}^{-x}\left( 1+\frac{x}{1!}+\frac{x^2}{2!}+\cdots +\frac{x^n}{n!} \right),x>0$,则$F'\left( x \right) =-\mathrm{e}^{-x}\cdot \frac{x^n}{n!}<0$,

故$F(x)$在$(0,+\infty)$上单调递减,故$F(x)<F(0)=1$,
则$\mathrm{e}^{-x}\left( 1+\frac{x}{1!}+\frac{x^2}{2!}+\cdots +\frac{x^n}{n!} \right) <1$,
于是$1+\frac{x}{1!}+\frac{x^2}{2!}+\cdots +\frac{x^n}{n!}<\mathrm{e}^x$.

令$x=1$可得$1+\frac{1}{1!}+\frac{1}{2!}+\cdots +\frac{1}{n!}<\mathrm{e}^x$,因此$1+S_n<\mathrm{e}$.
}

 

 


\item
\nandu{5}
\tiyuan{济南三模第18题:导数与微积分}
\tigan{12
}
\daan{}
\jiexi{
1
}

[昆明一中2021届高三12月月考]春节期间某网络支付平台开展集“福”字活动:共有5种不同的“福”字电子卡,每完成一笔网络支付交易就能随机获赠一张“福”字卡,集齐$5$张不同的“福”字卡即可获奖.某网购平台上购买一袋干脆面,内随赠一张水浒传一百单八将的好汉卡,集齐完整一套好汉卡将获得生产商颁发的大奖(好汉卡一套共$108$张,每张上画有一将,每将都有很多张).

(1) 若每完成一笔网络支付交易获赠每种“福”字卡的可能性相同.

\ding{172} 求获得第二种“福”字卡的概率;

\ding{173} 平均要完成多少笔交易才能集齐$5$个不同的“福”字卡?


(2)如果购买一袋干脆面随赠一张一百单八将的好汉卡中每一张的可能性是一样的,那么平均要购买多少袋干脆面才能获得生产商颁发的大奖? (结果保留到整数)


参考信息:

(i)如果在一次试验中某事件发生的概率是$p$,那么在独立重复试验中,某事件第$1$次发生时所作试验的次数$\xi$的概率分本
$P(\xi=k)=p(1-p)^{k-1}\ (k=1,2,3,\cdots)$,称$\xi$服从几何分布,记作$\xi\sim G(k,p)$; $\xi$的数学期望$E(\xi)=\frac1p$;


(ii)若干个相互独立、且是按先后次序依次连续发生的随机变量之和的数学期望等于这些随机变量数学期望之和;


(iii) $\sum _{i= 1}^{n}\frac 1i\approx \ln n+ 0.577$, $\ln 108\approx 4. 68$.


(1)由题意知: $5$种福卡获赠的概率均为$\frac{1}{5}$,第一次所获福卡$1$,则后续获除福卡$1$外其它福卡的概率为$\frac{4}{5}$;若获得福卡$2$,则后续获除福卡$1$、$2$外其它福卡的概率为$\frac{3}{5}$;若获得福卡$3$,则后续获除福卡$1$、$2$、$3$外其它福卡的概率为$\frac{2}{5}$;若获得福卡$4$,则后续获除福卡$1$、$2$、$3$、$4$外的福卡的概率为$\frac{1}{5}$;

\ding{172} 由上知:获得第二种福卡的概率为$\frac{4}{5}$;

\ding{173} 若$X$表示抽到$5$种不同福卡所完成的交易数,则 $X_i\ (i=1, 2, 3, 4, 5)$为获得第$i-1$张不同福卡后,重计交易次数直到获得第$i$张不同福卡,所以集齐$5$个不同的“福”字卡所需的总交易次数为$X=X_1+X_2+X_3+X_4+X_5$,其中$X_1=1$,
由题意, $E(X_2)=\frac1{p_2}=\frac54$, $E(X_3)=\frac1{p_3}=\frac53$, $E(X_4)=\frac1{p_4}=\frac52$, $E(X_5)=\frac1{p_5}=5$,

故$E(X)=E(X_{1}+X_{2}+X_{3}+X_{4}+X_{5})
=E(X_{1})+E(X_{2})+E(X_{3})+E(X_{4})+E(X_{5})
=\frac{137}{12}$,而$12>\frac{137}{12}>11$,
故平均至少要完成$12$笔交易才能集齐$5$个不同的“福”字卡.


(2)同(1),若$X$表示抽到$108$种不同好汉卡所购买的次数,则$X_i\ (i=1,2,3,\cdots,108)$为获得第$i-1$张不同好汉卡后,重计
交易次数直到获得第$i$张不同好汉卡,所以集齐$108$个不同好汉卡所需的总购买次数为$X=X_1+X_2+X_3+\cdots+X_{108}$,其中$X_1=1$,
当抽到第$i-1$张不同好汉卡后,后续获得第$i$张不同好汉卡的概率为$p_i=\frac{108-i+1}{108}=\frac{109-i}{108}$,

故$E(X)=E(X_{1}+\cdots+X_{108})=E(X_{1})+\cdots+E(X_{108})$,

于是$E\left(X\right)=1+\frac{1}{p_{2}}+\cdots+\frac{1}{p_{108}}
=108\left(1+\frac{1}{2}+\cdots+\frac{1}{108}\right)
=108\sum_{i=1}^{108}\frac{1}{i}\approx 108\left(\ln108+0.577\right)\approx 568$,所以平均要购买$568$袋干脆面才能获得生产商颁发的大奖.


[2020届西城区高三一模]对于正整数$n$,如果$k\ (k\in \mathbf{N}^\ast)$个整数$a_1,a_2,\cdots,a_k$满足$1\leqslant a_1\leqslant a_2\leqslant\cdots\leqslant a_k\leqslant n$,且$a_1+a_2+\cdots+a_k=n$,则称数组$(a_1,a_2,\cdots,a_k)$为$n$的一个“正整数分拆”. 记$a_1,a_2,\cdots,a_k$均为偶数的“正整数分拆”的个数
为$f_n$, $a_1,a_2,\cdots,a_k$均为奇数的“正整数分拆”的个数为$g_n$.

(I)写出整数$4$的所有“正整数分拆”;

(II)对于给定的整数$n\ (n\geqslant 4)$,设$(a_1,a_2,\cdots,a_k)$是$n$的一个“正整数分拆”,且$a_1=2$,求$k$的最大值;

(III)对所有的正整数$n$,证明: $f_n\leqslant g_n$,并求出使得等号成立的$n$的值.

(注:对于$n$的两个“正整数分拆” $(a_1,a_2,\cdots,a_k)$与$(b_1,b_2,\cdots,b_m)$,当且仅当$k=m$且$a_1=b_1,a_2=b_2,\cdots,a_k=b_m$时,称这两
个“正整数分拆”是相同的.)

(I)整数的所有“正整数分拆”为: $(1,1,1,1),(1,1,2),(1,3),(2,2),(4)$.


(II)当$n$为偶数时, $a_1=a_2=a_3=\cdots=a_k=2$时, $k$最大为$k={n}{2}$;

当$n$为奇数时, $a_1=a_2=a_3=\cdots=a_{k-1}=2,a_k=3$时, $k$最大为$k={n-1}{2}$;

综上所述: $n$为偶数时, $k$最大为$k={n}{2}$; $n$为奇数时, $k$最大为$k={n-1}{2}$.

(III)当$n$为奇数时, $f_n=0$,至少存在一个全为$1$的拆分,故$f_n<g_n$;

当$n$为偶数时,设$(a_1,a_2,\cdots,a_k)$是每个数均为偶数的“正整数分拆”,则它至少对应了$(1,1,\cdots,1)$和 $(1,1,\cdots, a_{1}-1,a_{2}-1,\cdots,a_{k}-1)$的均为奇数的“正整数分拆”, 故$f_{n}\leqslant g_{n}$.

综上所述: $f_{n}\leqslant g_{n}$.


当$n=2$时,偶数“正整数分拆”为$(2)$,奇数“正整数分拆”为$(1,1)$, $f_2=g_2=1$;

当$n=4$时,偶数“正整数分拆”为$(2,2),(4)$,奇数“正整数分拆”为$(1,1,1,1),(1,3)$,故$f_4=g_4=2$;


当$n\geqslant 6$时,对于偶数“正整数分拆”,除了各项不全为$1$的奇数拆分外,至少多出一项各项均为$1$的“正整数分拆”,故$f_n<g_n$.

综上所述:使$f_n=g_n$成立的$n$为: $n=2$或$n=4$.

[东城区2019届高三期末]
对给定的$d\in \mathbf{N}^\ast$,记由数列构成的集合$\Omega(d)=\left\{\left\{a_n\right\}\mid a_1=1,\left|a_ {n+1}\right|=\left|a_n+d\right|,n\in \mathbf{N}^\ast\right\}$.

(1) 若数列$\{a_n\} \in \Omega (2)$,写出$a_3$的所有可能取值;

(2)对于集合$\Omega\left(d\right)$,若$d\geqslant 2$. 求证:存在整数$k$,使得对$\Omega\left(d\right)$中的任意数列$\{a_n\}$,整数$k$不是数列$\{a_n\}$中的项;


(3)已知数列$\{a_n\},\{b_n\}\in\Omega(d)$,
记$\{a_n\},\{b_n\}$的前$n$项和分别为$A_n,B_n$. 若$|a_n+1|\leqslant |b_{n+1}|$,求证: $A_n\leqslant B_n$.


(1)由于数列$\{a_n\}\in\Omega(2)$,即$d=2$, $a_1=1$.


由已知有$|a_2|=|a_1+d|=|1+2|=3$,所以$a_2=\pm 3$,
$|a_3|=|a_2+d|=|a_2+2|$,将$a_2=\pm 3$代入得$a_{3}$的所有可能取值为$-5,-1,1,5$.


证明: (2)先应用数学归纳法证明数列:

若$\{a_{n}\}\in\Omega (d)$,则$a_{n}$具有$md\pm 1,(m\in \mathbf{Z})$的形式.


\ding{172} 当$n= 1$时, $a_1=0\cdot d+ 1$,因此$n=1$时结论成 立 .


\ding{173} 假设当$n=k\ (k\in\mathbf{N}^\ast)$时结论成立,即存在整数$m_0$,使得$a_k=m_0d\pm 1$成立.

当$n=k+1$时, $|a_{k+1}|=|m_{0}d\pm 1+d|=|(m_{0}+1) d\pm 1|$,则$a_{k+1}=(m_0+1)d\pm 1$,或$a_{k+1}=-(m_0+1)\pm 1$,
所以当$n=k+1$时结论也成立.

由\ding{172} \ding{173}可知,若数列$\{a_{n}\}\in\Omega (d)$,对任意 $n\in \mathbf{N}^\ast$, $a_n$具有$md\pm 1,(m\in \mathbf{Z})$的形式 .

由于$a_n$具有$md\pm 1,(m\in \mathbf{Z})$的形式,以及$d\geqslant 2$,可得$a_n$不是$d$的整数倍.

故取整数$k=d$,则整数$k$均不是数列$\{a_{n}\}$中的项.


(3)由$|a_{n+1}|=|a_{n}+d|$可得: $a_ {n+1}^{2}=a_{n}^{2}+2a_{n}d+d^{2}$,
所以有$a_{n+1}^{2}=a_{n}^{2}+2a_{n}d+d^{2}$, $a_{n}^{2}=a_{n-1}^{2}+2a_{n-1}d+d^{2}$,
$a_{n-1}^{2}=a_{n-2}^{2}+2a_{n-2}d+d^{2}$,

$\cdots$
$a_{2}^{2}=a_{1}^{2}+2a_{1}d+d^{2}$,

以上各式相加可得$a_{n+1}^{2}-1=d_{n}^{2}n+2S_{n}+2S_{n}d$,

即$A_{n}=\frac{a_{n+1}^{2}}{2d}-\frac{nd^{2}+1}{2d}$,
同理$B_{n}=\frac{b_{n+1}^{2}}{2d}-\frac{nd^{2}+1}{2d}$,

当$|a_{n+1}|\leqslant |b_{n+1}|$时,有$a_{n+1}^{2}\leqslant b_{n+1}^{2}$,

由于$d\in \mathbf{N}^\ast$, 所以$\frac{a_{n+1}^{2}}{2d}\leqslant \frac{b_{n+1}^{2}}{2d}$,
于是$\frac{a_{n+1}^{2}}{2d}-\frac{nd^{2}+1}{2d}\leqslant
\frac{b_{n+1}^{2}+1}{2d}-\frac{nd^{2}+1}{2d}$,
即$A_{n}\leqslant B_{n}$.

 


[详解](1)由$a_1<a_2<\cdots<a_n$,且均为正整数,故$a_1\geqslant 1,a_2\geqslant 2,a_3\geqslant 3$.故$S(A)\geqslant 6$.故当不大于$S(A)$的正整数$k=1$时,由题意必有
$a_1=1$,当$k=2$时,可得$a_2=2$.


(2)先证必要性:


因为$a_1=1,a_2=2$,又$a_1,a_2,\cdots,a_n$成等差数列,故$a_n=n$,所以$S(A)=\frac{n(n+1)}2$;


再证充分性:

因为$a_1<a_2<\cdots<a_n$, $a_1,a_2,\cdots,a_n$为正整数数列,故有
$a_1= 1, a_2= 2, a_3\geqslant 3$, $a_4\geqslant 4, \cdots , a_n\geqslant n$,

所以$S(A)=a_1+a_2+\cdots + a_n\geqslant 1+ 2+ \ldots + n= \frac {n( n+ 1) }2$,

又$S(A)=\frac{n(n+1)}2$,故$a_m=m\ (m=1,2,\cdots,n)$,故 $a_1,a_2,\cdots,a_n$为等差数列.


(3)先证明$\forall a_{m}\leqslant 2^{m-1}\ \left (m=1,2,\cdots , n\right)$.


假设存在$a_p>2^{p-1}$,且$p$为最小的正整数.


依题意$p\geqslant 3$,则$a_1+a_2+\cdots+a_{p-1}\leqslant 1+2+\ldots+2^{p-2}=2^{p-1}-1$,又因为$a_1<a_2<\cdots<a_n$,

故当$k\in(2^{p-1}-1,a_p)$时, $k$不能等于集合$A$的任何一个子集所有元素的和.故假设不成立,即$\forall a_m\leqslant 2^{m-1}\ (m=1,2,\cdots,n)$成立.

因此$2024=a_1+a_2+\cdots+a_n\leqslant 1+2+\ldots+2^{n-1}=2^n-1$, 即$2^n\geqslant 2025$,所以$n\geqslant 11$.


因为$S=2024$,则$a_1+a_2+\cdots+a_{n-1}=2024-a_n$,


若$2024-a_n<a_n-1$时,则当$k\in (2024-a_n,a_n)$时,集合$A$中不可能存在若干不同元素的和为$k$,

故$2024-a_n\geqslant a_n-1$,即$a_n\leqslant 1012.5$.

此时可构造集合$A=\left\{1,2,4,8,16,32,64,128,256,501,1012\right\}$.

因为当$k\in\{2,2+1\}$时, $k$可以等于集合$\{1,2\}$中若干个元素的和;

故当$k\in\left\{2^2,2^2+1,2^2+2,2^2+3\right\}$时, $k$可以等于集合$\{1,2,2^2\}$中若干不同元素的和;

$\cdots\cdots$


故当$k\in\{2^8,2^8+1,2^8+2,\cdots,2^8+255\}$时, $k$可以等于集合$\{1,2,\cdots,2^8\}$中若干不同元素的和;


故当$k\in\left\{501+1,501+2,\cdots,501+511\right\}$时, $k$可以等于集合$\{1,2,\cdots,2^8,501\}$中若干不同元素的和;

故当$k\in\{1012,1012+1,1012+2,\cdots,1012+1012\}$时, $k$可以等于集合$\{1,2,\cdots,2^8,501,1012\}$中若干不同元素的和,

所以集合$A=\left\{1,2,4,8,16,32,64,128,256,501,1012\right\}$满足题设,

所以当$n$取最小值$11$时, $a_{n}$的最大值为$1012$.

[2009年湖南理科]
对于数列$\left\{u_n\right\}$,若存在常数 $M>0$,对任意的 $n\in\mathbf{N}^\ast$,恒有$|u_{n+1}-u_n|+| u_n-u_{n-1}|+\cdots+| u_2-u_1|\leqslant M$,则称数列$\left\{u_n\right\}$为 $B-$数列.


(I)首项为$1$,公比为 $q\ (|q|<1)$的等比数列是否为
$B-$数列?请说明理由;


(II)设 $S_n$是数列$\left\{x_n\right\}$的前$n$项和.给出下列两组
论断:

$A$组: \ding{172} 数列$\left\{x_n\right\}$是$B-$数列, \ding{173}数列$\left\{x_n\right\}$不是 $B-$数列;

$B$组: \ding{174} 数列$\left\{S_n\right\}$是$B-$数列, \ding{175} 数列$\left\{S_n\right\}$不是 $B-$数列.

请以其中一组中的一个论断为条件,另一组中的一个论断为结论组成一个命题.判断所给命题的真假,并证明你的结论;


(III)若数列$\{a_n\},\{b_n\}$都是$B-$数列,证明:数列$\{a_nb_n\}$也是$B-$数列.


解: (I)设满足题设的等比数列为$\left\{a_{n}\right\}$,则$a_n=q^{n-1}$.
于是$\left|a_n-a_{n-1}\right|=\left|q^{n-1}-q^{n-2}\right|
=\left|q\right|^{n-2}\left|q-1\right|,n\geqslant 2$.

因此$|a_{n+1}-a_n|+|a_n-a_{n-1}|+\cdots+|a_2-a_1|
=|q-1|(1+|q|+|q|^{2}+\cdots+|q|^{n-1})$.

因为$|q|<1$,所以$1+|q|+|q|^2+\cdots+|q|^{n-1}=\frac{1-|q|^n}{1-|q|}<\frac1{1-|q|}$.

即$|a_{n+1}-a_n|+|a_n-a_{n-1}|+\cdots+|a_2-a_1|<\frac{|q-1|}{1-|q|}$.

故首项为$1$,公比为 $q\ (|q|<1)$的等比数列是$B-$数列.


(II)命题 1:若数列$\left\{x_n\right\}$是$B-$数列,则数列$\left\{S_n\right\}$是$B-$数列.

此命题为假命题.

事实上,设 $x_n=1,n\in\mathbf{N}^\ast$,易知数列$\left\{x_{n}\right\}$是$B-$数列.但$S_{n}=n$, $|S_{n+1}-S_{n}|+|S_{n}-S_{n-1}|+\cdots+|S_{2}-S_{1}|=n$.由 $n$ 的任意性知,数列$\left\{S_n\right\}$不是$B-$数列.

命题 2:若数列$\left\{S_n\right\}$是$B-$数列,则数列$\left\{x_n\right\}$是$B-$数列.

此命题为真命题.

事实上,因为数列$\left\{S_n\right\}$是 $B-$数列,所以存在正数$M$,
对任意的$n\in\mathbf{N}^\ast$,有$|S_{n+1}-S_{n}|+|S_{n}-S_{n-1}|+\cdots
+|S_{2}-S_{1}|\leqslant M$,
即$|x_{n+1}|+|x_{n}|+\cdots+|x_{2}|\leqslant M$.

于是
\begin{align*}
&|x_{n+1}-x_{n}|+|x_{n}-x_{n-1}|+\cdots+|x_{2}-x_{1}|\\ &\leqslant|x_{n+1}|+2|x_{n}|+2|x_{n-1}|+\cdots+2|x_{2}|+|x_{1}| \\
&\leqslant 2M+|x_1|,
\end{align*}
所以数列$\left\{x_n\right\}$是$B-$数列.

(注:按题中要求组成其他命题解答时,仿上述解法)

(III)若数列$\left\{a_n\right\},\left\{b_n\right\}$是 $B-$数列,则存在正数$M_1$, $M_2$,对任意的$n\in\mathbf{N}^{\ast}$,
有$\left|a_{n+1}-a_{n}\right|+\left|a_{n}-a_{n-1}\right |+\cdots +\left|a_{2}-a_{1}\right|\leqslant M_{1}$; $|b_{n+1}-b_n|+|b_n-b_{n-1}|+\cdots+|b_2-b_1|\leqslant M_2$.

注意到
\begin{align*}
|a_{n}|&=|a_{n}-a_{n-1}+a_{n-1}-a_{n-2}+\cdots+a_{2}-a_{1}+a_{1}|\\ &\leqslant|a_{n}-a_{n-1}|+|a_{n-1}-a_{n-2}|+\cdots+|a_{2}-a_{1}|
+|a_{1}|\\
&\leqslant M_{1}+|a_{1}|.
\end{align*}

同理, $|b_n|\leqslant M_2+|b_1|$.


记 $K_{1}=M_{1}+|a_{1}|,K_{2}=M_{2}+|b_{1}|$,则有
\begin{align*}
|a_{n+1}b_{n+1}-a_nb_n| &=|a_{n+1}b_{n+1}-a_nb_{n+1}+a_nb_{n+1}-a_nb_n|\\
&\leqslant |b_{n+1}| |a_{n+1}-a_n|+|a_n||b_{n+1}-b_n|\\
&\leqslant K_2\left|a_{n+1}-a_n\right|+K_1\left|b_{n+1}-b_n\right|.
\end{align*}
因此
\begin{align*}
&|a_ {n+1}b_{n+1}-a_{n}b_{n}|
+|a_{n}b_{n}-a_{n-1}b_{n-1}|+\cdots+|a_{2}b_{2}-a_{1}b_{1}|\\ &\leqslant K_{2}\left(\left|a_{n+1}-a_{n}\right|+\left|a_{n}-a_{n-1}\right|
+\cdots+\left|a_{2}-a_{1}\right|\right)+\\
&\quad K_1\left(\left|b_{n+1}-b_n\right|+\left|b_n-b_{n-1}\right|+\cdots
+\left|b_2-b_1\right|\right)\\
&\leqslant K_2M_1+K_1M_2.
\end{align*}
故数列$\{a_nb_n\}$是$B-$数列.

2024清华求真书院夏令营直博笔试试题

1. (10分)证明或举例否定$n$阶矩阵的存在性$(n\geqslant 2)$.

(1) $Ae^A=I$;

(2) $A$的特征值均为实数.


2. (10分)设$v$是$n\times 1$非零向量, $v^\ast$是其共轭转置,求$I-\sigma vv^\ast$的所有特征值及其特征向量.

3. (20分)证明或举例否定:

(1)对于$n$阶反对称矩阵$A$和对角线上都是正数的对角矩阵$D$有$|A+D|>0$.

(2)对于$n$阶正定矩阵$A$和半正定矩阵$B$有$|A+B|>|A|+|B|$.

 

4. (15分)设$f(x)$是$[a,b]$上的函数,对任意$x\in[a,b]$有$\lim_{t\to x}f(t)$存在,记该值为$g(x)$.证明:

(1) $g(x)$在$[a,b]$上连续;

(2) $f(x)$的间断点至多可数个.

 

5. (15分)证明或举例否定:函数$f(x)$在$[a,b]$上可导,则其

(1)导函数在$[a,b]$上连续;


(2)导函数在$[a,b]$上具有介值性.

 

6. (10分)证明或否定$\int_0^{+\infty}\left(\frac{\sin(3x)}{3x^2}
-\frac{\sin(2x)}{2x^2}\right)\mathrm{d}x$的收敛性;若收敛,求值.

 

7. (10分)将下面的级数化简到最简形式:
$$\sum_{n=-\infty}^{+\infty}\frac{n^2+n+1}{n^4+1}.$$


8. (10分)证明实函数列$\{f_n(x)\}$的$L^p$收敛可以推出存在几乎处处收敛的子列.

[详解](1)因为$\{a_n\}$的各项均为正数,所以$a_1>0,q>0$,
$|a_{k+1}-a_k|=|a_kq-a_k|=a_k|q-1|$,

当$q=1$时, $|a_{k+1}-a_{k}|=0$, $\sum_{k=1}^{n}|a_{k+1}-a_{k}|= 0$,任取$M>0$即可,所以$\{a_{n}\}$为有界变差数列.


当 $q\neq 1$时, $\sum_{k=1}^{n}|a_{k+1}-a_{k}|=(a_{1}+a_{2}+ \cdots+a_{n})\left|q-1\right|=\frac{a_{1}( 1- q^{n})}{1-q}\left |q-1\right|$,

若$0<q<1$,则$\frac{a_1(1-q^n)}{1-q}\left|q-1\right|
=a_1(1-q^n)<a_1$,令$M=a_1$即可,所以$\{a_n\}$为有界变差数列.

若 $q>1$,则$\frac{a_{1}( 1- q^{n})}{1-q}\left|q-1\right|=a_{1}( q^{n}-1)$,当 $n\to +\infty$时, $a_{1}( q^{n}-1)\to +\infty$,
显然不存在符合条件的$M$,故$\{a_n\}$不是有界变差数列.

综上, $q$的取值范围是$(0,1]$.

(2)由$b_{n+1}+\frac1{b_n}=2$,可得$b_{n+1}-1=\frac{b_n-1}{b_n}$,易知$b_n\neq 1$,所以$\frac1{b_{n+1}-1}=\frac{b_n}{b_n-1}=\frac1{b_n-1}+1$,
因此$\left\{\frac1{b_n-1}\right\}$是首项为$\frac1{b_1-1}=1$,公差为$1$的等差数列,所以$\frac1{b_n-1}=n$,即$b_n=\frac1n+1$.

所以$|b_{k+1}-b_{k}|=\left|\frac1{k+1}-\frac1k\right|=\frac1k-\frac1{k+1}$, $\sum_{k=1}^{n}|b_{k+1}-b_{k}|=\sum_{k=1}^{n}\left(\frac{1}{k}
-\frac{1}{k+1}\right)=1-\frac{1}{n+1}<1$,
所以$\{b_n\}$是有界变差数列.

(3)由有界变差数列的定义可知,
\begin{align*}
\sum_{k=1}^{n}|x_{k+1}-x_{k}| &=|x_{n+1}-x_{n}|+|x_{n}-x_{n-1}|+\cdots
+|x_{2}-x_{1}|<M_{1},\\
\sum_{k=1}^{n}|y_{k+1}-y_{k}| &=|y_{n+1}-y_{n}|+|y_{n}-y_{n-1}|+\cdots
+|y_{2}-y_{1}|<M_{2}.
\end{align*}


因为$|x_{n+1}|-|x_1|\leqslant |x_ {n+1}-x_1|=|x_{n+1}-x_n+x_n-x_{n-1}+\cdots+x_2-x_1|\leqslant
|x_{n+1}-x_n|+|x_n-x_{n-1}|+\cdots+|x_2-x_1|<M_1$,所以$|x_{n+1}|\leqslant M_1+|x_1|$.


\begin{align*}
\left| \frac{x_{n+1}}{y_{n+1}}-\frac{x_n}{y_n} \right|&=\frac{\left| x_{n+1}y_n-x_ny_{n+1} \right|}{|y_{n+1}||y_n|}=\frac{\left| x_{n+1}y_{n+1}-x_ny_{n+1}+x_{n+1}y_n-x_{n+1}y_{n+1} \right|}{\left| y_{n+1} \right|\left| y_n \right|}\\
&=\frac{\left| y_{n+1}\left( x_{n+1}-x_n \right) -x_{n+1}\left( y_{n+1}-y_n \right) \right|}{\left| y_{n+1} \right|\left| y_n \right|}\leq \frac{\left| x_{n+1}-x_n \right|}{\left| y_n \right|}+\frac{\left| x_{n+1} \right|\left| y_{n+1}-y_n \right|}{\left| y_{n+1} \right|\left| y_n \right|}\\
&\leqslant \frac{\left| x_{n+1}-x_n \right|}{y_1}+\frac{\left( M_1+\left| x_1 \right| \right) \left| y_{n+1}-y_n \right|}{y_{1}^{2}},
\end{align*}

因此$\sum_{k=1}^n{\left| \frac{x_{k+1}}{y_{k+1}}-\frac{x_k}{y_k} \right|}\leq \frac{\sum_{k=1}^n{\left| x_{k+1}-x_k \right|}}{y_1}+\frac{\left( M_1+\left| x_1 \right| \right) \sum_{k=1}^n{\left| y_{k+1}-y_k \right|}}{y_{1}^{2}}\leqslant \frac{M_1}{y_1}+\frac{\left( M_1+\left| x_1 \right| \right) M_2}{y_{1}^{2}}$,所以$\left\{\frac{x_{n}}{y_{n}}\right\}$是有界变差数列.

 

26道多项式问题


一、单位根

1. (1976年USAMO)多项式$A(x),B(x),C(x),D(x)$满足
$$A(x^5)+xB(x^5)+x^2C(x^5)=(1+x+x^2+x^3+x^4)D(x)$$
求证: $A(1)=0$.

 

2.称数列$a_1,a_2,\cdots,a_n$为$k-$平衡的,如果$a_1+a_{k+1}+\cdots=a_2+a_{k+2}+\cdots=\cdots=a_k+a_{2k}+\cdots$.若对于$k=3,5,7,11,13,17$,数列$a_1,a_2,\cdots,a_{50}$均为$k-$平衡
的.求证:所有的$a_i$均为零.

 

3.设$P(x)$为首一的整系数多项式,且其所有复根均在复平面的单位圆上.求证: $P(x)$的所有根均为单位根,即存在$n,k\in\mathbf{N}$,使得$P(x)\mid (x^n-1)^k$.


二、整除性质

一个重要结论:设$P(x)\in\mathbb{Z}[x]$,则对整数$a$, $b$,有$a-b\mid P(a)-P(b)$.


4. (1) (1974年USAMO)设$a,b,c$为三个互异的整数, $P$为整系数多项式.求证: $P(a)=b$, $P( b) = c$, $P( c) = a$不同时成立.

(2)设$P(x)$为整系数多项式, $n$为正奇数.整数列$x_1,x_2,\cdots,x_n$满足$x_2=P(x_1)$, $x_3= P( x_2)$, $\cdots$, $x_n= P( x_{n- 1})$, $x_1= P( x_n)$.求证:所有的$x_i$均为$1$.

 

(3) (2000年Putnam) $f(x)$为整系数多项式. 整数列$a_0,a_1,\cdots$满足$a_0=0$,对任意$n\geqslant 0$, $a_{n+ 1}= f( a_n)$.求证:若存在正整数$m$,使得$a_m=0$, 则$a_1=0$或$a_2=0$.

(4) (2006年IMO)设$n$, $k$是正整数且$n\geqslant 2$.设$P(x)$是$n$次整系数多项式,记$Q(x)=P^{(k)}(x)$.求证:最多存在$n$个整数$t$,使得$Q(t)=t$.

5.非零整数$a,b,c$满足$\frac ab+\frac bc+\frac ca$和$\frac ac+\frac cb+\frac ba$均为整数.求证: $|a|=|b|=|c|$.

6. (2005年IMO预选题)设$a,b,c,d,e,f$是正整数,记$S=a+b+c+d+e+f$.已知
$S$整除$abc+def$和$ab+bc+ca-de-ef-fd$.求证: $S$是合数.

 

三、零点定理

对任意连续函数(例如:多项式函数) $f$,若对$a<b$, $f(a)$和$f(b)$异号,则存在$t\in(a,b)$,使得$f(t)=0$.

 

7. (1995年CTST)甲、乙二人对一个至少4次的多项式。
$$x^{2n}+\square x^{2n-1}+\square x^{2n-2}+\cdots+\square x+1$$
玩填系数的游戏:二人轮流选定上式中的一个空格,在其中填写一个实数作为该项的系数,直到填完为止.若所得的多项式无实根,甲胜;否则乙胜.若甲先,谁有必胜策略?试说明理由.


8. (2002年USAMO)证明:任一$n$次实系数的首一多项式是两个$n$次的、有$n$个实根的首一多项式的平均.


四、拉格朗日插值公式


9.若$f(x)$为首一的$n-1$次多项式, $a_1,a_2,\cdots,a_n$为互异的实数.求证:
$$\sum_{i=1}^n\frac{f(a_i)}{\prod_{j\neq i}(a_j-a_i)}=1.$$


10. (1997年 IMO预 选 题 )设$f$为整系数多项式, $p$为素数,满足$f(0)=0,f(1)=1$,对任意正整数$k$, $f(k)\equiv 0$或$1\ (\bmod p)$.求证: $\deg f\geqslant p-1$.


五、切比雪夫多项式


11.设$P(x)$为$n$次实系数多项式,对任意$x\in[0,1]$, $|f(x)|\leqslant 1$.求证: $\left|f\left(-\frac1n\right)\right|\leqslant 2^{n+1}-1$.

 

12.设$P(x)$为首一的$n$次实系数多项式.求证:存在$t\in[-1,1]$,使得$|P(t)|\geqslant\frac1{2^n}$.


13. (Crux)实数$a_0,a_1,\cdots,a_n$满足对任意$x\in[-1,1]$,都有$|a_0+a_1x+\cdots+a_nx^n|\leqslant 1$.
求证:对任意$x\in[-1,1]$,都有$|a_n+a_{n-1}x+\cdots+a_0x^n|\leqslant 2^ {n-1}$.


14.设$n\geqslant 2$, $x_1,x_{2},\cdots,x_{n}$为$[-1,1]$中互异的$n$个实数.求证:
$$\frac{1}{t_1}+\frac{1}{t_2}+\cdots+\frac{1}{t_n}\geqslant 2^{n-1},$$
其中$t_i=\prod_{j\neq i}|x_j-x_i|$.


六、利用同余判断不可约性

 

15. (1) (Eisenstein判别法)设整系数多项式$f(x)=\sum_{i=0}^na_ix^i$.若存在素数$p$, 使得对任意
$0\leqslant i\leqslant n-1$,都有$p\mid a_i$,且$p\nmid a_n,p^2\nmid a_0$.则$f(x)$不可约.

(2)设整系数多项式$f(x)=\sum_{i=0}^na_ix^i$.若存在素数$p$,使得对任意$0\leqslant i\leqslant k-1$,都有$p\mid a_i$,且$p\nmid a_n,p^2\nmid a_0$.则$f(x)$有一个次数不小于$k$的不可约因式.

 

16.设$p$为素数,求证: $x^{p-1}+x^{p-2}+\cdots+1$不可约.


17.设$n$为正整数.求证: $(x^2+x)^{2^n}+1$不可约.


七、利用根判断不可约性

 

18.设整系数多项式$f(x)=\sum_{i=0}^na_ix^i$满足$|a_0|$为素数,且
$$|a_0|>|a_1|+|a_2|+\cdots+|a_n|.$$
求证: $f(x)$不可约.

 

19.设实系数多项式$f(x)=\sum_{i=0}^na_ix^i$的系数满足$0<a_0\leqslant a_1\leqslant\cdots\leqslant a_n$.求证: $f(x)$的每个复根模长均不超过$1$.


20.设$p$为素数.求证: $x^{p-1}+2x^{p-2}+3x^{p-3}+\cdots+(p-1)x+p$不可约.


21.设素数$p$在十进制下可以表示为$p=\overline{a_na_{n-1}\cdots a_1a_0}$,其中对任意$i$,都有$0\leqslant a_i<10$,且
$a_{n}\neq 0$.求证: $f(x)=\sum_{i=0}^{n}a_{i}x^{i}$不可约.

 

22. (2003年Romania TST)设首一的整系数多项式$f(x)$不可约,且$|f(0)|$不为完全平方数.求证: $f(x^2)$不可约.


八、Rouché定理的应用


Rouché定理:设解析函数$f(x)$及$g(x)$ (例如:多项式函数)定义在简单闭曲线$\mathcal{C}$ (例如:圆)内,且满足对任意$\mathcal{C}$中的复数$z$,都有$|f(z)|>|g(z)|$.则$f$和$f-g$在$\mathcal{C}$内部的根的个数(计重
根)相同.

 

23.设复系数多项式$P(z)=\sum_{i=0}^na_iz^i$满足:存在$0\leqslant k\leqslant n$,使得:
$$|a_k|>|a_0|+|a_1|+\cdots+|a_{k-1}|+|a_{k+1}|+\cdots+|a_n|.$$
求证: $P(z)$恰有$k$个根位于单位圆内部, $n-k$个根位于单位圆外部(均不含单位圆上).

24.多项式$P(x)=x^n+\sum_{i=0}^{n-1}a_ix^i$满足$a_0\neq 0$,
$$|a_{n-1}|>1+|a_{n-2}|+\cdots+|a_1|+|a_0|.$$
求证: $P(x)$不可约.


25. (1993年IMO)设$f(x)=x^n+5x^{n-1}+3$,其中$n$是大于1的整数.求证: $f(x)$不能表示成系数为整数的两个非常量的多项式的积.


26.设$f(x)$为首一的复系数多项式.求证:存在$z\in\mathbf{C}$,使得$|z|=1$且$|f(z)|\geqslant 1$.


第12章 复数

1 知识概要

定义 1 (复数的指数形式).设$\theta$为实数,把$\mathrm{e}^{\mathrm{i}\theta}$定义为$\cos\theta+\mathrm{i}\sin\theta$.

 

定理 1 (棣莫佛定理). $[r(\cos\theta+\mathrm{i}\sin\theta)]^n=r^n(\cos n\theta+\mathrm{i}\sin n\theta)\ (n\in\mathbf{N})$.

 

定理 2 (复数的开方)复数$z=r(\cos\theta+\mathrm{i}\sin\theta)\ (r\geqslant0,\theta\in\mathbf{R})$的$n$次方根是
$$\sqrt[n]{r}\left(\cos\frac{\theta+2k\pi}{n}
+\mathrm{i}\sin\frac{\theta+2k\pi}{n}\right)\ (k=0,1,\cdots,n-1).$$


定理 3 (模的三角不等式). $||z_1|-|z_2||\leqslant|z_1\pm z_2|\leqslant |z_1|+|z_2|$.


定理 4 (平行四边形四边关系). $\left|z_1+z_2\right|^2+\left|z_1-z_2\right|^2
=2\left|z_1\right|^2+2\left|z_2\right|^2$.

定理 5 (虚根成对).若虚数$z$为实系数一元$n$次方程$a_nx^n+a_{n-1}x^{n-1}+\cdots+a_1x+a_0=0$的根,则$\bar{z}$也是这个方程的根.

 


定义 2 (单位根).方程$x^n=1$的$n$个根$\varepsilon_k=\cos\frac{2k\pi}n+\mathrm{i}\sin\frac{2k\pi}n\ \left(k=0,1,\cdots,n-1\right)$称为单位根.

 

命题6. (1) $\varepsilon _j\varepsilon _k= \varepsilon _{j+ k}\ \left(j,k\in \mathbf{N}\right)$;

(2)设 $m$为整数,则$1+\varepsilon_1^m+\varepsilon_2^m+\cdots+\varepsilon_{n-1}^m=\begin{cases}
n,&\text{当}m\text{是}n\text{的倍数},\\
0,&\text{当}m\text{ 不是}n\text{ 的倍数}.
\end{cases}$

定理 7 (代数基本定理). $n$次复系数多项式方程有且仅有$n$个根 (重根按重数计).

 

例 1. 用复数推导三倍角公式.


例 2.设$n\in\mathbf{N}^\ast$,用复数推导$\sum_{k=0}^n\sin(x+kd),\sum_ {k=0}^n\cos(x+kd)$公式.

 

例 3. 求最小的正整数$n$,使得$I=\left(\frac12+\frac1{2\sqrt{3}}\mathrm{i}\right)^n$为纯虚数,并求出$I$.

 


例 4.已知$\theta=\arctan\frac{5}{12}$,则复数$z=\frac{\cos2\theta+\mathrm{i}\sin2\theta}{239+\mathrm{i}}$的辐角主值是\underline{\hspace{2cm}}.

 

例 5. (2019 上交自招)对于$m,n\in\mathbf{N}_+$,若$(\sqrt 3+\mathrm{i})^m=(1-\mathrm{i})^n$,求$n-m$的最小值.

 

例 6. 设$n\leqslant 1902,n\in\mathbf{N}$,且存在$\theta$满足$(\sin\theta+\mathrm{i}\cos\theta)^n=\sin n\theta+\mathrm{i}\cos n\theta$,那么这样的$n$的总个数是多少?

 

例 7.已知: $\frac{\cos x+\cos y+\cos z}{\cos(x+y+z)}=\frac{\sin x+\sin y+\sin z}{\sin(x+y+z)}=a$,则$\cos(y+z)+\cos(z+x)+\cos(x+y)=$\underline{\hspace{2cm}}.

 

例 8.已知复数$z_1,z_2,z_3$的辐角分别为$\alpha,\beta,\gamma$, $|z_1|=1,|z_2|=k,|z_3|=2-k$,且$z_1+z_2+z_3=0$.问: $k$为何值时, $\cos(\beta-\gamma)$分别取最大值和最小值?并求最大值和最小值.

 

例 9.模长均为$1$的复数$z_1,z_2,z_3$满足$z_1+z_2+z_3\neq 0$,则$\left|\frac{z_1z_2+z_2z_3+z_3z_1}{z_1+z_2+z_3}\right|$的值为\hfill (\quad)
\begin{tasks}(4)
\task $-\frac{1}{2}$
\task $1$
\task $2$
\task 无法确定
\end{tasks}


例10. (2023年北大寒假学堂) 设复数$a,b,c$满足$a+b+c=a^2+b^2+c^2=0,a^3+b^3+c^3=3$,则$a^{2023}+b^{2023}+c^{2023}$的值为\hfill (\quad)
\begin{tasks}(4)
\task $0$
\task $3$
\task $2023$
\task 前三个选项都不对
\end{tasks}

 

例 11.设复数$z_1,z_2$满足$|z_1|=|z_1+z_2|=3$, $|z_1-z_2|=3\sqrt{3}$,求$\log_3\left|(z_1\overline{z_2})^{2000}+(\overline{z_1}z_2)^{2000}\right|$的值.

 

例 12. (2022年北大强基)已知复致$z$,满足$\frac z2$与$\frac2z$的实部和虚部均属于$[-1,1]$,则$z$在复平面上形成轨迹的面积为\underline{\hspace{2cm}}.


例 13.设复平面上单位圆内接正二十边形的$20$个顶点所对应的复数依次为$z_1,z_2,\cdots,z_{20}$,则复数$z_1^{1995},z_2^{1995},\cdots,z_{20}^{1995}$所对应的不同的点的个数是\hfill (\quad)
\begin{tasks}(4)
\task $4$
\task $5$
\task $10$
\task $20$
\end{tasks}

 

例 14. (2021年清华强基)已知 $\varepsilon = \cos \frac \pi 5+\mathrm{i}\sin \frac \pi 5$,则\hfill (\quad)
\begin{tasks}(1)
\task $x^4+x^3+x^2+x+1=\left(x-\varepsilon\right)
\left(x-\varepsilon^3\right)\left(x-\varepsilon^7\right)
\left(x-\varepsilon^9\right)$
\task $x^4-x^3+x^2-x+1=\left(x-\varepsilon\right)
\left(x-\varepsilon^3\right)\left(x-\varepsilon^7\right)
\left(x-\varepsilon^9\right)$
\task $x^4-x^3-x^2+x+1=\left(x-\varepsilon\right)
\left(x-\varepsilon^3\right)\left(x-\varepsilon^7\right)
\left(x-\varepsilon^9\right)$
\task $x^4+x^3+x^2-x-1=\left(x-\varepsilon\right)\left(x-\varepsilon^3\right)
\left(x-\varepsilon^7\right)\left(x-\varepsilon^9\right)$
\end{tasks}

 

例 15. 设 $\varepsilon = \cos \frac {2\pi }n+\mathrm{i} \sin \frac {2\pi }n$,求证:

(1) $\left(1-\varepsilon\right)\left(1-\varepsilon^{2}\right)
\cdots\left(1-\varepsilon^{n-1}\right)=n$;

(2) (2017 清华金秋营) $\sin\frac\pi n\sin\frac{2\pi}n\cdots\sin\frac{(n-1)\pi}n=\frac n{2^{n-1}}$.

 

例 16. (2020年北大高水平艺术团)已知 $1,x_1,x_2,\cdots,x_{2018}$是 $x^2019-1=0$的$2019$个根,
求$\frac1{x_1+1}+\frac1{x_2+1}+\cdots+\frac1{x_{2018}+1}$的值.

 

例 17.设$\alpha,\beta$为复数,证明: $|\alpha-\beta| ^2=|\alpha|^2+|\beta|^2-2\mathrm{Re}(\overline{\alpha}\beta)$.


例 18.设复数$\alpha,\beta,\gamma$满足$|\alpha|=|\beta|=|\gamma|=1$,证明$\frac{(\alpha+\beta)(\beta+\gamma)(\gamma+\alpha)}{\alpha\beta\gamma}$是实数.


例 19.已知复变量满足$11z^{10}+10\mathrm{i}z^9+10\mathrm{i} z-11=0$,求证: $|z|=1$.


例 20.复数$z_1,z_2$滿足条件$|z_1-\overline{z}_2|^2=|1-\overline{z_1}\overline{z_2}|^2$,则$(|z_1|-1)(|z_2|-1)=$\underline{\hspace{2cm}}.


例 21. 给定实数$a,b,c$,已知复数$z_1,z_2,z_3$满足$|z_1|=|z_2|=|z_3|=1$, $\frac{z_1} {z_2}+\frac{z_2}{z_3}+\frac{z_3}{z_1}=1$,
求$|az_1+bz_2+cz_3|$的值.


例 22. 设复数$z,w$满足$|z|=3,(z+\overline{w})(\overline{z}-w)=7+4\mathrm{i}$,其中$\mathrm{i}$是虚数单位, $\overline{z},\overline{w}$分别表
示$z,w$的共轭复数,则$(z+2\overline{w})(\overline{z}-2w)$的模为\underline{\hspace{2cm}}.


例 23. 已知复数$z_1,z_2,z_3$满足$|z_1|=|z_2|=|z_3|=1,|z_1+z_2+z_3|=r$,其中$r$是给定实
数,则$\frac{z_1}{z_2}+\frac{z_2}{z_3}+\frac{z_3}{z_1}$的实部是\underline{\hspace{2cm}} (用含有$r$的式子表示).


例24.设$z_1,z_2\in\mathbf{C}$,且$z_1z_2\neq 0$, $A=z_1\overline{z_1}+z_2\overline{z_2}$, $B=z_1\overline{z_2}+\overline{z_1}z_2$,试问: $A$与$B$能否
比较大小关系?若能,请指明大小关系;若不能,请说明理由.

 


例 25. (1)在复数范围内因式分解$x^3+y^3+z^3-3xyz$;

(2)设复数$x,y,z$满足$|x|^2+|y|^2+|z|^2=1$.证明: $\left|x^3+y^3+z^3-3xyz\right|\leqslant 1$.


例 26.设$z$是模为$1$的复数,则函数$f(z)=z^2+\frac1{z^2}$的最小值为\hfill (\quad)
\begin{tasks}(4)
\task $0$
\task $-2$
\task $2$
\task 不存在
\end{tasks}


例 27. 设$\alpha,\beta$为一对共轭复数,如果$|\alpha-\beta|=\sqrt6$,且$\frac\alpha{\beta^2}$为实数,那么$|\alpha|=$\underline{\hspace{2cm}}.

例 28. 已知$|z_1|=2$, $|z_2|=3$, $|z_1+z_2|=4$,则 $\frac{z_1}{z_2}=$\underline{\hspace{2cm}}.

 

例 29.设复数$z_1,z_2$满足$|z_1|=|z_1+z_2|$, $\overline{z_1}z_2=a(1+\sqrt3 \mathrm{i})$, $a$是负实数,求$\frac{z_2}{z_1}$.


例 30. (2019年中科大自招)复数$z$满足$\frac{z-1}{z+1}$是纯虚数,则$\left|z^2+z+3\right|$的最小值为\underline{\hspace{2cm}}.


例 31. (2020年清华强基)设复数$z_1,z_2$在复平面内对应的点分别为$Z_1,Z_2$, $O$为坐标原点,若$|z_1|=1$, $5z_1^2+z_2^2-2z_1z_2=0$,则$\triangle OZ_1Z_2$的面积为\hfill (\quad)
\begin{tasks}(4)
\task $1$
\task $\sqrt{3}$
\task $2$
\task $2\sqrt {3}$
\end{tasks}

 

例 32.设复数$z_1,z_2$在复平面上对应的点分别为$A,B$,且$|z_1|=4$, $4z_1^2-2z_1z_2+z_2^2=0$, $O$为坐标原点,则$\triangle AOB$的面积为\underline{\hspace{2cm}}.

 

例 33. 复平面上, $A$点对应复数$-2$,动点$B$在以原点为圆心, $1$为半径的圆上运动.以$AB$为边作正$\triangle ABC$ ($A,B,C$按逆时针方向排列),求动点$C$的轨迹.

 

例 34. (2019年北大自招) 已知复数$z_1,z_2$滿足$|z_1-3\mathrm{i}|=2$, $|z_2-8|=1$,則由复数$w=z_1-z_2$对应的点所围成的围形的面积为\hfill (\quad)
\begin{tasks}(4)
\task $4\pi$
\task $8\pi$
\task $10\pi$
\task 前三个选项都不对
\end{tasks}

 

例35. (2020 清华强基)设复数$z$满足$|3z-7\mathrm{i}|=3$,则$\left|\frac{z^2-2z+2}{z-1+\mathrm{i}}\right|$的\hfill (\quad)
\begin{tasks}(4)
\task 最大值为$\frac 83$
\task 最大值为$\frac 73$
\task 最小值为$\frac 43$
\task 最小值为$\frac{2}{3}$
\end{tasks}

 

例 36. (2019年北大寒假学堂) 已知$z_1= 1+\mathrm{i}$, $z_2=-2+ 2\mathrm{i}$,在复平面内$z_1,z_2,z_3$构成正
三角形,则$z_3=$\underline{\hspace{2cm}}.

 

例 37. (2023年北大寒假学堂) 设三角形$ABC$的三个顶点为复平面上的三点$z_1,z_2,z_3$,满足$z_1z_2z_3= 0$, $z_1+ z_2+ z_3= 8+ 2\mathrm{i}$, $z_1z_2+ z_2z_3+ z_3z_1=15+ 10\mathrm{i}$,则三角形$ABC$内心的复数坐标$z$的虚部所在区间为\hfill (\quad)
\begin{tasks}(4)
\task $( 0, 0. 5)$
\task $( 0. 5, 1)$
\task $(0,1, 2)$
\task 前三个选项都不对
\end{tasks}

 

例 38. 用复数证明托勒密不等式:在平面四边形$ABCD$中, $AB\cdot CD+AD\cdot BC\geqslant AC\cdot BD$,当且仅当四边形$ABCD$为圆内接四边形时等号成立.

 

例 39.设$A_0A_1\cdots A_{n-1}$是正$n$边形,它的外接圆$O$的半径为$1$,求证:自圆$O$上任意一点$P$到各顶点的距离的平方和为定值.


例 40. 复平面上三个点$A,B,C$所对应的复数分别为$z_1,z_2,z_3$.若$z_1,z_2,z_3$是方程$z^3-3pz^2+3qz-r=0$的三个根$(r\neq0)$.求证: $\triangle ABC$是正三角形的充分必要条件是$p^2=q$.

 

例 41. 关于$x$的方程$x^2+mx+1+2\mathrm{i} =0$有实数根.则复数$m$的模的最小值为\underline{\hspace{2cm}}.

 

例 42. 证明或否定:在凸四边形$ABCD$所在的平面上存在一点$E$,使得$\triangle ABE\sim \triangle CDE$.


例 43.对于$n\in\mathbf{N}^\ast$,若存在复数$z$满足$|z|=1$, $z^{n+1}=z^n+1$,则$n$的最小可能值是\underline{\hspace{2cm}}.


例 44. (2019年北大寒假学堂)已知复数$z$满足$|z|=1$,且有$z^17+z=1$,求$z=$\hfill (\quad)
\begin{tasks}(4)
\task $\frac 12\pm \frac{\sqrt {3}}2\mathrm{i}$
\task $\frac {\sqrt {3}}2\pm \frac 12\mathrm{i}$
\task $\frac {\sqrt {2}}2\pm \frac {\sqrt {2}}2\mathrm{i}$
\task 前三个选项都不对
\end{tasks}

 


例 45.设$z,\omega,\lambda$为复数, $|\lambda|\neq 1$,解关于$z$的方程: $\overline{z}-\lambda z=\omega$.

 


例 46. (2019年上交自招)已知复数$z$满足$|z|=1$,若存在负实数$a$使得$z^2-2az+a^{2}-a=0$,则$a=$\underline{\hspace{2cm}}.


例 47.在关于$x$的二次方程$x^2+z_1x+z_2+m=0$中, $z_1,z_2,m$均是复数,且满足$z_1^2-4z_2=16+20\mathrm{i}$.设这个方程的两个根为$\alpha,\beta$,且两根满足$|\alpha-\beta|=2\sqrt7$,求$|m|$的最大值和最小值.

 


例 48. (多选题) (2020年清华强基压轴)已知$f(z)=z^{10}+\frac1{z^{10}}+\frac12\left(z^5+\frac1{z^5}\right)$,则\hfill (\quad)
\begin{tasks}(1)
\task $f( z) = 0$存在实数解
\task $f(z)=0$共有$20$个不同的复数解
\task $f(z)=0$的复数解的模长均为$1$
\task $f(z)=0$存在模长大于$1$的复数解
\end{tasks}

 


例 49.已知复数$z_1,z_2$满足: $|z_1|=2,|z_2|=3,3z_1-2z_2=2-\mathrm{i}$.求$z_1z_2$的值.

 


例 50. 证明:实系数多项式$f(x)=x^3+a_1x^2+a_2x+a_3$的$3$个根均在左半复平面内 (即实部小于$0$)的充要条件是$a_i>0\ (i=1,2,3)$且$a_3<a_1a_2$.

 

例 51. $\alpha,\beta,\gamma$为方程$x^3-4x+2=0$的三根,求以$\frac16\left(\alpha^6+\beta^6+\gamma^6\right)$为常数项,以$\alpha\beta\gamma$为一次项系数而二次项系数为$1$的一元二次方程的根.


例 52. 设 $a_1,a_2,\cdots,a_n$是互不相等的实数,证明:方程$\frac1{x+a_1}+\frac1{x+a_2}+\cdots+\frac1{x+a_n}=0$
没有虚数根.


例 53. 已知$z\in\mathbf{C}$, $\left|z+\frac1z\right|=1$.则 $|z|_{\max}+|z|_{\min}=$\underline{\hspace{2cm}}.


例 54. 已知关于$x$的方程$x^2-2x+2=0$和$x^2+2mx+1=0$的四个不同的根在复平面上对应的点共圆.则实数$m$的取值范围是\underline{\hspace{2cm}}.


例 55. 若$x_1,x_2,x_3$是关于$x$的一元三次方程$x^3-5x^2+5x+1=0$的三个两两不等的复数根,则代数式 $\left(x_1^2+x_1x_2+x_2^2\right)\left(x_2^2+x_2x_3+x_3^2\right)
\left(x_3^2+x_3x_1+x_1^2\right)$的值为\underline{\hspace{2cm}}.


例 56. 设$a,b,c$为复数,并且方程$x^3+ax^2+bx+c=0$的三个复根$z$都满足$|z|=1$.证明:方程$x^3+|a|x^2+|b|x+|c|=0$的三个复根$w$也满足$|w|=1$.


例 57. 设复数$z$满足$|z|=1$,使得关于$x$的方程$zx^2+2\overline{z}x+2=0$有实根,则这样的复
数$z$的和为\underline{\hspace{2cm}}.

 

例 58. 实系数一元二次方程$ax^2+bx+c=0$有两个虚数根$x_1,x_2$,且$x_1^5\in\mathbf{R}$,求$\frac{b^2}{ac}+\frac{ac}{b^2}$.

 

例 59. 方程$ax^2+b|x|+c=0\ (a,b,c\in\mathbf{R},a\neq 0)$在复数集内的根的个数是$n$,则\hfill (\quad)
\begin{tasks}(1)
\task $n$最大是$2$
\task $n$最大是$4$
\task $n$最大是$6$
\task $n$最大是$8$
\end{tasks}

 

例 60. 方程$x^{10}+(13x-1)^{10}=0$的$10$个复数根分别为$r_1,\overline{r_1},r_2,\overline{r_2},r_3,\overline{r_3},
r_4,\overline{r_4},r_5,\overline{r_5}$.
求代数式$\frac1{r_1\overline{r_1}}+\frac1{r_2\overline{r_2}}
+\cdots+\frac1{r_5\overline{r_5}}$ 的值.

15道数学竞赛中的方程(组)问题


1. (2006年加拿大数学奥林匹克)解方程组:
$$v_i=1+\frac{6v_i^2}{v_1^2+v_2^2+\cdots+v_{10}^2}\ (1\leqslant i\leqslant 10).$$


2. (2016年ELMO)求所有的正整数$n$,使得存在$2n$个不全相同的实数$x_1,x_2,\cdots,x_{2n}$,满足其中任意$n$个之和等于另$n$个之积.


3. (2021年美国数学奥林匹克)设整数$n\geqslant 4$,求如下方程组的正实数解:
\begin{align*}
a_{1} &=\frac1{a_{2n}}+\frac1{a_2},&a_2 &=a_1+a_3,\\
a_{3} &=\frac1{a_2}+\frac1{a_4}, &a_4 &=a_3+a_5,\\
a_{5} &=\frac1{a_4}+\frac1{a_6}, &a_6 &=a_5+a_7\\
&\vdots &&\vdots\\
a_{2n-1} &=\frac1{a_{2n-2}}+\frac1{a_{2n}},& a_{2n} &=a_{2n-1}+a_1.
\end{align*}

 

4. (2006年美国数学奥林匹克)求所有的实数$x,y,z\geqslant 1$,满足
$$\min\left\{\sqrt{x+xyz},\sqrt{y+xyz},\sqrt{z+xyz}\right\}
=\sqrt{x-1}+\sqrt{y-1}+\sqrt{z-1}.$$


5. (2019年欧洲女子数学奥林匹克) 求满足下述条件的所有三元有序实数组$(a,b,c)$:
$$a^2b+c=b^2c+a=c^2a+b,\quad ab+bc+ca=1.$$


6. (2009年亚太地区数学奥林匹克)设实数$a_1,a_2,a_3,a_4,a_5$满足,对$k=1,2,3,4,5$,
$$\frac{a_{1}}{k^{2}+1}+\frac{a_{2}}{k^{2}+2}+\frac{a_{3}}{k^{2}+3}
+\frac{a_{4}}{k^{2}+4}+\frac{a_{5}}{k^{2}+5}=\frac{1}{k^{2}}.$$
求$\frac{a_{1}}{37}+\frac{a_{2}}{38}+\frac{a_{3}}{39}+\frac{a_{4}}{40}
+\frac{a_{5}}{41}$的值.

 

7. (2006年中国西部数学奥林匹克)求满足下述条件的最小正实数$k$:对任意不小于$k$的$4$个互不相同的实数$a,b,c,d$,都存在$a,b,c,d$的一个排列$p,q,r,s$,使得方程$(x^2+px+q)(x^2+rx+s)=0$有$4$个互不相同的实根.

 


8. (2005年中国女子数学奥林匹克)解方程组:
$$\begin{cases}
5\left(x+\frac{1}{x}\right)=12\left(y+\frac{1}{y}\right)
=13\left(z+\dfrac{1}{z}\right),\\
xy+yz+zx=1.
\end{cases}$$


9. (2006年全国高中数学联赛)解方程组:
$$\begin{cases}
x-y+z=w=2,\\
x^2-y^2+z^2-w^2=6,\\
x^3-y^3+z^3-w^3=20,\\
x^4-y^4+z^4-w^4=66.
\end{cases}$$


10. (根源杯模拟题15)实数范围内求解方程组:
$$\begin{cases}
1-x_1^2=x_2,\\
1-x_2^2=x_3,\\
\cdots\\
1-x_{2n-1}^2=x_{2n},\\
1-x_{2n}^2=x_1.
\end{cases}$$


11. 正整数$n\geqslant 2$,互不相等的非零实数$a_1,a_2,\cdots,a_n$满足:
$$a_1+\frac1{a_2}=a_2+\frac1{a_3}=\cdots=a_n+\frac1{a_1}=t,$$
求实数$t$的值.

 

12. (2020年越南数学奥林匹克)以$a$为参数的方程组:
$$\begin{cases}
x-ay=yz,\\
y-az=zx,\\
z-ax=xy,\end{cases}(x,y,z\in\mathbf{R}).$$

(1)当$a=0$时,解这个方程组;

(2)证明:当$a>1$时该方程组有$5$组解.

 


13. (2021年百子S9)求所有的正整数$n$,使得方程组
$$\begin{cases}x+y+z=3,\\
x^2+y^2+z^2=3,\\
x^n+y^n+z^n=3,\end{cases}
(x,y,z\in\mathbf{C})$$
只有唯一解$x=y=z=1$.


14. (1995年土耳其国家队选拔)给定实数$b\geqslant a>0$,在复数范围内解方程组:
$$\begin{cases}
x_1^2+2ax_1+b^2=x_2,\\
x_2^2+2ax_2+b^2=x_3,\\
\cdots\cdots\\
x_n^2+2ax_n+b^2=x_1.
\end{cases}$$


15. (1979年国际数学奥林匹克)求所有满足条件的实数$a$,使得存在非负实数$x_1,x_2,x_3,x_4,x_5$满足关系: $\sum_ {k=1}^5kx_k=a$, $\sum_{k=1}^5k^3x_k=a^2$, $\sum_{k=1}^5k^5x_k=a^3$.

 


32道含绝对值的不等式

1.两个实数列$a_1,\cdots,a_n;b_1,\cdots,b_n$满足$\sum a_i^2=\sum b_i^2=1$,证明
$$(a_1b_2-a_2b_1)^2\leqslant 2|a_1b_1+a_2b_2+\cdots+a_nb_n-1|.$$


2.设$a_1,a_2,\cdots,a_n$是任意实数,证明:
$$\sum_{i,j=1}^n|a_i+a_j|\geqslant n\sum_{i=1}^n|a_i|.$$


3.设$a,b,c$是三个不同的正实数,证明:
$$\frac1{|a^2-b^2|}+\frac1{|b^2-c^2|}+\frac1{|c^2-a^2|}
+\frac8{a^2+b^2+c^2}\geqslant \frac{28}{(a+b+c)^2}.$$


4. (2009年CMO)给定整数$n\geqslant 3$,实数$a_1,a_2,\cdots,a_n$满足$\min_{1\leqslant i<j\leqslant n}|a_i-a_j|=1$.求
$\sum_{k=1}^n|a_k|^3$的最小值.

 

5. (2013年CMO)求所有的正实数$t$,满足:存在一个由实数组成的无限集合$X$,使得对任
意的$x,y,z\in X$ ($x,y,z$可以相同),及任意实数$a$与正实数$d$,均有
$$\max\{|x-(a-d)|,|y-a|,|z-(a+d)|\}>td.$$


6. (2019年CMO)设实数$a_1,a_2,\cdots,a_{40}$满足$a_1+a_2+\cdots+a_{40}=0$,且对$1\leqslant i\leqslant 40$,均有
$|a_i-a_{i+1}|\leqslant 1$,这里 $a_{41}=a_1$.记 $a=a_{10},b=a_{20},c=a_{30},d=a_{40}$.

(1)求$a+b+c+d$的最大值;

(2)求$ab+cd$的最大值.

 


7. (1995年CTST)给实数$P\geqslant 2$,求$\lambda$的最小值使得对于任何$a,b\in (-1,1)$都有
$$|a-b|^p\leqslant\lambda (2-|a-b|)\left|\frac{a|a|^{p-1}}{1-|a|}
-\frac{b|b|^{p-1}}{1-|b|}\right|.$$


8. (2009年CTST)已知$x_1,\cdots,x_m,y_1,\cdots,y_n$是正实数,
记$X=\sum_{i=1}^{n}x_i$, $\sum_{i= 1}^{n}y_i$.求证:
$$2XY\sum_{i=1}^n\sum_{j=1}^n|x_i-y_j|\geqslant X^2\sum_{i=1}^n\sum_{j=1}^n|y_i-y_j|
+Y^2\sum_{i=1}^n\sum_{j=1}^n|x_i-x_j|.$$

 

9. (2010年高联) 给定整数$n>2$,设正实数$a_1,a_2\cdots,a_n$满足$a_k\leqslant 1,k=1,2,\cdots,n$,记
$$A_k=\frac{a_1+a_2+\cdots+a_k}k,k=1,2,\cdots,n,$$
求证:
$$\left|\sum_{k=1}^na_k-\sum_{k=1}^nA_k\right|<\frac{n-1}{2}.$$

 


10. (2017年高联B卷)设实数$a,b,c$满足$a+b+c=0$.令 $d=\max\{|a|,|b|,|c|\}$.证明:
$$|(1+a)(1+b)(1+c)|\geqslant 1-d^2.$$


11.对所有的实数$x,y,z$,证明: $|x|+|y|+|z|-|x+y|-|y+z|-|z+x|+|x+y+z|\geqslant 0.$


12.对于三个互不相同的实数$a_1,a_2,a_3$,按如下方式定义三个实数: $b_1,b_2,b_3$:
$$b_j=\left(1+\frac{a_ja_i}{a_j-a_i}\right)
\left(1+\frac{a_ja_k}{a_j-a_k}\right)\ (i,j,k=1,2,3)$$
证明:
$$1+|a_1b_1+a_2b_2+a_3b_3|\leqslant (1+|a_1|)(1+|a_2|)(1+|a_3|),$$
并指出等号成立的条件.


13.设$x_1,x_2,\cdots,x_n$为实数.证明:
$$\sum_{i,j=1}^n|x_i+x_j|\geqslant n\sum_{i=1}^n|x_i|.$$

 

14.已知$n\ (n\geqslant 3)$为正整数.证明:对于任意不全相同的$n$个实数对
$(x_1,y_1),(x_2,y_2),\cdots,(x_n,y_n)$,有
$$\left|\sum_{i=1}^n(x_iy_{i+1}-x_{i+1}y_i)\right|\leqslant
\sum_{i=1}^n\left[\left(x_i-\sum_{j=1}^n\frac{x_j}n\right)^2
+\left(y_i-\sum_{j=1}^n\frac{y_j}n\right)^2\right].$$
求所有满足等号成立的正整数$n$并举例.

 

15. (1987年IMO)令$x_1,x_2,\cdots,x_n$是实数,满足条件 $x_1^2+x_2^2+\cdots+x_n^2=1$,求证:对于每
一个整数$k\geqslant 2$,存在不全为$0$的整数$a_1,a_2,\cdots,a_n$,对于所有的整数$i$使得$|a_i|\leqslant k-1$,并且有
$$|a_1x_1+a_2x_2+\cdots+a_nx_n|\leqslant\frac{(k-1)\sqrt{n}}{k^n-1}.$$

 

 

16. (1997年IMO)设$x_1,x_2,\cdots,x_n$是满足下列条件的实数 $|x_1+x_2+\cdots+x_n|=1$且$|x_i|\leqslant (n+1)/2,i=1,2,\cdots,n$.证明:存在$x_1,x_2,\cdots,x_n$的一个排列$y_1,y_2,\cdots,y_n$,使得$|y_1+2y_2+\cdots+ny_n|\leqslant (n+1)/2$.

 

17. (2003年IMO)设$n$为正整数,实数$x_1,x_2,\cdots,x_n$满足$x_1\leqslant x_2\leqslant \cdots\leqslant x_n$.

(1)证明:
$$\left(\sum_{i=1}^n\sum_{j=1}^n|x_i-x_j|\right)^2\leqslant
\frac{2(n^2-1)}{3}\sum_{i=1}^n\sum_{j=1}^n(x_i-x_j)^2.$$


(2)证明等号成立的充要条件是$x_1,\cdots,x_n$成等差数列.


18. (2006年IMO)求最小的实数$M$,使得对所有的实数$a,b,c$,有
$$|ab(a^2-b^2)+bc(b^2-c^2)+ac(c^2-a^2)|\leqslant M(a^2+b^2+c^2)^2.$$


19.设$x_1,x_2,\cdots,x_n$均属于区间$[0,1]$,证明:
$$\sum_{i=1}^n\sum_{j=1}^n|x_i-x_j|\leqslant\frac{n^2}2.$$


20.设实数$x_1,x_2,\cdots,x_{100}$之和为$1$且
$$|x_{k+1}-x_k|<\frac{1}{50},\quad k=1,2,\cdots,99.$$
求证:可以从这$100$个数中选出$50$个数,使得它们的和与$\frac{1}{2}$之差的绝对值不大于$\frac{1}{100}$.

 

21.设$0\leqslant p_i\leqslant 1,i=1,2,\cdots,n$,求证:存在$x\in [0,1]$满足
$$\sum_{i=1}^n\frac{1}{|x-p_i|}\leqslant 8n\left(1+\frac{1}{3}+\frac{1}{5}+\cdots+\frac{1}{2n-1}\right).$$


22.设实数$x,y$满足$|x|<1,|y|<1$.求证:
$$\left|\frac{x-y}{1-xy}\right|<1.$$


23. $x, y, z$为实数,证明:
$$|x|+|y|+|z|\leqslant |x+y-z|+|x-y+z|+|-x+y+z|.$$


24.已知$x_1,x_{2},\cdots,x_{n}$都是实数, $n\geqslant 2$,且$\sum_{k=1}^{n}|x_{k}|=1$, $\sum_{i=1}^{n}x_{i}=0$.求证:
$$\left|\sum_{k=1}^n\frac{x_k}k\right|\leqslant \frac12-\frac1{2n}.$$


25.求证:对任意实数$a_1,a_2,\cdots,a_n$,存在自然数$k$, $1\leqslant k\leqslant n$,使得对任意
$1\geqslant b_1\geqslant b_2\geqslant\cdots\geqslant b_n\geqslant 0$,都有$\left|\sum_{i=1}^nb_ia_i\right|\leqslant\left|\sum_ {i=1}^ka_i\right|$.

 

26.如果正数$M$与数组$a_{11},a_{12},\cdots,a_{1n};
a_{21},a_{22},\cdots,a_{2n};\cdots;a_{n1},a_{n2},\cdots,a_{nn}$使得对任意$x_1,x_2,\cdots,x_n\in\{-1,1\}$,有$\sum_{k=1}^n|a_{k1}x_1+a_{k2}x_2+\cdots+a_{kn}x_n|\leqslant M$,求证:
$$|a_{11}|+|a_{22}|+\cdots+|a_{nn}|\leqslant M.$$


27.求证:对任意 $a_1,a_2,\cdots,a_n\in [0,2]$, $n\geqslant 2$,有$\sum_{i,j=1}^n|a_i-a_j|\leqslant n^2$.并确定对于什么样的
$a_1,a_2,\cdots,a_n$上式中的等号成立.

 


28.给定整数$n\geqslant 3$.证明:对任意实数$a_i\geqslant 1\ (i=1,2,\cdots,n)$,都有
$$(a_1+a_2+\cdots+a_n)\left(\frac{1}{a_1}+\frac{1}{a_2}+\cdots
+\frac{1}{a_n}\right)\leqslant n^2+\sum_{1\leqslant i<j\leqslant n}|a_i-a_j|.$$


29.设$n$为正整数,实数$x_1,x_2,\cdots,x_n$满足$x_1^2+x_2^2+\cdots+x_n^2\leqslant 1$.证明:
$$|x_1|+|x_2|+\cdots+|x_n|\leqslant
\sqrt{n}\left(1+\frac{1}{n}\right)+n^{\frac{n-1}{2}}x_1x_2\cdots x_n,$$
并指出等号成立的充要条件.


30. 给定正整数$n$和正实数$C$.已知实数$x_1,x_2,\cdots,x_{2n}$满足$x_1+x_2+\cdots+x_{2n}=C$,
$|x_{k+1}-x_k|<\frac{C}{n}\ \left(k=1,2,\cdots,2n,x_{2n+1}=x_1\right)$.
证明:存在$n$个整数
$1\leqslant i_1<i_2<\cdots<i_n\leqslant 2n$,使得
$$\left|x_{i_1}+x_{i_2}+\cdots+x_{i_n}-\frac{C}{2}\right|
<\frac{C}{2n}.$$


31.设$x_1,x_2,\cdots,x_n$为实数.证明:
$$\left(\sum_{1\leqslant i<j\leqslant n}|x_i-x_j|\right)^2\geqslant (n-1)\sum_{1\leqslant i<j\leqslant n}|x_i-x_j|^2.$$


32.设$n\ (n\geqslant 3)$为正整数.证明:对于任意不全相同的$n$个实数对
$(x_1,y_1),(x_2,y_2),\cdots,(x_n,y_n)$,有
$$\left|\sum_{i=1}^n(x_iy_{i+1}-x_{i+1}y_i)\right|\leqslant
\sum_{i=1}^n\left(\left(x_i-\sum_{j=1}^n\frac{x_j}n\right)^2
+\left(y_i-\sum_{j=1}^n\frac{y_j}n\right)^2\right)$$
约定: $x_{n+1}=x_1,y_{n+1}=y_1$;并求所有满足等号成立的正整数$n$.


(INMO 2024) For each positive integer $n \ge 3$, define $A_n$ and $B_n$ as
\[A_n = \sqrt{n^2 + 1} + \sqrt{n^2 + 3} + \cdots + \sqrt{n^2+2n-1}\]\[B_n = \sqrt{n^2 + 2} + \sqrt{n^2 + 4} + \cdots + \sqrt{n^2 + 2n}.\]Determine all positive integers $n\ge 3$ for which $\lfloor A_n \rfloor = \lfloor B_n \rfloor$.
Note. For any real number $x$, $\lfloor x\rfloor$ denotes the largest integer $N\le x$.

 

$ (n+\frac{k-1}{n})^2<n^2+2k-1<(n+\frac{2k-1}{2n})^2$ for $1 \leq k \leq n$
So $ n^2+\frac{n-1}{2}<A_n< n^2+\frac{n}{2}$

$(n+\frac{2k-1}{2n})^2<n^2+2k<(n+\frac{k}{n})^2$ for $1 \leq k \leq n$
So $ n^2+\frac{n}{2}<B_n<n^2+\frac{n+1}{2}$

If $n$ is even, then $ [A_n]=n^2+\frac{n}{2}-1$ and $[B_n]=n^2+\frac{n}{2}$
If $n$ is odd then $[A_n]=[B_n]=n^2+\frac{n-1}{2}$

Claim: $ \frac{1}{2(x+1)} <\sqrt{x^2+n+1} - \sqrt{x^2 +n}< \frac{1}{2x}$ for $x \geq 3$ and $n \leq 2x$
Proof: $(\sqrt{x^2+n+1} + \sqrt{x^2 +n} )(\sqrt{x^2+n+1} - \sqrt{x^2 +n} ) = 1$
$\implies \frac{1}{2(x+1)} < \frac{1}{\sqrt{x^2+n+1} + \sqrt{x^2 +n}} < \frac{1}{2x}$
Now $ \frac{1}{2(x+1)} < \sqrt{x^2+n+1} - \sqrt{x^2 +n} < \frac{1}{2x} \implies \frac{1}{2(n+1)} < \sqrt{n^2+2k+2} - \sqrt{n^2 +2k+1} < \frac{1}{2n} $
$\implies \frac{n}{2(n+1)} < B_n - A_n <\frac{1}{2} $
Now for $k < 2n+1$ we have $ 2n+ 1 < \sqrt{n^2+k} + \sqrt{n^2+2n+1-k} < 2\sqrt{n^2+n+\frac{1}{2}} $
$\implies 2n\sqrt{n^2+n+\frac{1}{2}} > B_n +A_n > n(2n+1)$
Simplifying these two inequalities we get
$ n \sqrt{n^2+n+\frac{1}{2}} + \frac{1}{4} > B_n > n(n+\frac{1}{2}) +\frac{n}{4(n+1)}$ ,and
$ n \sqrt{n^2+n+\frac{1}{2}} + \frac{1}{4} - \frac{n}{2(n+1)} > A_n > n(n+\frac{1}{2}) +\frac{n}{4(n+1)} - \frac{1}{2}$
Now, $\sqrt{n^2+n+\frac{1}{2}} < n + \frac{1}{2}+ \frac{1}{8n}$
Putting this in our equations
$n(n+\frac{1}{2})+\frac{3}{8} > B_n > n(n+\frac{1}{2}) +\frac{n}{4(n+1)}$
$n(n+\frac{1}{2}) + \frac{1}{2(n+1)} - \frac{1}{8} > A_n >n(n+\frac{1}{2}) +\frac{n}{4(n+1)} - \frac{1}{2} $
Now for even $n$ , $[B_n]=n^2+\frac{n}{2}$ and $ [A_n]=n^2+\frac{n}{2}-1$
and for odd $n$, $[A_n]=[B_n]=n^2+\frac{n-1}{2}$


We claim that all odd $n$ work and no even $n$ work. It is clear that $B_n>A_n$ for all $n$.

Claim:
\[\frac{n^2}{2n+1}<A_n-n^2<\frac n2, \ \ \ (\spadesuit)\]\[\frac{n(n+1)}{2n+1}<B_n-n^2<\frac {n+1}2. \ \ \ (\clubsuit)\]
Proof. Note that
\[A_n-n^2=\sum_{i=1}^{n}\sqrt{n^2+2i-1}-n=\sum_{i=1}^{n}\frac{2i-1}{n+\sqrt{n^2+2i-1}}\]Also, $\frac{2i-1}{2n+1}<\frac{2i-1}{n+\sqrt{n^2+2i-1}}<\frac{2i-1}{2n}$, which gives
\[\frac{n^2}{2n+1}=\frac1{2n+1}\sum_{i=1}^{n}(2i-1)<A_n-n^2=\sum_{i=1}^{n}\frac{2i-1}{n+\sqrt{n^2+2i-1}}<\frac1{2n}\sum_{i=1}^{n}(2i-1)=\frac{n^2}{2n}=\frac n2,\]which proves $(\spadesuit)$.

Similarly, we have
\[\frac{n(n+1)}{2n+1}=\frac1{2n+1}\sum_{i=1}^{n}2i<B_n-n^2=\sum_{i=1}^{n}\frac{2i}{n+\sqrt{n^2+2i}}<\frac1{2n}\sum_{i=1}^{n}2i=\frac{n(n+1)}{2n}=\frac{n+1}2,\]which proves $(\clubsuit)$.
Hence, the claim is true. $\blacksquare$

Note that since $n>0$, $\frac{n^2}{2n+1}>\frac{n-1}2$ and using $(\spadesuit)$ gives $\frac{n-1}2<A_n-n^2$. Similarly, we have $\frac{n(n+1)}{2n+1}>\frac n2$ and using $(\clubsuit)$ gives $\frac n2<B_n-n^2$.

Combining, we have
\[\frac n2-1<\frac{n-1}2<\frac{n^2}{2n+1}<A_n-n^2<\frac n2<\frac{n(n+1)}{2n+1}<B_n-n^2<\frac{n+1}2<\frac n2+1. \ \ \ (\star)\]Hence, if $n$ is even, then $\frac n2$ is an integer, and $(\star)$ gives
\[\lfloor A_n\rfloor=n^2+\frac n2-1\neq \lfloor B_n\rfloor=n^2+\frac n2,\]while if $n$ is odd, then
\[\lfloor A_n\rfloor=n^2+\frac{n-1}2=\lfloor B_n\rfloor,\]as required.

The proof is complete. $\blacksquare$

第四题 给定整数$n\geq2.$设实数$a_1,a_2,\cdots,a_n\in[0,1)$,若对任意不超过$k$的正整数$i$,在$a_1,a_2,\cdots,a_n$中都存在若干项之和等于$i$,求正整数$k$的最大可能值.


已知数列$A:a_1,a_2,\cdots,a_n$满足: $a_i\in [0,1)$ $(i=1,2,\cdots,n,n\geqslant 2)$,从$A$中选取第$i_1$项、第$i_2$项、$\cdots$、第$i_k$项 $(i_1<i_2<\cdots<i_k,k\geqslant 2)$,称所得的数列$A_k:a_{i_1},a_{i_2},\cdots,a_{i_k}$为$A$的长度为$k$的子列,并记$S(A_k)=a_{i_1}+a_{i_2}+\cdots+a_{i_k}$.

(1)设数列$A:0,\frac{1}{2},0,\frac{1}{2},0$且$S(A_2)=1$,求$i_1+i_2$;


(2)设数列$A:\frac{1}{2},\frac{1}{3},\cdots,\frac{1}{(n+1)^2},\frac{1}{n+1}$,证明:存在$A_{n+1}$使得$S(A_{n+1})=1$;


(3)设$n=2024$, $m\in \mathbf{N}^\ast$,若对任意$N\in \{1,2,\cdots,m\}$,均存在$A_k$使得$S(A_k)=N$,求$m$的最大可能值.

(1)由题意可知$i_1=2,i_2=4$,则$i_1+i_2=6$.

(2)取$A$的长度为$n+1$的子列$A_{n+1}:\frac{1}{1\times 2},\frac{1}{2\times 3},\cdots,\frac{1}{n(n+1)},\frac{1}{n+1}$,则有
\begin{align*}
S\left( A_{n+1} \right) &=\frac{1}{1\times 2}+\frac{1}{2\times 3}+\cdots +\frac{1}{n\left( n+1 \right)}+\frac{1}{n+1}\\
&=1-\frac{1}{2}+\frac{1}{2}-\frac{1}{3}+\cdots +\frac{1}{n}-\frac{1}{n+1}+\frac{1}{n+1}=1.
\end{align*}

(3)设$a_1+a_2+\cdots+a_r=1$,其中$2\leqslant r\leqslant 2024$.

若$r=2024$,则$m=1$;

若$r<2024$,则$a_1+a_2+\cdots+a_{2024}=(a_1+a_2+\cdots+a_r)+
a_{r+1}+\cdots+a_{2024}<1+2024-r\leqslant 2023$,所以$m\leqslant 2022$.

当$m=2022$时,考虑$A:1-\frac{1}{2},1-\frac{1}{2},1-\frac{1}{2^2},1-\frac{1}{2^2},\cdots ,1-\frac{1}{2^{1011}},1-\frac{1}{2^{1011}},
1-\frac{1}{2^{1011}},1-\frac{1}{2^{1011}}$,
$A$中所有数之和为
$$
2\sum_{k=1}^{1011}{\left( 1-\frac{1}{2^k} \right)}+2\left( 1-\frac{1}{2^{1011}} \right) =2\left( 1011-\frac{\frac{1}{2}\left( 1-\frac{1}{2^{1011}} \right)}{1-\frac{1}{2}} \right) +2\left( 1-\frac{1}{2^{1011}} \right) =2022.
$$

而$a_1+a_2=1$, $a_1+a_2+\cdots+a_{2024}-(a_1+a_2)=a_3+a_4+\cdots+a_{2024}
=2022-1=2021$.

又当$2\leqslant k\leqslant 1011$时,
\begin{align*}
a_1+a_3+\cdots +a_{2k-1}+a_{2k} &=\sum_{i=1}^k{\left( 1-\frac{1}{2^i} \right)}+1-\frac{1}{2^k}\\
&=k-\frac{\frac{1}{2}\left( 1-\frac{1}{2^k} \right)}{1-\frac{1}{2}}+1-\frac{1}{2^k}=k,
\end{align*}
此时有$a_1+a_2+\cdots+a_{2024}-(a_1+a_3+\cdots +a_{2k-1}+a_{2k})=2022-k\in\{1011,1012,\cdots,2020\}$.

故对任意$N\in \{1,2,\cdots,2022\}$,均存在$A_k$使得$S(A_k)=N$.

因此$m$的最大可能值为$2022$.

 

Roth 定理
%https://math.fandom.com/zh/wiki/Roth_%E5%AE%9A%E7%90%86

【新定义模拟题】
设$N\in \mathbf{N}^\ast,N\geqslant 4$,记$[N]=\{1,2,\cdots,N\}$.若$A\subseteq [N]$,且不存在$x,y,z\in A,x\neq z$,使得$x,y,z$依次成等差数列,则称$A$为$N$的简单集,并记$N$的简单集中元素个数的最大值为$M(N)$.


设$N\in \mathbf{N}^\ast,N\geqslant 4$.若$A\subseteq \{1,2,\cdots,N\}$,且不存在$x,y,z\in A$,使得$x,y,z\ (x\neq z)$依次成等差数列,则称$A$为$N$的简单集,称元素个数最多的$A$为$N$的最大简单集,并记$N$的最大简单集的元素个数为$M(N)$.


(1)求$M(4)$及$4$的最大简单集;

(2)求$M(14)$; %$M(14)>M(7)$


(3)证明: $M(2024)\leqslant 1012$.


(1) $M(4)=3$,此时所对应的简单集$A=\{1,3,4\}$或$\{1,2,4\}$;

(2)求$M(14)$; %$M(14)>M(7)$


(3)先证明: $M(2n)\leqslant n$,其中$n\geqslant 8,n\in \mathbf{N}^\ast$.


设$1\leqslant a_1<a_2<\cdots<a_r\leqslant N$,且$\{a_1,a_2,\cdots,a_r\}$为$N$的最大简单集,
则$N+1-a_r<N+1-a_{r-1}<\cdots<N+1-a_1$也为$N$的最大简单集.\quad \ding{172}

设$k<a_1,k\in \mathbf{N}^\ast$,则$a_1-k<a_2-k<\cdots<a_r-k$,
且$\{a_1-k,a_2-k,\cdots,a_r-k\}$为$N$的最大简单集. \quad \ding{173}

 

 

设$m,n\in \mathbf{N}^\ast$.
若$\{a_1,a_2,\cdots,a_i,a_{i+1},\cdots,a_j\}$为$m+n$的最大简单集,且$a_1<a_2<\cdots<a_i\leqslant m<a_{i+1},\cdots,a_j\leqslant m+n$,则$M(m+n)=j$.
由于$\{a_1,a_2,\cdots,a_i\}$为$m$的简单集,
$\{a_{i+1}-m,\cdots,a_j-m\}$为$n$的简单集,
由最大简单集的性质可知$i\leqslant M(m),j-i\leqslant M(n)$,
故$M(m+n)=j=i+(j-i)\leqslant M(m)+M(n)$. \quad \ding{174}


若$M(8)=5$,由\ding{172}和\ding{173}可知,不妨设$1$和其余两个不超过$4$的正整数属于$8$的最大简单集.则$8$的最大简单集要么包含$1,2,4$,要么包含$1,3,4$,经检验可知此时$M(8)=5$不成立.
故$M(8)\leqslant 4$.又因为$\{1,2,4,5\}$为$8$的简单集,
故$M(8)=4$.

因为$\{1,2,4,8,9\}$为$9$的简单集,
故$M(9)=5$.

 

若$M(10)=6$,则由$M(8)=4$和\ding{173}可知$1,2,9,10$属于$10$的最大简单集.但$3,5,6$和$8$不能出现,故$10$的最大简单集只可能为$\{1,2,4,7,9,10\}$,由于$1,4,7$成等差数列,矛盾!故$M(10)\leqslant 5$.又因为$\{1,2,4,9,10\}$为$10$的简单集,
故$M(10)=5$.

因为$\{1,2,4,8,9,11\}$为$11$的简单集,
故$M(11)=6$.

若$M(12)=7$,则由上述讨论可知$1,2,11,12$属于$12$的最大简单集.则由$M(8)=4$和\ding{173}可知$4,9$也属于$12$的最大简单集.故$12$的最大简单集包含$1,2,4,9,11,12$,但不能包含其它元素,故只能为$\{1,2,4,9,11,12\}$.
故$M(12)=6$.

又因为$\{1,2,4,5,10,11,13,14\}$为简单集,故$M(13)=7$, $M(14)=8$.

(i)由\ding{174}可知$M(16)\leqslant 8$, $M(18)\leqslant 9$, $M(20)\leqslant 10$, $M(22)\leqslant 11$.

(ii)假设当$n=k,k\geqslant 8$时,有$M(2n)=M(2k)\leqslant k$,
则当$n=k+4$时,由\ding{174}可知$M(n)=M(2k+8)\leqslant M(2k)+M(8)\leqslant k+4=n$.
故当$n=2k+8$时,结论$M(2n)\leqslant n$也成立.

根据(i) (ii)可知, $M(2n)\leqslant n$.

因此$M(2024)\leqslant 1012$.

 


证明: $M(4048)\leqslant 2024$.

(3)证明: $M\left( \frac{3^k+1}{2} \right) \geqslant 2^k$.

事实上,若$u\leqslant \frac{3^k+1}{2}$且$u$不包含三进制中的数字$2$,则$u+1$构成简单集.

猜测$M\left( \frac{3^k+1}{2} \right)=2^k$.


下面证明:对任意$\varepsilon >0$和$N>N_0(\varepsilon)$,有
$$
M\left( N \right) <\left( \frac{4}{9}+\varepsilon \right) N.
$$

首先证明$M(17)=8$.由$M(14)=8$可知$M(17)\geqslant 8$.

若$M(17)=9$,因为$M(14)=8$,则$1$和$17$必须同时出现.但$9$不能出现,由\ding{173}可知$M(17)\leqslant M(8)+M(8)=8$.

则$M(34)\leqslant 16$, $M(35)\leqslant 16$.否则,若$M(35)\geqslant 17$,则由$M(34)\leqslant 16$可知$1$和$35$必须出现,则$18$不能出现,因为$1,18,35$是等差数列.
因此$M(35)\leqslant 16$.

类似地$M(71)\leqslant 32$, $\cdots$, $M\left( 2^k+2^{k-3}-1 \right) \leqslant 2^{k-1}$.

类似地,可以证明$M(18)=M(19)=M(20)=8$, $M(21)=9$,因为$1,3,4,8,9,16,18,19,21$为简单集.进一步有$M(22)=M(23)=9$.
故对足够大的$N>N_0(\varepsilon)$,有$M\left( N \right) <\left( \frac{3}{8}+\varepsilon \right) N$.


【打磨】
设$N\in \mathbf{N}^\ast,N\geqslant 4$.若$A\subseteq \{1,2,\cdots,N\}$,且不存在$x,y,z\in A$,使得$x,y,z\ (x\neq z)$依次成等差数列,则称$A$为$N$的简单集,称元素个数最多的简单集为$N$的最大简单集,并记$N$的最大简单集的元素个数为$M(N)$.


(1)求$M(4)$及$4$的最大简单集;

(2)求$M(8)$; %$M(14)>M(7)$


(3)设$k,n\in \mathbf{N}^\ast$,当$n\geqslant k$时, $M(2n)\leqslant n$恒成立,求$k$的最小值.


(1) 由题意可知$M(4)=3$,所求$4$的最大简单集为$\{1,3,4\}$或$\{1,2,4\}$.


(2) \textbf{解法一.}先证明两个结论:

设$1\leqslant a_1<a_2<\cdots<a_r\leqslant N$,且$\{a_1,a_2,\cdots,a_r\}$为$N$的最大简单集,
则有

 

\ding{172} $N+1-a_r<N+1-a_{r-1}<\cdots<N+1-a_1$,
且$\{N+1-a_r,N+1-a_{r-1},\cdots,N+1-a_1\}$也为$N$的最大简单集.\quad \ding{172}

 

\ding{173} 设$k<a_1,k\in \mathbf{N}^\ast$,则$a_1-k<a_2-k<\cdots<a_r-k$,
且$\{a_1-k,a_2-k,\cdots,a_r-k\}$为$N$的最大简单集.

若$M(8)=5$,由\ding{172}和\ding{173}可知,不妨设$1$和其余两个不超过$4$的正整数属于$8$的最大简单集.则$8$的最大简单集要么包含$1,2,4$,要么包含$1,3,4$,经检验可知此时$M(8)=5$不成立.
故$M(8)\leqslant 4$.又因为$\{1,2,4,8\}$是$8$的简单集,
故$M(8)=4$.

\textbf{解法二.}由于$\{1,2,\cdots,8\}=\{1,3,5,7\}\cup \{2,4,6,8\}$,则$\{1,3,5,7\}$中最多只能取三个数: $1,3,7$或$1,5,7$,而$\{2,4,6,8\}$中最多也只能取三个数: $2,4,8$或$2,6,8$,故$M(8)\leqslant 3+3=6$.

在$1,2,3,4,7,8$或$1,2,3,6,7,8$中, $1,2,3$成等差数列,矛盾!

在$1,2,4,5,7,8$中, $1,4,7$成等差数列,矛盾!

在$1,2,5,6,7,8$中, $6,7,8$成等差数列,矛盾!

故$M(8)\leqslant 5$.

若从$\{1,3,5,7\}$中取两个数: $1,3$或$1,5$或$1,7$或$3,5$或$3,7$或$5,7$, 从$\{2,4,6,8\}$中取三个数: $2,4,8$或$2,6,8$,
经检验可知这些由五个元素构成的集合均不是简单集.

若$\{1,3,5,7\}$中取三个数: $1,3,7$或$1,5,7$, $\{2,4,6,8\}$中取两个数: $2,4$或$2,6$或$2,8$或$4,6$或$4,8$或$6,8$,经检验可知这些由五个元素构成的集合均不是简单集.

故$M(8)\leqslant 4$.

又$\{1,2,4,8\}$为简单集,故$M(8)=4$.


(3)设$m,n\in \mathbf{N}^\ast$.
若$\{a_1,a_2,\cdots,a_i,a_{i+1},\cdots,a_j\}$为$m+n$的最大简单集,且$a_1<a_2<\cdots<a_i\leqslant m<a_{i+1}<\cdots<a_j\leqslant m+n$,则$M(m+n)=j$.
由于$\{a_1,a_2,\cdots,a_i\}$为$m$的简单集,
$\{a_{i+1}-m,\cdots,a_j-m\}$为$n$的简单集,
由最大简单集的性质可知$i\leqslant M(m),j-i\leqslant M(n)$,
故$M(m+n)=j=i+(j-i)\leqslant M(m)+M(n)$.

因此$M(m+n)\leqslant M(m)+M(n),m,n\in \mathbf{N}^\ast$. \quad \ding{174}


由(2)可知$M(8)=4$,因为$\{1,2,4,8,9\}$为$9$的简单集,
故$M(9)=5$.

 

若$M(10)=6$,则由$M(8)=4$和\ding{173}可知, 不妨设$1,2,9,10$属于$10$的最大简单集.但$3,5,6$和$8$不能出现,故$10$的最大简单集只可能为$\{1,2,4,7,9,10\}$,由于$1,4,7$成等差数列,矛盾!故$M(10)\leqslant 5$.又因为$\{1,2,4,9,10\}$为$10$的简单集,
故$M(10)=5$.

因为$\{1,2,4,8,9,11\}$为$11$的简单集,
故$M(11)=6$.

若$M(12)=7$,则由上述讨论可知,不妨设$1,2,11,12$属于$12$的最大简单集.则由$M(8)=4$和\ding{173}可知$4,9$也属于$12$的最大简单集.故$12$的最大简单集包含$1,2,4,9,11,12$,但不能包含其它元素,故只能为$\{1,2,4,9,11,12\}$.
故$M(12)=6$.

又因为$\{1,2,4,5,10,11,13,14\}$为简单集,故$M(13)=7$, $M(14)=8$.即$n=7$时, $M(2n)> n$.

下面利用数学归纳法证明: $M(2n)\leqslant n$,其中$n\geqslant 8,n\in \mathbf{N}^\ast$.

 

(i)由\ding{174}可知$M(16)\leqslant M(8)+M(8)=8$, $M(18)\leqslant M(10)+M(8)=9$, $M(20)\leqslant M(10)+M(10)= 10$, $M(22)\leqslant M(10)+M(12)=11$.

(ii)假设当$n=k,k\geqslant 8$时,有$M(2n)=M(2k)\leqslant k$,
则当$n=k+4$时,由\ding{174}可知$M(n)=M(2k+8)\leqslant M(2k)+M(8)\leqslant k+4=n$.
故当$n=2k+8$时,结论$M(2n)\leqslant n$也成立.

根据(i) (ii)可知,当$k\geqslant 8$时, $M(2n)\leqslant n$.

因此所求$k$的最小值为$8$.

 

$p$进数简介

 

$p$进数最早来源于数论的研究, 使用幂级数来研究数. 现在$p$进数的影响已经很深远, 例如可以在$p$进数上建立$p$进分析. $p$进数需要配合一种奇妙的$p$进绝对值一起使用. $p$进绝对值给出了不同于寻常绝对值的比较数的大小的标准, 全体有理数将在数轴上重新排序, 进而收敛这个概念也会重新定义, 原先发散的级数将会变得收敛, 例如$9+90+900+9000+\cdots$在某种意义下可以等于$-1$. $p$进绝对值也带来了一种新的距离(度量), 它比寻常的距离要强, 可以称为超距离, 超距离下有新的几何,比如所有三角形都是等腰三角形,圆内的任意一点都是圆心等等. 有些问题用$p$进数会得到意料之外的解决方案,较著名的例子是Monsky定理:一个正方形不可能分割成奇数个面积相等的三角形. 正文中将会详细介绍这个例子. $p$进数内容实在太多, 本文只作简要介绍,希望能让读者体会到$p$进数世界的奇妙.

\subsection{相似之处}

 

整数和整式存在着相似之处.整数($\pm1$除外)可以分解为一些素数的积,而整式(常数除外)可以分解成一次式的乘积:
\begin{align*}
N&=\pm p_{1}^{\alpha_{1}}p_{2}^{\alpha_{2}}\cdots p_{n}^{\alpha_{n}},\\
P(z)&=c(z-a_{1})^{m_{1}}(z-a_{2})^{m_{2}}
\cdots(z-a_{n})^{m_{n}}.
\end{align*}


对比发现素数$p$与$z-a$的“地位”相同. 再来看,一个整数可以表示成$p$进制,而一个整式可以用$z-a$来展开.
\begin{align*}
N&=a_0+a_1p+a_2p^2+\cdots+a_np^n,\\
P(z)&=a_0+a_1(z-a)+a_2(z-a)^2+\cdots+a_n(z-a)^n.
\end{align*}


我们进一步来将有理数与有理式进行对比. 有理式能写成两个整式之比,并且在 $z=a$处, $f(z)=P(z)/Q(z)$可以展开成
$$\frac{a_{-n}}{(z-a)^n}+\cdots+\frac{a_{-1}}{z-a}
+a_0+a_1(z-a)+a_2(z-a)^2+\cdots,$$
这其实就是复分析里的洛朗级数了,和泰勒级数的区别是,它允许出现负幂项.例如要求将$\left(x+1\right)/(x-1)$用$x-1$展开,那么
$$(x+1)/(x-1)=2(x-1)^{-1}+1.$$
复杂一点的例子是$1/(x^2-1)$用$x-1$展开,那么
$$\frac{1}{x^{2}-1} =\frac{1}{x-1}\cdot\frac{1}{2+(x-1)}.$$

若$|x-1|<2$,那么
$$\frac{1}{2+(x-1)}= {\frac{1}{2}}{\frac{1}{1+{\frac{x-1}{2}}}}
={\frac{1}{2}}\sum_{n=0}^{\infty}
\left(-{\frac{x-1}{2}}\right)^{n},$$
于是
\begin{align*}
\frac{1}{x^{2}-1} =\sum_{n=-1}^\infty\frac{(-1)^{n+1}}{2^{n+2}}
(x-1)^n.
\end{align*}

如果做一个类比, $P(z)/Q(z)$类比成有理数$P/Q$, $z-a$类比成素数 $p$,那么应该有展开式
$$a_{-n}p^{-n}+\cdots+a_{-1}p^{-1}+a_0
+a_1p+a_2p^2+\cdots,$$
这将是本文的主角: $p$进数($p$-adic numbers).


$p$进数最早在1897年由德国数学家Kurt Hensel引进,他从其导师,著名数学家克罗内克(Leopold Kronecker)处学习到整数环$\mathbb{Z}$及其分式域$\mathbb{Q}$与多项式环$\mathbb{C}[x]$及其分式域之间的相似之处,以此为出发点试图建立统一的理论.将级数应用到数论中,这是引进$p$进数最初的动机. $p$进数是研究数论问题的强有力工具,最著名的例子应是怀尔斯所证明的费马大定理. 不仅如此,可以引入新的绝对值将有理数集完备化得到$p$进数域 $\mathbb{Q}_p$.普通的微积分建立在实数域$\mathbb{R}$上,而 $\mathbb{Q}_p$上可以建立新的微积分,这就是$p$进分析了.


著名数学家巴拿赫(Stefan Banach)曾说:合格的数学家能看到定理之间的相似之处,优秀的数学家能看到定理证明之间的相似之处,而卓越的数学家能看到数学分支之间的相似之处,最后,究极的数学家能俯瞰这些相似之处的相似之处.这段话发人深省,可以说,数学的一大魅力在于类比,通过发现相似之处走向更深处,若能发现两件看似关系遥远的事物之间的相似之处,那就是绝妙的数学,两件事物表面的关系越远, 结论越震撼.

\subsection{形式上的 10 进数,镜像世界}

 

通常所说的$p$进数, $p$指的是素数或者素数的幂,但由于我们比较熟悉十进制,这里就先介绍$10$进数,方便做比较. 需要郑重声明的是: $p$进数不是单纯的进制游戏,和 $p$进制数有着明显区别.

定义 2.1 形如
$$\sum_{k=-n}^\infty a_k10^k,\quad a_k\in \{0,1,2,\cdots,9\}$$
的数称为$10$进数.如果引入小数点,可以写成
$$\cdots a_1a_0.a_{-1}\cdots a_{-n}.$$
这里需要注意,无限的部分写在小数点左侧,而不是右侧,例如
$$\cdots333.3$$
是一个$10$进数,然而$3.333\cdots$不是$10$进数. 数的记法和通常的记法相反,互为镜像,有
人称 $p$ 进数为”左撤子数”, 这是比较形象的,因为现在的数需要从右往左书写.


$10$进数的加法和减法不难定义,只要把级数对应项的系数相加减,系数若有大于等于$10$的需要进位,通过列竖式的办法来计算是比较简单的,比如:
$$\begin{array}{cc}
&\cdots 33333.33\\
+&\phantom{\cdots}\, 22222.78\\
\hline
=&\cdots 55556.11\end{array}$$

计算是从右往左进行的,这比无限位实数的加法要容易实现.


$p$进数的乘法等同于级数的乘法,最后也要注意进位的问题,通过列竖式来计算乘法是比较方便的,例如:
$$\begin{array}{cc}
&\cdots3333.33\\
\times &\phantom{\cdots }\, 15.\\\hline
&\cdots6666.65\\
&\cdots3333.33\\\hline
=&\cdots9999.98
\end{array}$$


$$\begin{array}{rr}
&\cdots 3333 \\
\times &\cdots 2222\\\hline
&\cdots 6666 \\
&\cdots 66660 \\
&\cdots 666600 \\
&\cdots 6666000\\\hline
=&\cdots 5925926
\end{array}
$$

减法也可以将级数的系数相减,但是$p$进数的世界里不需要负数.例如$-2$其实等于$\cdots 9998$,这是因为通过列竖式可以观察到$\cdots9998+2=0.$
$$\begin{array}{cr}&\cdots9998\\
+&2\\\hline=&\cdots0000\end{array}$$

类似地,每一个$10$进数的“相反数”都是正的,所以负数在$10$进制的世界里是没有必要的,不需要考虑数的符号的问题. $10$进数$\cdots a_2a_1a_0.a_{-1}\cdots a_{-\pi}$的相反数
$$(9-a_2)(9-a_1)(9-a_0).(9-a_{-1})\cdots
(9-a_{-(n-1)})(10-a_{-n})$$
仍然是$10$进数.


说到这里,肯定有疑问了,这个疑问一开始就埋下了.像$\cdots 333$这样的数,在通常意义下不是无穷大吗?级数$\sum_{k=-n}^\infty a_k10^k$是发散的呀,为什么就能用来定义“数”?


一个隐蔽的很深的事实是:当我们说一个级数收敛或发散的时候,需要使用极限,而极限的定义依赖于绝对值.例如我们所熟知的“一尺之棰,日取其半,万世不竭”:
$$\frac12+\frac1{2^2}+\frac1{2^3}+\cdots=1,$$
上式左端项随着求和项的增加,与右端越来越“接近”,这个所谓的接近,就是要用距离来刻画.通常所使用的距离是利用绝对值来定义的,即:两个实数$x$和$y$之间的距离是$|x-y|$. 那么,现在若将这个距离替换掉,那会如何呢?

 

通常情况下,正整数$n$越大, $10^n$越大, $10^{-n}$越小,现在反过来,引入一种新的绝对
值$|\cdot|_{10}$,定义
$$|10^n|_{10}=10^{-n},$$
也就是说含有的$10$的幂次越高反而越小.


两个数$1234$和$2234$在$10$进制下相差$1000$,但若改用$10$进绝对值,两个数仅相差$0.001$.度量数的大小的标准改了,就会有不一样的风景出现.


更一般的,对于一个正的有理数$r$,若存在整数$k$和正整数$a,b$,使得
$$r=10^k\frac{a}{b},\quad \gcd(a,b)=1,\gcd(b,10)=1,\gcd(a,10)<10,$$
那么
$$|r|_{10}=10^{-k}.$$

例如
$$|1.23|_{10}=2,|500|_{10}=10^{-2},
|1/3|_{10}=1.$$
补充定义$\left|0\right|_{10}=0$.

 

两个数$x$和 $y$之间的距离有一个新的定义:
$$|x-y|_{10},$$
该定义重新解释了什么是“接近”,也可以认为是将有理数在数轴上重新排列.%给出新的


可以看到
\begin{align*}
|9-(-1)|_{10} &=10^{-1},\\
|99-(-1)|_{10}&=10^{-2},\\
|999-(-1)|_{10}&=10^{-3},\\
|9999-(-1)|_{10}&=10^{-4},
\end{align*}
这意味者
$$|\underbrace{9\cdots9}_{n\text{个}9}-(-1)|_{10}
=10^{-n}\to 0\ (n\to\infty),$$
也就是说
$$\cdots 9999=-1,$$
如果改写成级数形式,那就是
$$9+9\times 10+9\times 10^2+9\times 10^3+\cdots=-1.$$
这个结论在通常意义下是错误的,但要注意现在我们更换了度量标准,衡量世界的尺度发生了变化,那么看待世界的方式也随之改变.


来考虑除法.例如$1/3$能否表示成$10$进数?这意味着解方程
$$3x=1,$$
但是不在实数范围内解.


第一步,解
$$3a=1\quad \bmod\ 10$$
得出$a=7$.

第二步,从$1$当中扣除掉$3\times7=21$,得到$\cdots 99980$,解
$$3b=8\quad \bmod\ 10$$
得出$b=6$.

 

第三步, $\cdots 99980$中进一步扣除$3\times 6\times 10=180$,得到$\cdots 99800$解
$$3c=8\quad \bmod\ 10$$
这和第二步一样,得出$c=6$.

之后一直出现循环,于是 $1/3=\cdots 6667$,不妨将$\cdots 6667$简记成$\overline{6}7$.

可以列除法努式如下:
$$\begin{array}{rl}
\cdots 66667 &\\
\overline{\cdots 00001}| &3\\
21 &\\
\overline{\cdots 99980} &\\
180 &\\
\overline{\cdots 99800}&\\
1800 &\\
\overline{\cdots 98000}&\\
\end{array}$$

可用乘法竖式来检验:
$$\begin{array}{rr}
&\cdots 6667\\
\times&3\\\hline
=&\cdots0001
\end{array}$$


$1/7$表示成$10$进数会复杂一些,列竖式的方法也是可以的,不过这里打算换一个办法.我们知道$10^6= 1\quad \bmod\ 7$,那么通过计算
\begin{align*}
&\frac{10^6-1}{7}=142857\\
&\frac{10^{12}-1}{7}=142857142857\\
&\frac{10^{18}-1}{7}=142857142857142857
\end{align*}


于是
$$-\frac17=\overline{142857}.$$


为了得出$1/7$,用$X$表示$10$,将$0$写成$\cdots 9999X$,然后列竖式
$$\begin{array}{cr}
\cdots 99999999999X\\
-&\cdots 142857142857\\\hline
=&\cdots 857142857143
\end{array}$$
这表示
$$\frac17=\overline{285714}3.$$
注意和熟知的$\frac17=0.\overline{142857}$进行对比,可以发现$10$进制表示和$10$进表示是不同的.但
我们注意到
\begin{align*}
-\frac{1}{7}&=\overline{142857}\\
\frac{1}{7}&=0.\overline{142857}
\end{align*}

事实上,将$7$换成与$10$互素的数都是对的,若$\gcd (q,10)=1$,那么
$$\frac1q=0.\overline{a_1\cdots a_k}\Leftrightarrow -\frac1q=\overline{a_1\cdots a_k}.$$


熟知有理数可以表示成循环小数,循环小数一定是有理数.在$10$进数的世界里也是如此.严格的证明在定理6.2中给出.这里简单证明一下循环的$10$进数一定是有理数.假定$x=\overline{a_1\cdots a_k}$,那么
$$x-10^kx=\overline{a_1\cdots a_k}-\overline{a_1\cdots a_k}0\cdots 0=a_1\cdots a_k,$$
于是
$$x=-\frac{a_1\cdots a_k}{9\cdots 9}$$
是有理数.可以发现这个证明和以前证明循环小数是有理数的过程几乎一致,彷佛是在镜子中做证明.

\subsection{$p$进绝对值, Monsky定理}


$10$进数可以作加法减法和乘法,也就是说$10$进数全体是一个环,然而除法可能会出问题.例如将$1/5$表示成$10$进数,第一步需要求解
$$5a=1\quad \mod\ 10,$$
然而这是无解的.若把$10$改成素数或素数的幂,就不会发生此类问题.以素数$p$为基底的数
$$\sum_{k=-n}^\infty a_kp^k,\quad a_k\in\{0,1,\cdots,p-1\}$$
构成一个数域,记为$\mathbb{Q}_p$.

 

现在考虑收敛性的问题.级数$\sum_{k=-n}^\infty a_kp^k$通常意义下是发散的.级数的收敛与发散依赖于一个非常隐蔽的概念:绝对值,这个概念度量了数的大小.为了使得级数在某种意义下收敛,需要给出新的度量数的大小的标准,比如原先很大的数,在新的标准下其实很小,这样就可能使得发散的级数变得收敛.


之前定义了$10$进绝对值,现在对于一个素数$p$,可以定义$p$进绝对值.


定义 3.1 若有理数$r=p^n\frac ab,\gcd(a,b)=1,\gcd(a,p)=1,\gcd(b,p)=1$,定义$r$的$p$进赋值$v_p(r)=n$,定义$r$ 的$p$进绝对值 $|r|_p=p^{-v_p(r)}=p^{-n}$.补充定义$|0|_p=0$.


从定义看,如果一个数含有的$p$的幂次越高,它就越小.例如$r=1125/56=2^{-3}\cdot 3^2\cdot 5^3\cdot 7^{-1}$,有
\begin{align*}
&|r|_{2}=2^{3}=8,\\
&|r|_{3}=3^{-2}=1/9,\\
&|r|_{5}=5^{-3}=1/125,\\
&|r|_{7}=7,\\
&|r|_{p}=1\ (p\text{是}2,3,5,7\text{以外的素数}).
\end{align*}


可以发现,当$p$取遍所有景数时,
$$\prod_p|r|_p=|r|^{-1}.$$

有了绝对值以后,可以刻画距离的概念.对于两个有理数$x$ 和$y$,定义它们之间的$p$进距离$d(x,y)$为$|x-y|_p$.


和通常的绝对值一样, $p$进绝对值满足下面这三条性质:


1. $|x| _p\geqslant 0$, 并且$|x|_p=0\Leftrightarrow x=0$.


2. $\left|xy\right|_{p}=\left|x\right|_{p}
\left|y\right|_{p}$.

3. $|x+y|_p\leqslant |x|_p+|y|_p$.

 

几何上讲, $|x|_p$表示$x$到原点的距离大小.依次来看各条件,第一条说明绝对值非负,这是自然的,因为距离非负,同时,若一个数到原点的距离为零,那么它只能是原点.第二条性质说明$|x|_p$是一个积性函数,积的绝对值等于绝对值的积.第三条性质不太平凡,在分析学中起到重要作用,通常称为三角形不等式.另一方面,此条件比起加性条件$|x+y|_p=|x|_p+|y|_p$要弱一些,不妨称为次可加性.


我们来证明$|\cdot|_{p}$满足三角形不等式,这条性质看上去不显然.


假设$x=p^m\frac ab,y=p^n\frac cd$,其中$a,b$互素, $c,d$互素, $a,b,c,d$均与$p$互素,那么$|x|_p=p^{-m},|y|_p=p^{-n}$.


不妨假设$m\geqslant n$,那么
$$x+y=p^n\frac{p^{m-n}ad+bc}{bd},$$
容易看出$p$与$bd$互素,如果$m>n$,则$p$与$p^{m- n}ad+bc$互素,此时$|x+y|_p=p^{-n}$.但若 $m=n$,那么 $p^{m-n}ad+bc=ad+bc$可能含有因子$p$,此时$|x+y|_p<p^{-n}$.无论如何,
$$|x+y|_p\leqslant p^{-n}\leqslant p^{-m}+p^{-n}=|x|_p+|y|_p.$$

从证明过程来看,实际上是证明了更强的结果
$$| x+y|_p\leqslant \max\{| x|_p,| y|_p\}$$
以及

定理 3.1 若$|x_p|\neq |y|_p$,那么$|x+y|_p=\max\{|x|_p,|y|_p\}$.

 

利用刚才证明的$p$进绝对值的性质,容易看出$p$进距离$d(x,y)=|x-y|_p$满足
$$d(x,y)\leqslant\max\{d(x,z),d(z,y)\}.$$


这个不等式比起通常的三角形不等式$d(x,y)\leqslant d(x,z)+d(z,y)$要来得强,可以称$d$为“超距离”.

 

来感受一下超距离下几何世界的奇妙.如果任意给出三个有理数 $a,b,c$,如果$|a-b|_p$和$|b-c|_p$不相等,那么 $|a-c|_p=|(a-b)+(b-c)|_p$一定等于$|a-b|_p$和$|b-c|_p$中较大的一个,换句话说:三个有理数两两之间的$p$进距离必有两个相等!几何上来讲, $p$进三角形一定是等腰三角形.


如果定义$p$进圆 $S(a,r)=\left\{x\in\mathbb{Q}\mid \left|x-a\right|_p<r\right\}$,该圆以$a$为圆心, $r$为半径.那么任取一个$b\in S(a,r)$,对于任意的$x\in S(a,r)$,有$|x-b|_p\leqslant \max\{|x-a|,|a-b|\}<r$,这意味着圆内每一个点都是圆心,通常情况下是不可想象的.还可以证明两个圆要么不相交,要么一个包含另一个.给出两个圆$S(a,r)$和$S(b,s)$,若有$x\in S(a,r)\cap S(b,s)$,由于圆内每一个点都是圆心,因此$S(a,r)=S(x,r),S(b,s)=S(x,s)$这就表示两个圆之中的一个包含另一个.


定理3.1可以推:

定理 3.2 若$a_1,a_2,\cdots,a_n$满足$\left|a_1\right|_p>\left|a_k\right|_p\ (k=2,\cdots,n)$,那么
$$|a_1+a_2+\cdots+a_n|_p=| a_1|_p.$$


证明: $|a_2+\cdots+a_n| _p\leqslant \max\{|a_2|_p,\cdots,|a_n|_p\}<|a_1|_p$,根据定理3.1可知$|a_1+(a_2+\ldots+a_n)|_p=\left|a_1\right|_p$.


也就是说,若干个数若有一个唯一的$p$进绝对值最大者,那么这些数之和的$p$进绝对值就等于这个最大者的 $p$进绝对值.


小试牛刀,来证明如下一个命题:


例 1 调和数$H_n=1+1/2+\cdots+1/n$当$n>1$时不是整数.

这是一道颇有难度的数论题,不过在$2$进数的世界中,此命题不难论证.对于指定的$n>1$,存在唯一的$m>0$使得$2^m\leqslant n\leqslant 2^{m+1}$,这表示$1,1/2,\cdots,1/n$这些数的$2$进绝对值存在唯一的最大者,它就是 $2^m$,于是根据定理3.2,有
$$|H_n|_2=|1/2^m|_2=2^m>1.$$
而整数的$2$进绝对值根据定义不可能大于$1$,这就表示$H_n$不可能是整数.


接下来的例子是非常精妙的.

例 2 一个正方形可以分割成偶数个面积相等的三角形,这是显而易见的,然而一个正方形能否分割成奇数个面积相等的三角形?


这个问题的难度相当大(三角形不要求全等),在1965年Fred Richman和John Thomas在美国数学月刊上提出此问题. 1968年, John Thomas证明了:如果三角形顶点坐标是分母为奇数的有理数,那么分割是不存在的. 1970年Paul Monsky在John Thomas的基础上证明了一定不存在满足要求的分割,证明中非常巧妙地用到了$2$进绝对值.一个正方形不可能分割成奇数个面积相等的三角形,这个结论称为Monsky定理.

 

不妨假定这个正方形为$\left[0,1\right]\times\left[0,1\right]$.假设该正方形能分割成$n$个面积相等的三
角形,那么每个三角形的面积为$1/n$.


虽然$2$进绝对值的记号是$|\cdot|_2$,但我们会反复用到,接下来省去下标$2$,用记号$|\cdot|$表示$2$进绝对值.


将点$\left(x,y\right)$分成三类并染色.

$R$类: $|x|<1$ 且$|y|<1$, 染成红色;

$G$类: $|x|\geqslant 1$ 且$|x|\geqslant |y|$,染成绿色;

$B$类: $|y|\geqslant 1$ 且$|y|>|x|$,染成蓝色.


我们将从代数上证明:一个三角形若三个顶点颜色不同(不妨称为“三色三角形”),这样的三角形面积不可能为$1/n$ ($n$为奇数).再从组合上证明:若有满足条件的分割,必定存在一个三色三角形,三个顶点分别属于$A,B,C$三类. 这两件事情足以证明Monsky定理.

 

假设有一个三色三角形,其三个顶点$\left(x_1,y_1\right)$, $\left(x_2,y_2\right)$, $\left(x_3,y_3\right)$分别属于$R,G,B$三
类,那么
$$\left|x_1\right|<1,\left|y_1\right|<1,
\left|x_2\right|\geqslant 1,\left|x_2\right|\geqslant \left|y_2\right|,\left|y_3\right|\geqslant 1,\left|y_3\right|>\left|x_3\right|.$$


利用行列式可以计算出三角形的面积(相差一个符号)为
$$\frac{1}{2}\det\left(\begin{bmatrix}
x_1&y_1&1\\
x_2&y_2&1\\
x_3&y_3&1
\end{bmatrix}\right)
=\frac{1}{2}(x_1y_2+x_2y_3+x_3y_1
-x_2y_1-x_3y_2-x_1y_3).$$

表达式中涉及到六项: $x_1y_2,x_2y_3,x_3y_1,x_2y_1,x_3y_2,x_1y_3$,下述不等式说明$2$进绝对值最大的一项是$x_2y_3$,其余五项的$2$进绝对值都严格小于该项的$2$进绝对值:
\begin{align*}
&|x_1y_2|=|x_1|\cdot|y_2|<|y_2|\leqslant |x_2|\leqslant |x_2y_3|,\\
&|x_3y_1|=|x_3|\cdot|y_1|<|x_3|<|y_3|\leqslant |x_2y_3|,\\
&|x_2y_1|=|x_2|\cdot|y_1|<|x_2|\leqslant |x_2y_3|,\\
&|x_3y_2|=|x_3|\cdot|y_2|<|y_3|\cdot|x_2|
=|x_2y_3|,\\
&|x_1y_3|=|x_1|\cdot|y_3|<|y_3|\leqslant |x_2y_3|.
\end{align*}


根据定理3.2可知
$$|x_1y_2+x_2y_3+x_3y_1-x_2y_1-x_3y_2-x_1y_3|=
|x_2y_3|=|x_2|\cdot |y_3|\geqslant 1.$$


这件事情意味着三色三角形的面积不可能为零,也就是说三种颜色的点不可能出现在一条直线上 .记$G=x_1y_2+x_2y_3+x_3y_1-x_2y_1-x_3y_2-x_1y_3$,若三色三角 形面积为$1/n$,那么$G=\pm 2/n$.一 方 面,根据刚才的讨论 , $|G| \geqslant 1$,另一方面,注意到$n$为奇数,
$$|G|=|2|\cdot|1/n|=1/2\cdot 1<1,$$

这个矛盾说明三色三角形的面积不可能为$1/n$.


Monsky定理的证明完成了一半,接下来只要证明必定存在三色三角形即可.假设正方形能分割成奇数个三角形. 考虑一端为红色,一端为绿色的边(简称红绿边)的个数,彩虹三角形的红绿边个数为$1$.非彩虹三角形的红绿边个数为$0$或$2$.


考虑正方形边界上红绿边的个数.底边之外的三条边上没有红绿边.底边上的端点$(0,0)$是红色的,端点$(1,0)$是绿色的,底边中的三角形顶点要么是红色,要么是绿色,因此底边的红绿边的个数一定是奇数. 综上,正方形边界处的红绿边个数是奇数.


是时候完成最后一击了.统计所有三角形的红绿边个数总和,正方形内部的红绿边个数被统计了两次(因为一条边被两个三角形共用),而正方形边界上的红绿边个数为奇数,因此所有三角形红绿边数目总和是奇数.由于三色三角形红绿边个数为奇数,非三色三角形红绿边个数为偶数,因此一定存在三色三角形!

\subsection{局部-整体原则}

 


数学分析里,我们知道有理数域$\mathbb{Q}$在绝对值$|\cdot|$下完备化成实数系$\mathbb{R}$,现在,将绝对值换成$p$进绝对值,将得到另一个完备的数域 $\mathbb{Q}$,具体细节在最后一节给出.当我们在有理数域上考虑一个问题时,把问题放到更大的数域可能是方便的,例如考虑有理系数一元二次方程$x^2+ax+b=0$的有理根,可以先在复数域内考虑问题,此时的解可以全部写出: $x=\frac{-a\pm\sqrt{a^2-4b}}2$.
为了让$x$是有理数,那么$a^2-4b$就一定是有理数的平方.现在,有理数域不仅能扩张成实数域和复数域,还有无穷多个扩张$\mathbb{Q}_{2}$,这样就有了很多新的看问题的角度.数域$\mathbb{Q}_p$长期以来一直隐藏在实数域的光芒之下不为我们所知,当发现这崭新的风景时,才会感叹还有如此广阔的新世界.


为了描述方便,以下记$\mathbb{Q}_\infty=\mathbb{R}$, 这仅是一个记号,没有深意.


对于一个正整数$n$,写出其分解式$n=p_1^{\alpha_1}p_2^{\alpha_2}\cdots p_k^{\alpha_k}$,我们知道,
$$|n|_{p_i}=p_i^{-\alpha_i},$$
并且$|n|_p=1$ ($p$不是$p_1,\cdots,p_k$中任何一个).当$p$取遍所有素数时,
$$\prod_p|n|_p=|n|^{-1}=|n|_\infty^{-1}.$$

从这个表达式来看,左边集中了所有$\mathbb{Q}_p$中的信息,而右侧是$\mathbb{Q}_\infty=\mathbb{R}$的信息.每一个$\mathbb{Q}_p$中的信息称为“局部”的信息,局部的信息拼凑起来可以得到整体的信息,这是一个重大的启示.


通过一个简单的例子来说明局部整体原则.考虑求平方根的问题,求$a$的平方根,相当于解方程$x^2=a$,必须要声明的是在什么数域里进行求解.如果$x^2=a$在数域$\mathbb{K}$上有解,就称$a$在$\mathbb{K}$上是完全平方.一个较为简单的结论是:


例 3 一个有理数 $x$是完全平方当且仅当它在所有$\mathbb{Q}_p\ \left ( p\leqslant \infty \right )$中是完全平方.


证明:根据
$$|x|=\prod_pp^{v_p(x)}$$
可知,若$x$在$\mathbb{Q}_p$中是完全平方,那么$v_p(x)$是偶数,从而$x$是完全平方.另一个方向是
显然的,因为$\mathbb{Q}$是$\mathbb{Q}_p$的子集.

%%https://mp.weixin.qq.com/s?__biz=Mzg4NTU5ODQyNw==&mid=2247483740&idx=1&sn=6fe1114836ba0cbeccaf51ca009c29eb&chksm=cfa730e8f8d0b9febac8b72bb05e80068ed565f8582553612cae335d9ef2a9d5fdaf367e78da&mpshare=1&scene=1&srcid=0515T8dYFj4jP9zebzoekv2a&sharer_shareinfo=909a02567f49e09e43fe9ecf902d880b&sharer_shareinfo_first=6a3cf4f1d966fd6187770d3dd8da743a#rd


刚才讨论的是非常简单的方程$x^2=a$,可以考虑更加复杂的丢番图方程,所谓的整体$-$局部原则是指:考察一个丢番图方程在$\mathbb{Q}$中解的存在性(整体解)可以通过考察方程在$\mathbb{Q}_p\ (p\leqslant\infty)$中的解(局部解)的存在性来获得.这是指导方针,指明了研究方向,但不能说是定理.应用该原则,一个成功的并且是著名的结论如下:

定理 4.1 Hasse-Minkowski定理

$F(X_1,\cdots,X_n)=\sum c_{ij}X_iX_j$是有理数域上的二次型,那么方程
$$F(X_1,\cdots,X_n)=0$$
在$\mathbb{Q}$中有非平凡解当且仅当在每一个$\mathbb{Q}_p\ \left(p\leqslant\infty\right)$中有非平凡解.


作为该定理的一个特例,可以给出二次曲线$ax^2+by^2=1\ \left(a,b\in\mathbb{Q} ^\ast=\mathbb{Q}\backslash\{0\}\right)$上有理点的判定条件: $ax^2+by^2=1$在$\mathbb{Q}$上有解当且仅当$ax^2+by^2=1$在$\mathbb{Q}_p$ ( $p$为素数和$\infty$)中有解.


\subsection{绝对值}

 

本节探讨一般意义下的绝对值.

定义 5.1 一个定义域为有理数域$\mathbb{Q}$,取值为有理数的单变量函数$\varphi$,满足下述三个条件:

1. (正定性) $\varphi(x)\geqslant 0$且 $\varphi(x)=0\Leftrightarrow x=0$,


2. (积性) $\varphi ( xy) = \varphi ( x) \varphi ( y)$,


3. (三角形不等式,次可加性) $\varphi (x+y)\leqslant \varphi (x)+\varphi (y)$,
那么$\varphi$称为绝对值.

 

通常使用的绝对值$\varphi(x)=|x|$满足上述条件,可以认为是将原先绝对值的概念作了推广.


根据这三条性质,不难得出一些简单的结论.


首先有 $\varphi(1)=1$, 这是因为 $\varphi(1)=\varphi(1^2)=\varphi(1)^2$ (积性), 并且$\varphi(1)\neq 0$ (正定性).


类似地, $\varphi(-1)=1$.

其次, $\varphi(x)=\varphi(-x)$,这是由于积性以及$\varphi(-1)=1$.


再次,若$x\neq 0$,则$\varphi(x^{-1})=[\varphi(x)]^{-1}$.这是由于积性以及$\phi(1)=1$.

 

最后,根据次可加性,不难得出对于正整数$n$成立$\varphi(n)\leqslant n$.另外还有$\varphi(a-b)\geqslant \varphi(a)-\varphi(b)$.


若定义
$$\varphi(x)=\begin{cases}
1, &x\neq 0,\\
0, &x=0,
\end{cases}$$
那么$\varphi$是绝对值,这称为平凡的绝对值或者常数绝对值,这种情况比较无趣,在考虑问题时通常将其排除.


若定义$\varphi(x)=x^2$,那么它不是绝对值,因为它不满足三角形不等式.但若定义$\varphi(x)=\sqrt{|x|}$,那么它是绝对值.

 

要构造一个绝对值出来,可能是不太容易的.不过,若已经有绝对值$\varphi$,那么$\varphi^s\ \left(0<s<1\right)$也是绝对值,下面来证明这件事.


正定性和积性容易验证,只证明三角形不等式.


需要证明$\varphi^s(x+y)\leqslant \varphi^s(x)+\varphi^s(y)$,已知$\varphi(x+y)\leqslant \varphi(x)+\varphi(y)$,只需证明$(\varphi(x)+\varphi(y))^s\leqslant
\varphi^s(x)+\varphi^s(y)$即可.
只要证明对于正数$a$和$b$以及$0<s<1$,有$(a+b)^s\leqslant a^s+b^s$.不妨假设$a\geqslant b$,根据拉格朗日中值定理,存在$\xi\in(a,a+b)$使得
$$(a+b)^s-a^s=s\xi^{s-1}b.$$
由于$s\xi^{s-1}b\leqslant 1\cdot a^{s-1}b\leqslant b^s$,因此$(a+b)^s\leqslant a^s+b^s$.

 

这样看来,绝对值有无穷多个,但是$\varphi$和$\varphi^s$给出相同的排序结果,换句话说,如果$\varphi(a)<\varphi(b)$,那么$\varphi^s(a)<\varphi^s(b)$,反之亦然.这两种绝对值我们称为等价的.若将等价的绝对值视为同一个,那么绝对值有几个呢?答案令人惊讶,除去平凡的绝对值外,只有两个.这件事情不显然,论证过程较长.

 

引理 5.1 $\varphi$是非平凡的绝对值,那么所有与之等价的绝对值必定具有形式$\varphi^{s}\ \left(s>0\right)$.

证 明:由于$\varphi$非常数,因此必定存在$a_0$使得 $0<\varphi(a_0)<1$ (否则考虑 $\varphi(a_0^-1)$).


对于任一有理数$a$以及正整数$m$和$n$,假设绝对值$\psi$和$\varphi$等价,那么
$$\varphi(a)^m<\varphi(a_0)^n\Leftrightarrow
\psi(a)^m<\psi(a_0)^n,$$

$$\frac{m}{n}<\frac{\ln\varphi(a_0)}{\ln\varphi(a)}
\Leftrightarrow\frac{m}{n}<\frac{\ln\psi(a_0)}
{\ln\psi(a)}.$$
$\frac{\ln\varphi(a_0)}{\ln\varphi(a)}$
和$\frac\ln\psi(a_0){\ln\psi(a)}$必然相等,否则两者之间存在一个有理数,这个有理数大于其
中一者而小于另一者,这与事实$\frac mn<\frac{\ln\varphi(a_0)}{\ln\varphi(a)}
\Leftrightarrow\frac mn<\frac{\ln\psi(a_0)}{\ln\psi(a)}$矛盾.

根据
$$\frac{\ln\varphi(a_{0})}{\ln\varphi(a)}
=\frac{\ln\psi(a_{0})}{\ln\psi(a)}$$
得出
$$\frac{\ln\psi(a_{0})}{\ln\varphi(a_{0})}
=\frac{\ln\psi(a)}{\ln\varphi(a)}.$$

 

记$s=\frac\ln\psi(a_0){\ln\varphi(a_0)}$,这是一个与$\varphi,\psi$和$a_0$有关的正的常数,并且有$\frac\ln\psi(a){\ln\varphi(a)}=s$,
即$\psi=\varphi^s$.

 

定理 5.1 Ostrouski定理.
$\mathbb{Q}$上的非平凡绝对值要么等价于普通绝对值$|\cdot|$,要么等价于$p$进绝对值$|\cdot|_p$.

 

证明:给定一个非平凡的绝对值$\varphi$,分两种情况:

Case 1. 存在一个正整数$n_0$满足$\varphi(n_0)>1$.

Case 2. 对于任意正整数$n$恒有$\varphi(n)\leqslant 1$.

 

第一种情况称为阿基米德绝对值,将证明它一定与普通绝对值等价.第二种情况称为非阿基米德绝对值,将证明它与某个$p$进绝对值等价.

 

考虑阿基米德绝对值的情形,找一个最小的$n_0>1$满足$\varphi(n_0)>1$.取$s=\frac{\ln\varphi(n_0)}{\ln n_0}$,则$s>0$且$\varphi(n_0)=n_0^s$.


对于任意的 $n$,将之表示成 $n_0$进制,即
$$n=a_0+a_1n_0+\cdots+a_kn_0^k,\quad 0\leqslant a_i<n_0,0\leqslant i\leqslant k.$$
于是$n_0^k\leqslant n<n_0^{k+1}$,并且
$$\varphi(n)\leqslant\varphi(a_0)
+\varphi(a_1)\varphi(n_0)+\cdots
+\varphi(a_k)[\varphi(n_0)]^k.$$
根据$n_0$的定义,可以知道$\varphi(a_i)\leqslant 1$,因此
\begin{align*}
\varphi(n)\leqslant &1+\varphi(n_{0})+\cdots+[\varphi(n_{0})]^{k}\\
=&1+n_{0}^{s}+\cdots+(n_{0}^{s})^{k}\\
=&\frac{(n_{0}^{s})^{k+1}-1}{n_{0}^{s}-1}
<\frac{(n_{0}^{s})^{k+1}}{n_{0}^{s}-1}
=n_{0}^{ks}\frac{n_{0}^{s}}{n_{0}^{s}-1}.
\end{align*}
记$C=\frac{n_0^s}{n_0^s-1}$,那么
$$\varphi(n)\leqslant Cn_0^{ks}\leqslant Cn^s.$$

接下来的论证体现出了典型的数学分析技巧,将要证明$\varphi(n)=n^s$.

 

不等式$\varphi(n)\leqslant Cn^s$对于任意的$n$成立,那么对于$n^N$自然也成立,即
$$\varphi(n^N)\leqslant Cn^{Ns}.$$
两边取$1/N$次方,得到
$$\varphi(n)\leqslant C^{1/N}n^s.$$
现在这个表达式对任意$N$成立,那么令$N\to \infty$,得出
$$\varphi(n)\leqslant n^s.$$
只要得到反向估计就可以了,根据三角形不等式有
$$\varphi(n)\geqslant\varphi(n_0^{k+1})
-\varphi(n_0^{k+1}-n).$$
由于$\varphi(n_0^{k+1})=n_0^{s(k+1)},
\varphi(n_0^{k+1}-n)\leqslant (n_0^{k+1}-n)^s$ (已证),因此
\begin{align*}
\varphi(n)\geqslant & n_{0}^{s(k+1)}-(n_{0}^{k+1}-n)^{s}\\
\geqslant &n_{0}^{s(k+1)}-(n_{0}^{k+1}-n_{0}^{k})^{s}\\
=&n_{0}^{s(k+1)}\left(1-\left(1-\frac{1}{n_{0}}
\right)^{s}\right)\\
>&Dn^{s},
\end{align*}
其中常数$D=1-\left(1-\frac1{n_0}\right)^s$.使用刚才的小技巧,就能得出$\varphi(n)\geqslant n^s$.


这样就证明了对于所有正整数有$\varphi(n)=n^s$.由于$\varphi(-n)=\varphi(n)$,那么对所有整数$n$成立$\varphi(n)=|n|^s$,不难得出对于所有有理数$a$有$\varphi(a)=|a|^s$,证得阿基米德绝对值与普通绝对值等价.

 

考虑非阿基米德绝对值的情形.现在对于所有正整数$n$有$\varphi(n)\leqslant 1$,由于$\varphi$非平凡,可假设$n_0$是满足$\varphi(n_0)<1$的最小者.


可以证明 $n_0$一定是素数.否则假设$n_0=ab$, $a$和 $b$小于$n_0$,根据$n_0$的最小性,得
出$\varphi(a)=\varphi(b)=1$,因此$\varphi(n_0)=\varphi(a)\varphi(b)=1$,矛盾.


改记$n_0$为$p$,将证明$\varphi$与$|\cdot|_p$等价.

根据带余除法,任意一个$n$可以写成
$$n=pq+r,\quad 0\leqslant r<p.$$
如果 $r\neq 0$,也就是说 $n$不能被 $p$整除时 $\varphi(n)=1$.那么根据$p$的最小性,有$\varphi(r)=1$.
$\varphi(pq)=\varphi(p)\varphi(q)\leqslant
\varphi(p)<1$,于是$\varphi(n)=\max\{\varphi(pq),\varphi(r)\}=1$.


任意的正整数$n$可以写成
$$n=p^kq,$$
这里$q$不被$p$整除,根据刚才的论证, $\varphi(q)=1$,于是
$$\varphi(n)=[\varphi(p)]^k=c^{-k},$$
其中$c=[\varphi(p)]^-1$.取$s$满足$c=p^s$,那么$\varphi(n)=(p^{-k})^s$与$|\cdot|_p$等价.

 

对于阿基米德绝对值和非阿基米德绝对值这样的命名,在此做出解释,实数满足一个所谓的阿基米德性质:任意的实数$a$和$b$,存在正整数$n$使得$na>b$,也可以表述为:对于任意的实数$a$,存在正整数$n$使得$n>a$.

 

需要注意这里比大小使用的是通常的绝对值,现在可以改成其它绝对值,此时阿基米德性质是否成立呢?我们已经证明了阿基米德绝对值和普通绝对值等价,因此阿基米德绝对值仍旧有阿基米德性质.不过非阿基米德绝对值,比如$p$进绝对值就没有这个性质了. 给出$a$和$b$,对于非阿基米德绝对值$\varphi$而言,不一定存在正整数$n$使得$\varphi(na)>\varphi(b)$. 取$\varphi(a)\leqslant\varphi(b)$,那么对于任意的正整数$n$, $\varphi(na)=\varphi(n)\varphi(a)\leqslant \varphi(a)\leqslant\varphi(b)$,即知所言为真.

 

至此,所有的绝对值可以分为两类:阿基米德绝对值与非阿基米德绝对值.平凡的绝对值是非阿基米德绝对值,而非平凡的绝对值与$|\cdot|$或$|\cdot|_p$等价.


\subsection{$p$进数的严格理论}


\subsubsection{数的扩张}

 

假设现在只是知道有理数,我们要创造新的数.按数学分析教材,有理数在通常的绝对值下可以完备化为实数,现在仿照这个过程,只不过将绝对值换成$|\cdot|_p$ ($p$是素数或者$\infty$).

 

定义 6.1 给出一列有理数$\{a_n\}$,若对于任意有理数$\varepsilon>0$,存在一个正整数$N$ (和$\varepsilon$有关),使得 $m,n>N$时,恒有$|x_m-x_n|_p<\varepsilon$,则称数列$\{a_n\}$为柯西列.


形象地说,从一个数列的第一项开始往后看,只要看得足够远,那么任意两项都足够接近.

 

对于数列$\{a_n\}$和$\{b_n\}$,很自然地,可以定义它们的和,差与积:
\begin{align*}
\{a_n\}+\{b_n\}&=\{a_n+b_n\},\\
\{a_n\}-\{b_n\}&=\{a_n-b_n\},\\
\{a_n\}\cdot\{b_n\}&=\{a_n\cdot b_n\}.
\end{align*}
这里没有违反直觉的地方.

如果$a_n$和$b_n$是柯西列,那么根据三角形不等式,有
$$|(a_m+b_m)-(a_n+b_n)|_p\leqslant
|a_m-a_n|_p+|b_m-b_n|_p,$$
可知$\{a_n+b_n\}$是柯西列,类似地, $\{a_n-b_n\}$也是柯西列.

 

乘积$\{a_nb_n\}$也是柯西列,不过证明起来稍微复杂一些.先证明柯西列是有界的.

若取$\varepsilon=1$可知存在$N$ (这个数现在是确定的),对于$m,n>N$有$|a_m-a_n|_p<1$,那么有$\left|a_n-a_{N+1}\right|_p<1\ \left(n>N\right)$.根据三角形不等式
$$|a_n|_p\leqslant |a_n-a_{N+1}|_p+| a_{N+1}|_p<1+|a_{N+1}|_p,\quad n>N.$$
无穷多项都被一个常数控制着,剩下的是有限项$a_1,\cdots,a_N$,若记$a_1,\cdots,a_N,a_{N+1}+1$中最大一项的值为$M$,那么对于所有$n$均有$|a_n|_p\leqslant M$ ($M$与$n$无关).


接下来根据
$$|a_mb_m-a_nb_n|_p\leqslant | a_m-a_n|_p\cdot| b_m|_p+| a_n|_p\cdot |b_m-b_n|_p$$
可以知道$\{a_nb_n\}$也是柯西列.


如果熟悉抽象代数,可以知道全体柯西列形成一个含幺交换环.


现在,这个数列$\left\{a_n\right\}$可能最终无限接近于某个有理数$a$,严格地说,给出如下关于收敛性的定义.

 

定义 6.2 对有理数列$\{a_n\}$,若存在有理数$a$,使得任意有理数 $\varepsilon>0$,存在正整数$N$ (和$\varepsilon$有关),对于任意$n>N$,有$|a_n-a|_p<\varepsilon$,称数列$\{a_n\}$在$|\cdot|_p$下收敛到$a$.


例如对于有理数$a$,常数列$a,a,a,\cdots$在$|\cdot|_p$下收敛到$a$.


$a_n=1/n$在$|\cdot|$下收敛到$0$.

$a_n=p^n$在$|\cdot|_p$下收敛到$0$.

若$\left\{a_n\right\}$和$\left\{b_n\right\}$的极限分别为$a$和$b$,那么$\left\{a_n+b_n\right\}$, $\left\{a_n-b_n\right\}$, $\left\{a_nb_n\right\}$的极限也
是存在的,分别为$a+b,a-b,ab$.

 

现在讨论两个数列的商. 若$b_n$不收敛到$0$,那么定义$\{a_n\}$和$\{b_n\}$的商为
$$\{a_n/b_n\}.$$
$\{b_n\}$虽然有为零的项,但仅有有限项,弃去后不会影响关于极限的讨论.


可以证明:若$\{a_n\}$和$\{b_n\}$都是柯西列, $\{b_n\}$不收敛到$0$,那么商$\{a_n/b_n\}$是柯西
列.证明过程比较复杂.根据
$$\frac{a_m}{b_m}-\frac{a_n}{b_n}
=\frac{a_m(b_n-b_m)+b_m(a_m-a_n)}{b_mb_n}$$
可知
$$\left|\frac{a_m}{b_m}-\frac{a_n}{b_n}\right|_p
\leqslant (|a_m|_p|b_n-b_m|_p+|b_m|_p|a_m-a_n|_p)
|b_m|_p^{-1}|b_n|_p^{-1}.$$
如果$\left|b_n\right|_p^{-1}$是有界的,那么容易得出商是柯西列.为了使得$\left|b_n\right|_p^{-1}$有界,需要让它最后远离$0$,也就是说存在正有理数$c>0$,存在正整数$N$,对于任意$n>N$,有$|b_n|_p\geqslant c>0$.接下来证明的大意如下,首先能找到一个子列远离零,由于是柯西列,项数足够多时,两项之间的距离可以足够小,这有希望让数列最终远离零.已知$\left\{b_n\right\}$不以零为极限,那么存在一个正数$\varepsilon$,对于任意正整数$N$,存在正整数$n$ 使得$|b_n|_p\geqslant\varepsilon$.这意味着能找到一列$n_1<n_2<n_3<\cdots$使得$|b_{n_k}|_p\geqslant\varepsilon$.


由于$\left\{b_n\right\}$是柯西列,那么存在$N_1$,只要$m,n>N_1$,就有$|b_m-b_n|_p<\varepsilon/2$.假设$n_1,n_2,\cdots$中大于$N_1$的最小数为$n'$.

现在,取$c=\varepsilon/2$, $N=n'$,对于$n>N$,有
$$|b_n|_p\geqslant |b_{n'}|_p-|b_n-b_{n'}|_p\geqslant \varepsilon-\varepsilon/2=c.$$


两个有理数列可能收敛到同一个有理数.例如$\left\{1/n\right\}$和$\left\{2/n\right\}$
在$\left|\cdot\right|$下都收敛到$0$.对于收敛到同一个有理数的数列,我们在某种程度上将之视为相同的,不加区分的.


定义 6.3 数列$\left\{a_n\right\}$和 $b_n$之差以$0$为极限,则称两个数列同余,记 为
$$\{a_n\}=\{b_n\}\quad \bmod\overline{\{0\}}.$$
记号$\overline{\left\{0\right\}}$表示以$0$极限的数列全体,这个集合对于加法,数乘和乘法运算都封闭.如果熟悉批象代数,可以知道这个集合是一个理想.在微积分中,无穷小量全体其实就是一个理想.

 

不难发现同余关系是一个等价关系,按等价关系可以将所有柯西列进行分类. 同一类的柯西列是同余的,不同类的柯西列不同余.在每一个类中选出一个代表$\left\{a_n\right\}$来表示该类,记为$\overline\{a_n\}$.例如$\overline{\{1/n\}}=\overline{\{0\}}$.


类之间可以运算. $\overline{\{a_n\}}$和$\overline\{b_n\}$中选取代表元$\{a_n\}$和$\{b_n\}$,那么
\begin{align*}
\overline{\{a_{n}\}}+\overline{\{b_{n}\}}
&=\overline{\{a_{n}+b_{n}\}},\\
\overline{\{a_{n}\}}-\overline{\{b_{n}\}}
&=\overline{\{a_{n}-b_{n}\}},\\
\overline{\{a_{n}\}}\cdot\overline{\{b_{n}\}}
&=\overline{\{a_{n}\cdot b_{n}\}},
\end{align*}
若$\overline{\{b_n\}}\neq \overline{\{0\}}$,可以定义
$$\overline{\{a_n\}}/\overline{\{b_n\}}
=\overline{\{a_n/b_n\}},$$
不难验证,运算的结果与代表元的选取无关.

 

定义 6.4 按同余关系进行分类,所有类形成的集合称为有理数在$|\cdot|_p$下的扩张,每一类称为该扩张下的一个数.在通常的绝对值下,此时的扩张即是实数系统,在$p$进绝对值$| \cdot | _p$下的扩张称为$p$进数系统,记为 $\mathbb{Q}_p$.

 

$p$进数系统中已经包含了所有有理数,对于有理数$a$,类$\overline{\{a\}}$在系统中且与$a$一一对应.


对于通常的绝对值$|\cdot|=|\cdot|_\infty$,一列有理数可能不收敛,或者说收敛的结果不是有理数,例如
$$a_1=1,a_{n+1}=\frac{1}{2}\left(a_n
+\frac{2}{a_n}\right).$$
(事实上,该数列收敛到$\sqrt2$,然而$\sqrt2$不是有理数).


实数系统中包含着不是有理数的数, $p$进数系统也是如此,下一节将给出$p$进数的表达式.


现在通过有理数$\mathbb{Q}$扩张得到$\mathbb{Q}_p$,那么$\mathbb{Q}_p$能否继续扩张?我们知道有理数$\mathbb{Q}$在绝对值$|\cdot|$下扩张成实数$\mathbb{R}$.实数$\mathbb{R}$在绝对值$|\cdot|$下扩张仍旧是实数$\mathbb{R}$.这里就有一个所谓的完备性的概念,假设扩张后得到的系统不能再扩张,那么系统是完备的.换句话说,柯西列的极限仍旧在系统之中,有理数是不完备的,因为有理数的柯西列,其极限未必是有理数,然而实数是完备的,实数柯西列的极限仍是实数,不是新的数.我们来证明$\mathbb{Q}_p$在$|\cdot|_p$下是完备的.


$|\cdot|_p$本来只是定义在$\mathbb{Q}$上,在扩张后,对于$\mathbb{Q}_p$中的一个元素$a$,这实际上是$\mathbb{Q}$上的一个柯西列$\{a_n\}$,可以定义
$$|a|_p=\lim_{n\to\infty}|a_n|_p,$$
这样的话$|\cdot|$的定义域延拓到了$\mathbb{Q}_p$.


修改柯西列之定义,假定我们已经清楚实数的含义了.给出$\mathbb{Q}_p$中的一列$\{a_n\}$,若对于任意实数$\varepsilon>0$,存在一个正整数$N$ (和$\varepsilon$有关),使得$m,n>N$时恒有$|a_m-a_n|_p<\varepsilon$,则称$\left\{a_n\right\}$为柯西列.注意,刚才柯西列定义中的有理数$\varepsilon$改成了实数.

 

修改收敛性的定义,对$\mathbb{Q}_p$中的一列$\left\{a_n\right\}$,若存在$a\in\mathbb{Q}_p$使得任意实数$\varepsilon>0$,存在正整数 $N$ (和$\varepsilon$有关),对于任意 $n>N$ 有$\left|a_n-a\right|_p<\varepsilon$,
称$\left\{a_n\right\}$在$\left|\cdot\right|_p$下收敛到$a$.

 

定理 6.1 $\mathbb{Q}_p$在$|\cdot|_p$下是完备的.


证明:给出$\mathbb{Q}_p$中的柯西列: $a_1,a_2,a_3,\cdots$,其中$a_k$是$\mathbb{Q}$中的柯西列: $a_k^ {(1)},a_k^{(2)},a_k^{(3)},\cdots$的
极限.


存在$N=N(k)$使得$n\geqslant N$时,有
$$|a_n^{(k)}-a_k|_p\leqslant\frac{1}{k}.$$
考虑有理数列 $\{a_n^{N(n)}\}_{n=1}^\infty$,根据
\begin{align*}
|a_{m}^{N(m)}-a_{n}^{N(n)}|_p &\leqslant |a_m^{N(m)}-a_m|_p+|a_m-a_n|_p
+|a_n-a_n^{N(n)}|_p\\
&\leqslant\frac{1}{m}+|a_{m}-a_{n}|_{p}
+\frac{1}{n}
\end{align*}
以及$\left\{a_n\right\}$是柯西列可知$\left\{a_n^{N(n)}\right\}$也是柯西列,假设极限是$a\in\mathbb{Q}_p$,那么由
$$|a_n-a|_p\leqslant |a_n-a_n^{N(n)}|_p+|a_n^{N(n)}-a|_p$$
可知$\{a_n\}$的极限是$a$, $\mathbb{Q}_p$中的序列在$|\cdot|_p$下的极限仍旧在$\mathbb{Q}_p$中.


\subsection{$p$进数的表达式}

 

我们知道实数没有表达式,有理数虽然在绝对值下完备化后得到了实数,但是我们对实数仍旧有很多不消楚的地方.然而$p$进数的表达式却是消楚的,并且是一个完备的数域.


先来看有理数$a/b$ ($a,b$是互素的整数, $p\nmid b)$的$p$进表示法.可研究同余式
$$bx\equiv a\quad \bmod\ p^k,0\leqslant x<p^k$$
的解,设为$x_k$.可知存在整数$c$使得
$$bx_k-a=p^kc,$$
因此$x_k-\frac ab=p^k\frac cb$,从而
$$\left|x_k-\frac{a}{b}\right|_p\leqslant p^{-k}.$$
这意味着$x_k$在$|\cdot|_p$下收敛到$a/b$.

 

$x_k$可以表示为
$$x_k=a_0+a_1p+a_2p^2+\cdots+a_{k-1}p^{k-1},\quad 0\leqslant a_i<p.$$
根据
\begin{align*}
|x_{m}-x_{n}|_{p}&\leqslant |a_{n+1}|_{p}p^{-(n+1)}+\cdots+|a_{m}| p^{-m}\\
&\leqslant p^{-(n+1)}+\cdots+p^{-m}\\
&<\frac{p^{-(n+1)}}{1-p^{-1}},\quad m>n
\end{align*}

不难知道$\{x_n\}$是柯西列,极限不妨记为
$$a_0+a_1p+a_2p^2+\cdots,\quad 0\leqslant a_i<p$$
就是有理数$a/b$的$p$进表示,这样的数是有理数在$|\cdot|$下的极限,因此是$\mathbb{Q}$中的元素.

 

考虑有理数$a/b$, $a,b$是互素的正整数,但是$p\mid b$,假设$m$是最大的使得$p^m\mid b$的正整
数.那么$p^ma/b=a/(b/p^m)$就是刚才所讨论的有理数,它可以表示成
$$a_0+a_1p+a_2p^2+\cdots,\quad 0\leqslant a_i<p,$$
于是$a/b$可以表示成
$$p^{-m}(a_0+a_1p+a_2p^2+\cdots),\quad 0\leqslant a_i<p.$$
这样的级数就是有理数表示成$p$进数的一般形式,和普通幂级数比起来,它允许负幂项的出现.或者这个级数可以写成小数的形式
$$\cdots a_m.a_{m-1}\cdots a_1a_0,$$
无限的部分写在左边.以前我们知道循环小数是有理数,有理数也能写成循环小数,现在是否有类似的结论呢?

 

定理 6.2 有理数的$p$进表示是$p$的循环级数, $p$的循环级数是有理数.

 

证明:假设有一个$p$的循环幂级数
$$x=p^{-m}(A+p^rB+p^{r+s}B+p^{r+2s}B+\cdots),$$
其中$A=a_0+a_1p+\cdots+a_{r-1}p^{r-1}$, $B=b_0+b_1p+\cdots+b_{s-1}p^{s-1}$,那么
$$p^mx-A=\frac{p^rB}{1-p^s},$$
从而得出$x$是有理数.


假设有一个绝对值小于$1$的有理数$a/b$, $a,b$互素, $a<0,b>0$,且$p\nmid b$,根据数论中的欧拉定理:若 $a$与$n$互素,那么$a^{\phi(n)}\equiv 1\ \bmod\ n$,可知存在一个最小的正整数$m$使得
$$p^m\equiv 1\ \bmod\ b.$$
也就是说存在正整数$c$使得$p^m-1=bc$.
$$\frac ab=\frac{ac}{bc}=\frac{-ac}{1-p^m}.$$
正整数$-ac$可以表示为
$$a_0+a_1p+\cdots+a_{m-1}p^{m-1},$$
这是由于$-ac$不超过$p^m-1$,因此不会有幂次超过$p^{m-1}$的项.


而$\frac1{1-p^m}$可以表示为
$$1+p^m+p^{2m}+\cdots $$

从而
$$a/b=(a_0+\cdots+a_{m-1}p^{m-1})
+p^m(a_0+\cdots+a_{m-1}p^{m-1})
+p^{2m}(a_0+\cdots+a_{m-1}p^{m-1})+\cdots $$


如果正有理数$a/b$中的$b$满足$p\mid b$,假设$k$是最大的使得 $p^k\mid b$的正整数,那么$p^ma/b$可
以表示成
$$b_0+b_1p+\cdots+b_np^n+r/s,$$
$r/s$或者为零,或者绝对值小于$1$, $r<0,s>0$, $p\nmid s$,仍旧可以表示成循环级数.


对于负有理数$-x$而言,可以将$0$写成
$$p+(p-1)p+(p-1)p^2+\cdots $$
然后利用减法$0-x$获得$-x$的循环表示.


现在就知道了,若一个级数不是循环的,那它就不是有理数,也就是说$\mathbb{Q}_p$这个集合的确比$\mathbb{Q}$要大.


$$p^{-m}(a_0+a_1p+a_2p^2+\cdots),\quad 0\leqslant a_i<p.$$
是$\mathbb{Q}$中的元素,也就是$p$进数,那么反过来,所有的$p$进数都是这样的级数形式吗?回答是肯定的.

 

已经知道任一$p$进数是由有理数在$|\cdot|_p$下取极限得到的,现在任意有理数$a_n$可以表示为
$$a_n=p^{-m_n}(a_n^{(0)}+a_n^{(1)}p+a_n^{(2)}p^2
+\cdots)$$
那么序列$\{a_n\}$的极限是否也是这样的形式?对于任意正整数$k$,存在$N$,若 $m,n>N$,有$|a_m-a_n|_p<p^{-k}$.这表示$n>N$时, $a_n,a_{n+1},a_{n+2},\cdots$的$p$进表示的前$k$项是相同的.由于$k$是任意的,这就完成了证明.


参考文献

[1] Fernando Q. Gouvěa. p- adic Numbers: An Introduction (Third Edition) [M].

[2] 冯克勒. $p$进数[M]. 湖南教育出版社, 1995.

[3]华罗庚.华罗庚文集数论卷II [M].科学出版社, 2010.

[4] Paul Monsky. On Dividing A Square into Triangles (J]. The American Mathematical Monthly, 77: 2, 161-164.

[5] Andrew Rich. Leftist Numbers [J]. The Colledge Mathematics Journal, 39:5, 330-336

[6] %\href{https://en.m.wikipedia.iwiki.eu.org/wiki/P-adic number}{维基百科}%\url{https://en.m.wikipedia.iwiki.eu.org/wiki/P-adic number}


(1)证明:存在无穷多个公比不为$\pm 1$的等比数列,使得其中三项依次成等差数列.

考虑高次方程$1+q^{2n+1}=2q^2$.

(2)若等比数列的公比$q\in \mathbf{N}^\ast,q\geqslant 2$,证明:该等比数列中不存在三项依次成等差数列.


(深圳中学2024届高三年级高考适应性测试)根据代数基本定理,给定正整数$n$,方程$z^n=1$有$n$个复数根,分别是$z_{k}=\cos\frac{2k\pi}n+ \mathrm{i}\sin\frac{2k\pi }n$, $k=1,2,\cdots,n$,这些根称为$n$次单位根,此时有 $z^n-1=(z-z_1)(z-z_2)\cdots(z-z_n)$.
当$k$与$n$互质时, $z_k$称为$n$次本原单位根. $n$次本原单位根的另一个等价定义是,若$a^n=1$,且不存在小于$n$的正整数$m$使得$a^m=1$,则称复数$a$为$n$次本原单位根.


给定正整数$n$,我们定义$n$级分圆多项式$C_n(x)=(x-\varepsilon_1)(x-\varepsilon_2)\cdots (x-\varepsilon_l)$,其中$\varepsilon_1,\varepsilon_2,\cdots,\varepsilon_r$是全部$n$次本原单位根.

(1)写出$C_2(x),C_3(x),C_6(x)$,并化简.

(2)若$p$是质数(或称素数),求出$C_p(x)$并化成最简形式.

(3)探究是否存在正整数$q,r$和$s$使得$C_q(x)\equiv C_r(x)C_q(x)$对任意复数$x$恒成立.

(1)
\begin{align*}
C_{2}(x)&=x+1,\\
C_{3}(x)&=\left(x-\left(\cos\frac{2\pi}{3}+
\mathrm{i}\sin\frac{2\pi}{3}\right)\right)
\left(x-\left(\cos\frac{4\pi}{3}+
\mathrm{i}\sin\frac{4\pi}{3}\right)\right)
=x^{2}+x+1.
\end{align*}

当$n=6$时,与$6$互质且小于等于$6$的正整数只有$1$和$5$,从而
$$
C_6\left( x \right) =\left( x-\left( \cos \frac{\pi}{3}+\text{i}\sin \frac{\pi}{3} \right) \right) \left( x-\left( \cos \frac{5\pi}{3}+\text{i}\sin \frac{5\pi}{3} \right) \right) =x^2-x+1.
$$

(2)当$p$是质数时,与$p$互质且小于等于$p$的正整数有$1,2,\cdots,p-1$,只有$p$与自己不互质.根据分圆多项式的定义,我们有$C_p(x)=\left(x-z_1\right)\left(x-z_2\right)\cdots \left(x-z_ {p-1}\right)$,其中$z_k=\cos\frac{2k\pi}p+\mathrm{i}\sin\frac{2k\pi}p$, $k=1,2\ldots,p-1$,
$z_p=\cos\frac{2p\pi}p+\mathrm{i}\sin\frac{2p\pi}p=1$.
由题意$x^p-1=(x-z_1)(x-z_2)\cdots (x-z_p)$,
从而$C_p(x)=\frac{x^p-1}{x-z_p}=\frac{x^p-1}{x-1}=x^{p-1}+\cdots+x+1$.

(3)不存在,我们采用反证法.

假设存在正整数$q,r$和$s$使得$C_q(x)\equiv C _r(x)C_s(x)$对任意复数$x$恒成立.

若$a$为$q$次本原单位根,则$a^q=1$且$C_q(a)=0$,从而有$C_r(a)C_s(a)=C_q(a)=0$.

若$C_r(a)=0$,则$C_r(a)=\left(a-\varepsilon_1\right)
\left(a-\varepsilon_2\right)\cdots \left(a-\varepsilon_k\right)=0$,则存在$1\leqslant j\leqslant k$使得$a-\varepsilon_j=0$,即$a=\varepsilon_j$成立,从而$a$ 为$r$次本原单位根, $a^r=1$.根据本原单位根的等价定义,此时必有$r=q$,否则可导出矛盾.这时我们有$C_q(x)\equiv C_r(x)$,因此$C_s(x)\equiv 1$,但这是不可能的.

同理若$C_s(a)=0$,我们也可以得出矛盾.


从而不存在正整数$q$, $r$和 $s$使 得 $C_q(x)\equiv C_r( x) C_s(x)$对任意复数$x$恒成立.

[详解](1)当$\theta=\frac\pi4$时, $z_n=\cos\frac\pi4 n+\mathrm{i}\sin\frac\pi4 n$,
则$z_1=\cos\frac\pi4+\mathrm{i}\sin\frac \pi 4= \frac {\sqrt 2}2(1+ \mathrm{i})$, $z_2= \cos \frac \pi 2+\mathrm{i}\sin\frac\pi2=\mathrm{i}$.


因为$z_{1}^{2}=\left[ \frac{\sqrt{2}}{2}\left( 1+\text{i} \right) \right] ^2=\frac{1}{2}\left( 1+2\text{i}+\text{i}^2 \right) =\text{i}=z_2$,
故“$2$维形态复数”与““$1$维形态复数”之间存在平方关系.

(2)因为“$2$维形态复数”与“$3$维形态复数”相等,
所以$\cos2\theta+\mathrm{i}\sin2\theta= \cos3\theta+\mathrm{i}\sin3\theta$,
因此$\begin{cases}
\cos2\theta=\cos3\theta,\\
\sin2\theta=\sin3\theta,
\end{cases}$
解$\cos2\theta=\cos3\theta$,得
$3\theta=2\theta+2k\pi\ (k\in\mathbf{Z})$ 或$3\theta+2\theta=2k\pi\ (k\in\mathbf{Z})$,

解$\sin2\theta=\sin 3\theta$,得
$3\theta=2\theta+2k\pi\ (k\in\mathbf{Z})$ 或$3\theta+2\theta=2k\pi+\pi\ (k\in\mathbf{Z})$,

由于两个方程同时成立,故只能有$3\theta=2\theta+2k\pi\ (k\in\mathbf{Z})$,即$\theta=2k\pi\ (k\in\mathbf{Z})$.

所以$\sin\left(\theta+\frac\pi4\right)
=\sin\left(2k\pi+\frac\pi4\right)
=\sin\frac\pi4=\frac{\sqrt2}2$.

(3) 由$z_m=z_1$,得$\cos m\theta+\mathrm{i}\sin m\theta=\cos\theta+\mathrm{i}\sin\theta$,
由(2)同理可得$m\theta=\theta+2k_1\pi\ (k_1\in\mathbf{Z})$,

即$\left(m-1\right)\theta=2k_1\pi\ (k_1\in\mathbf{Z})$.

因为$m>1$,所以$\theta=\frac{2k_1\pi}{m-1}\ \left(k_1\in\mathbf{Z}\right)$.

因为$z_n=z_m^2=z_1^2$,由(1)知$z_2=z_1^2$,所以$z_n=z_2$.

由(2)同理可得$n\theta=2\theta+2k_2\pi\ (k_2\in\mathbf{Z})$,即$(n-2)\theta=2k_2\pi\ (k_2\in\mathbf{Z})$.

因为$n>2$,所以$\theta=\frac{2k_2\pi}{n-2}\ \left(k_2\in\mathbf{Z}\right)$,

所以$\frac{2k_{1}\pi}{m-1}=\frac{2k_{2}\pi}{n-2}\ \left(k_{1},k_{2}\in\mathbf{Z}\right)$,

又因为$\theta \neq 0$,所以$k_1k_2\neq 0$,所以$\frac {m-1}{n-2}=\frac{k_1}{k_2}\ \left(k_1,k_2\in\mathbf{Z}\right)$,

即$m=\frac{k_{1}}{k_{2}}\left(n-2\right)+1=\frac{k_{1}}{k_{2}}\cdot n+1-\frac{2k_{1}}{k_{2}}\ \left(k_{1},k_{2}\in\mathbf{Z}\right)$,

所以存在有理数$q=\frac {k_1}{k_2}$,使得$m=q\cdot n+1-2q$.

(深圳中学2024届高三二轮四阶测试)若函数$y=f(x)$的图象上的两个不同点处的切线互相重合,则称该切线为函数$y=f(x)$的图象的“自公切线”,称这两点为函数$y=f(x)$的图象的一对“同切点”.


(1)分别判断函数$f_1(x)=\sin x$与$f_2(x)=\ln x$的图象是否存在“自公切线”,并说明理由;

(2)若$a\in\mathbf{R}$,求证:函数$g(x)=\tan x-x+a\ \left(-\frac{\pi}{2}<x<\frac{\pi}{2}\right)$有唯一零点且该函数的图象不存在“自公切线”;

(3)设$n\in\mathbf{N}_+$,函数$h(x)=\tan x-x+n\pi\ \left(-\frac{\pi}{2}<x<\frac{\pi}{2}\right)$的零点构成数列$\{x_n\}$.已知$t\in\left(-\frac\pi2,\frac\pi2\right)$,求证: “存在$s\in(2\pi,+\infty)$,使得点$(s,\sin s)$与$(t,\sin t)$是函数$y=\sin x$的图象的一对“同切点””的充要条件是“$t$是数列$\{x_n\}$中的项”.

19.[详解] (1)显然$f_1(x)=\sin x$的图象在点$\left(\frac\pi2,1\right)$, $\left(\frac{5\pi}2,1\right)$处的切线均为直线$y=1$,故直线$y=1$是$f_{1}(x)=\sin x$的图象的一条“自公切线”,因此函数$f_1(x)$的图象存在“自公切线”;


对于$f_2(x)=\ln x,f'_2(x)=\frac1x\ (x>0)$是单调递减函数,则$f_2(x)$的图像在不同点处的切线斜率必定不同,
所以函数$f_2(x)$的图象不存在“自公切线”.


(2)因为$g'(x)=\frac{1}{\cos^{2}x}-1=\frac{\sin^{2}x}{\cos^{2}x}
=\tan^{2}x\geqslant 0$恒成立,所以$y=g(x)$是$\left(-\frac\pi2,\frac\pi2\right)$上的单调递增函数,故它至多有一个零点.


$g(x)=\tan x-x+a=\frac{\sin x-x\cos x+a\cos x}{\cos x}$.令$g_1(x)=\sin x-x\cos x+a\cos x$,
因为$g_1\left(-\frac\pi2\right)=-1$, $g_1\left(\frac\pi2\right)=1$,所以$g_1\left(-\frac\pi2\right)g_1\left(\frac\pi2\right)<0$,由零点存在定理可知, $g_1(x)$在$\left(-\frac\pi2,\frac\pi2\right)$上存在
零点,而$g_1(x)=g(x)\cos x$,当$x\in\left(-\frac\pi2,\frac\pi2\right)$时$\cos x\neq 0$,故$g(x)$有唯一零点.

(注:也可以通过$\lim_x\to -{\frac{\pi} {2}}^{+}g(x)=-\infty,\lim_{x\to {\frac{\pi}{2}}^-}g(x)=+\infty$来说明$g(x)$的零点的唯一性)

设切点为$(x_1,\tan x_1-x_1+a)$,则$g(x)$的图像上的一条切线方程为$y=\tan^2x_1\left(x-x_1\right)+\tan x_1-x_1+a$.
假设$g(x)$的图象存在“自公切线”,则存在$x_1,x_2\in\left(-\frac\pi2,\frac\pi2\right)$且$x_1\neq x_2$,使得$g(x)$的图象在$x=x_1$与$x=x_2$处的切线重合,所以$\tan^2x_1=\tan^2x_2$,故有$x_2=-x_1$,不妨设$x_1\in\left(0,\frac\pi2\right)$,
同时$-x_1\tan^2x_1+\tan x_1-x_1+a=-x_2\tan^2x_2+\tan x_2-x_2+a$,即$x_1(1+\tan^2x_1)=\tan x_1\Leftrightarrow$
$\frac{x_{1}}{\cos^{2}x_{1}}=\tan x_{1}\Leftrightarrow x_{1}=\sin x_{1}\cos x_{1}$,即$2x_1=\sin x_{1}$.


令$\phi(x)=2x-2\sin x,x\in(0,\pi)$, $\phi'(x)=2-2\cos x>0$,故函数$\phi(x)$在$(0,\pi)$上单调递增, $\phi(x)>
\phi(0)=0$.因此当$x\in(0,\pi)$时, $2x>2\sin x$.即$2x_1=\sin x_1$在$\left(0,\frac\pi2\right)$上无解,
所以$g(x)$的图象不存在“自公切线”.


(3)函数$y=\sin x$在点$(t,\sin t)$处的切线方程为$y=\cos t\left(x-t\right)+\sin t=x\cos t+\sin t-t\cos t$,
函数$y=\sin x$在点$(s,\sin s)$处的切线方程为$y=x\cos s+\sin s-s\cos s$.


\ding{172} 先证明必要性:若存在$s\in (2\pi,+\infty)$,使得点$(s,\sin s)$与$(t,\sin t)$是函数$y=\sin x$的图象的一对“同切点”,
则$\begin{cases}
\cos t=\cos s\\
\sin t-t\cos t=\sin s-s\cos s
\end{cases}$且$s\neq t$,
所以存在$k\in \mathbf{N}_+$使得$s=t+2k\pi$或$s+t=2k\pi$.

 

当$s=t+2k\pi\ (k\in\mathbf{N}_{+})$时, $\sin t=\sin s$,故$t\cos t=s\cos s$,即$(s-t)\cos s=0$,因为$t\in\left(-\frac\pi2,\frac\pi2\right),\cos s>0$,
所以$s=t$,这不可能,舍去.


当$s+t=2k\pi\ (k\in\mathbf{N}_{+})$时, $\sin t=-\sin s,\cos t=\cos s\neq 0$,故$\sin t-t\cos t=-\sin t-(2k\pi-t)\cos t$,
即$2\sin t+(2k\pi-2t)\cos t=0\Leftrightarrow\tan t-t+k\pi=0$.


又由(2)知对于给定的$n\in\mathbb{N}_+$,函数$h(x)$有唯一零点,故$t=x_k$,即$t$是数列$\{x_n\}$中的项.


\ding{173} 再证明充分性:若$t$是数列$\{x_n\}$中的项,则存在$n\in\mathbf{N}_+$, $h(t)=\tan t-t+n\pi=0$,
由(2)可知$h(x)$在$\left(-\frac\pi2,\frac\pi2\right)$上单调递增,又$h(0)=n\pi>0$, 故$-\frac\pi2<t<0$.


令$s=2n\pi-t$,则$s\in(2\pi,+\infty)$且$\cos s=\cos t,\sin s=-\sin t$,则
$\sin s-s\cos s=-\sin t-(2n\pi-t)\cos t=(\sin t-t\cos t)-[2\sin t+(2n\pi-2t)\cos t]=(\sin t-
t\cos t)-2\cos t\left[\tan t+n\pi-t\right]=\sin t-t\cos t$,

此时点$(s,\sin s)$与$(t,\sin t)$是函数$y=\sin x$的图象的一对“同切点”.

综上: “存在$s\in(2\pi,+\infty)$,使得点$(s,\sin s)$与$(t,\sin t)$是函数$y=\sin x$的图象的一对“同切点” ”的充要条件是“$t$是数列$\{x_n\}$中的项”.


(清华附2023年高一下期中)对于角的集合$\{\theta_1,\theta_2,\cdots,\theta_n\}$和角$\alpha$,定义$\mu=\frac1n\left[\cos^2(\theta_1-\alpha)
+\cos^2(\theta_2-\alpha)+\cdots
+\cos^2(\theta_n-\alpha)\right]$为集合$\{\theta_1,\theta_2,\cdots,\theta_n\}$相对角$\alpha$的“余弦方差”.


(1)集合$A=\left\{\frac\pi2,\pi\right\}$
和$B=\left\{\frac\pi3,\frac{2\pi}3,\pi\right\}$相对角$\alpha$的“余弦方差”分别为多少?


(2)角$\alpha=\frac\pi2$, 集合$\theta=\left\{\frac\pi5,\frac{2\pi}5,
\frac{3\pi}5,\frac{4\pi}5,\pi\right\}$,求$\theta$相对角$\alpha$的“余弦方差”为多少?


(3)角$\alpha=\frac{\pi}{n}$,集合$\{\theta_1,\theta_2,\cdots,\theta_n\}$, $\theta_1+\theta_2+\cdots+\theta_n=2\pi,n\in\mathbf{N} ^\ast$,求$\theta$相对角$\alpha$的”余弦方差”是否有最大值?若
有,求出最大值,若没有,说明理由?


(浙江省五校联考2024届高考模拟)第二次世界大战期间,了解德军坦克的生产能力对盟军具有非常重要的战略意义.已知德军的每辆坦克上都有一个按生产顺序从$1$开始的连续编号. 假设德军某月生产的坦克总数为$N$,随机缴获该月生产的$n$辆$\left(n<N\right)$坦克的编号为$X_1,X_2,\cdots,X_n$,记$M=\max\{X_1,X_2,\cdots,X_n\}$,即缴获坦克中的最大编号.现考虑用概率统计的方法利用缴获的坦克编号信息估计总数$N$.


甲同学根据样本均值估计总体均值的思想,用 $\bar{X}=\frac{X_1+X_2+\cdots+X_n}n$估计总体的均值,因此
$N\bar{X}\approx\sum_{i=1}^{N}i
=\frac{N\left(N+1\right)}{2}$,
得$\bar{X}\approx\frac{N+1}{2}$,故可用$Y=2\bar{X}-1$作为$N$的估计.


乙同学对此提出异议,认为这种方法可能出现$Y<M$的无意义结果.例如,当$N=5,n=3$时,若
$X_{1}=1,X_{2}=2,X_{3}=4$,则$M=4$,此时$Y=2\cdot\frac{1+2+4}{3}-1=\frac{11}{3}<M$.

(1)当$N=5,n=3$时,求条件概率$P(Y<M\mid M=5)$;

(2)为了避免甲同学方法的缺点,乙同学提出直接用$M$作为$N$的估计值.当$N=8,n=4$时,求随机变量$M$的
分布列和均值$E(M)$;

(3)两同学认为估计值的均值应稳定于实际值,但直观上可以发现$E(M)$与$N$存在明确的大小关系,因此乙同学的方法也存在缺陷.请判断$E(M)$与$N$的大小关系,并给出证明.

 


(浙江省五校联考2024届高考模拟)卷积运算在图像处理、人工智能、通信系统等领域有广泛的应用.一般地,对无穷数列$\left\{a_n\right\}$, $\left\{b_n\right\}$,定义无穷数列$c_n=\sum_{k=1}^ {n}a_kb_{n+1-k}\ \left(n\in\mathbf{N}_{+}\right)$,记作$\left\{a_n\right\}\ast \left\{b_n\right\}=\left\{c_n\right\}$,称为$\left\{a_n\right\}$与$\left\{b_n\right\}$
的卷积.卷积运算有右图所示的直观含义,即$\{c_n\}$中的项依次为所列数阵从左上角开始各条对角线上元素的和,易知有交换律
$\{a_{n}\}\ast \{b_{n}\}=\{b_{n}\}\ast \{a_{n}\}$.


(1)若$a_n=n$, $b_n=2^n$, $\{a_n\}\ast \{b_n\}=\{c_n\}$,求$c_1,c_2,c_3,c_4$;

(2)对$i\in\mathbf{N}_+$,定义$T_i\{a_n\}$如下: \ding{172}当$i=1$时, $T_i\{a_n\}=\{a_n\}$; \ding{173}当$i\geqslant 2$时, $T_i\{a_n\}$为满足通项
$d_n=\begin{cases}
0,& n<i,\\
a_{n+1-i} & n\geqslant i,
\end{cases}$的数列$\{d_n\}$,
即将$\{ a_n\}$的每一项向后平移$i-1$项,前$i-1$项都取为$0$.

试找到数列$\left\{t_n^{(i)}\right\}$,使得$\left\{t_n^{(i)}\right\}\ast \left\{a_n\right\}=T_i\left\{a_n\right\}$;

(3)若$a_n=n,\{a_n\}\ast \{b_n\}=\{c_n\}$,证明:当$n\geqslant 3$时, $b_n=c_n-2c_{n-1}+c_{n-2}$.


(埃及分数)定义:一个正整数$n$称为“漂亮数”,当且仅当存在一个正整数数列
$a_{1}$, $a_{2}$, $\cdots$, $a_{k}$满足(i) (ii):

(i) $a_1<a_2<\cdots<a_{k-1}<a_k=n$;

(ii) $\frac1{a_1}+\frac1{a_2}+\cdots+\frac1{a_k}=1$.

试解答下列问题:

(1)最小的“漂亮数”是多少?证明你的结论;

(2)若$n$是“漂亮数”,则$n^{n-1}$是“漂亮数”吗?证明你的结论;

(3)当$k=4$时,任取一个“漂亮数”$n$,求$n-1$是质数的概率.


小平邦彦初三年级,阅读日本东北大学的讲义藤原松三郎著的《代数学1,2》,把不明白的证明反复抄写在笔记上背下来,直到弄懂为止。确有其事

(Fibonomial Calculus)定义斐波那契数列$\{F_n\}_{n\geqslant 0}$为
$$\begin{cases}
F_{n+2}=F_{n+1}+F_n, \\
F_0=0,\quad F_1=1.
\end{cases}$$
定义:

(i) $F-$阶乘($F-$ factorial)为
$$F_n!=F_nF_{n-1}\cdots F_2F_1,\quad F_0!=1.$$

(ii) $F-$多项式系数
$${\binom nk}_F=\frac{F_nF_{n-1}\cdots F_{n-k+1}}{F_kF_{k-1}\cdots F_2F_1}=\frac{F_n!}{F_k!F_{n-k}!},
\quad {\binom n0}_F=1.$$
则有:


(1) 对称性: ${\binom{n}{k}}_{F}={\binom{n}{n-k}}_{F}$;

(b) $F_{n-k}{\binom{n}{k}}_{F}=F_{n}{\binom{n-1}{k}}_{F}$;

(c) 对任意$n,k\in\mathbf{N}$,有${\binom{n}{k}}_{F}\in\mathbf{N}$.

(d)二项式定理:记$a=\frac{1-\sqrt{5}}{2},b=\frac{1+\sqrt{5}}{2},ab=-1$,则有
$$
\prod_{k=1}^n{\left( 1-a^{k-1}b^{n-k}x \right)}=\sum_{k=0}^n{
\left( -1 \right) ^{\frac{\left( k+1 \right) k}{2}}
{\binom{n}{k}}_Fx^k}.
$$

西安交通大学强基计划往年真题及解析,附2024西交大强基培训简章
%https://mp.weixin.qq.com/s?__biz=MzkxMDY4NTYyNQ==&mid=2247484090&idx=1&sn=661c0f96bd9542debe2bf6201559a0b6&chksm=c126e66cf6516f7a2fd23e1bf6cc0f261a78a9baeb15a92c19798d90630deff772cb950219ec&mpshare=1&scene=1&srcid=0517anFR0XZWcEd3gSmYS3Qq&sharer_shareinfo=bb8d01ff5a54f666673e63df85e46995&sharer_shareinfo_first=bb8d01ff5a54f666673e63df85e46995#rd

西安交通大学2023年强基计划测试数学笔试试题

1.设集合$X$有$m$个元素,集合$Y$有$n$个元素,求集合$X\times Y$有多少子集,并说明理由.

2. (1)证明$(A\cap B)\times (C\cap D)=(A\times C)\cap(B\times D)$.

(2) $(A\cup B)\times (C\cup D)=(A\times C)\cup(B\times D)$一定成立吗?并说明理由.

 

3.设$R=\{(x,y)\mid x+y=6,x,y\text{为正整数}\}$,
$S=\{(x,y)\mid xy+ x= 30, x, y\text{为正整数}\}$.

(1) $A=\{1,2,3,4,5\}$,求$S^{-1}(A)$.


(2)求$R^{-1}$与$S\circ R$.

4.设$X,Y,Z,W$为四个集合, $R$为$X$到$Y$的一种关系, $S$ 为$Y$到$Z$的一种关系, $T$为$Z$到$W$的一种关系.


证明: (1) $\left(R^{-1}\right)^{-1}=R$;

(2) $(S\circ R)^{-1}=R^{-1}\circ S^{-1}$;

(3) $T\circ(S\circ R)=(T\circ S)\circ R$.

5. $\Omega=\{R\in X\times X\mid X \text{为非空数集且对任意}S\in X\times X \text{均有}R\circ S=S\circ R\}$.
证明: $\Omega=\{\varnothing,\Delta\}$,其中$\Delta=\{(x,x)\mid x\in X\}$.

 

%【参考文献】Krot E .An Introduction to Finite Fibonomial Calculus [J].Central European Journal of Mathematics, 2004.


Fibonacci polynomials

%https://encyclopediaofmath.org/wiki/Fibonacci_polynomials

\section{2020年南京大学强基计划数学试题}

1.已知数列$\left\{F_n\right\}$满足:
\ding{172} $F_0=0,F_1=1$; \ding{173} $F_ {n+1}=F_n+F_{n-1}$\ $\left(\forall n\in\mathbf{N}_+\right)$.

(1)证明: $F_n=F_{n-k}\cdot F_{k+1}+F_{n-k-1}\cdot F_k$ 对$\forall n,k\in\mathbf{N}$且$k\leqslant n-1$成立;

(2)证明: $F_{m+n}>F_m\cdot F_n$,其中$m,n\in\mathbf{N}_+$且$m+n>2$;


(3)证明: $F_{mn}>F_m^n$,其中$m,n\geqslant 2$.


2.已知 $A,B,C$为$\triangle ABC$的三个内角,求$\sin^2A+\cos A\sin B\sin C$的最大值并给出取等条件.


3.已知正三棱锥$P-ABC$中,底面边长为$1$,侧棱长是$a$ (定值),过点$A$的动平面与$PB,PC$分别交于$E,F$.若当$\triangle AEF$周 长最小时, $\triangle AEF$的面积是$\frac{\sqrt{7}}{8}$,求$a$的值.

 

4.已知$f\left(x\right)$是实系数四次多项式,且对$\forall x\in\mathbf{R}$, $f\left(x^2\right)=f\left(x\right)\cdot f\left(x-1\right)$,求$f\left(2\right)$.


\section{2021年南京大学强基计划数学试题及解析}

 

南京大学强基计划理科测试,考试时间 90 分钟,共 6道大题: 3道数学, 3道物理,回忆版只有 2 道数学题.

1.求$M=\left[\sqrt[3]{1}\right]+\left[\sqrt[3]{2}\right]
+\left[\sqrt[3]{3}\right]+\cdots
+\left[\sqrt[3]{2021}\right]$.

解析:当$k^3\leqslant n<\left(k+1\right)^{3}$,有$\left[ \sqrt[3]{n}\right]=k$, 又$12^{3}< 2021< 13^{3}$,
所以$M=\sum_{k=1}^{11}k\left [ \left ( k+ 1\right ) ^3- k^3\right ] + 12\cdot \left ( 2021- 12^3+ 1\right ) = 18180$.


2.已知$0\leqslant a+b,b+c,c+a\leqslant 1$,求$M=\sqrt{\left|a-b\right|}+\sqrt{\left|b-c\right|}
+\sqrt{\left|c-a\right|}$的最值.

解析:易知$M_{\min}=0$,当且仅当$a=b=c\in\left[0,\frac12\right]$取等号.

记$x=a+b,y=b+c,z=c+a$,则有 $x,y,z\in\left[0,1\right]$,不妨设$x\geqslant y\geqslant z$,则有 $M=\sqrt{x-y}+\sqrt{y-z}+\sqrt{x-z}\leqslant
\left(\sqrt{2}+1\right)\sqrt{x-z}\leqslant\sqrt{2}+1$,
当$x=1,y=\frac{1}{2},z=0$,即 $a=\frac{1}{4},b=\frac{3}{4},c=\frac{1}{4}$时取等号,所以$M_{\max}=\sqrt{2}+1$.

\section{2022年南京大学强基测试数学试题(初试)}

 

\begin{center}
2022年6月11日,\quad 14:00-15:30
\end{center}

 

1.函数$y=\sqrt{x-4}+\sqrt{15-3x}$的值域为\underline{\hspace{2cm}}.

2.设$x\in(0,\pi/2)$,则函数$y=\sin^2x\cos x$的最大值为\underline{\hspace{2cm}}.

3. 已知$x,y,z$满足$x+y+z=1$,则$x^2+4y^2+9z^2$的最小值为\underline{\hspace{2cm}}.


4.在$\triangle ABC$中,角$A$、$B$、$C$的对边分别为$a$、$b$、$c$,已知
$$a\cos C-b\cos^2A=a\sin A\sin B-c\sin A,$$
则$\tan A$的值为\underline{\hspace{2cm}}.

 

5.已知实数$x,a_1,a_2,y$成等差数列, $x,b_1,b_2,y$成等比数列,则$\frac{(a_1+a_2)^2}{b_1b_2}$的取值范围为\underline{\hspace{2cm}}.


6.已知向量$\vec{a},\vec{b},\vec{c}$满足$\begin{vmatrix} \vec{a} \end{vmatrix} = 3$, $\begin{vmatrix} \vec{b} \end{vmatrix} = 2\sqrt {2}$, $\vec{a} \cdot \vec{b} = 6$, 且$\begin{pmatrix} \vec{a} + \vec{c} \end{pmatrix} \begin{pmatrix} \vec{b} + 2\vec{c} \end{pmatrix} = 0$, 则
$\left|\vec{b}+\vec{c}\right|$最小值为\underline{\hspace{2cm}}.

 

7.已知直线$y=ax+2$与三次曲线$y=x^3-ax$有三个不同交点,则$a$的取值范围为\underline{\hspace{2cm}}.


8.在棱长为$6$的正四面体$ABCD$中$,M$为面$BCD$上一点,且$|AM|=5$,设异面直线$AM$与$BC$所成的角为$\alpha$,则$\left|\cos\alpha\right|$的最大值为\underline{\hspace{2cm}}.

 

9.方程$x_1+x_2+x_3+3x_4+3x_5+5x_6=7$的非负整数解的个数为\underline{\hspace{2cm}}.


10. 设$F,l$分 别 为 双 曲 线 $\frac {\left(x-4\right) ^2}{12}-\frac{y^2}{12}=1$的右焦点与右准线,椭圆$\Gamma$以$F$和$l$为其对应的焦点及准线,过$F$作一条平行于$y=\sqrt3x$的直线,交椭圆$\Gamma$于$A$、$B$两点,若$\Gamma$的中心位于以$AB$为直径的圆外,则椭圆离心率$e$ 的范围为\underline{\hspace{2cm}}.

 

\section{2022年南京大学强基测试数学试题(复试)}

 

\begin{center}
2022年6月18日, \quad 10:00-11:30
\end{center}

1.已知整数$n>1$,证明: $\left(\frac{n+1} {3}\right)^n<n!<\left(\frac{n+1}{2}\right)^n$.

 

2. 设$\alpha,\beta\in(0,\pi)$, 且$\cos\alpha+\cos\beta-\cos(\alpha+\beta)=\frac32$,
求$\alpha,\beta$的值.

 

3.设$x^2-6x+1=0$的两个根分别为$x_1,x_2$, 设$a_n=\frac{x_1^n+x_2^n}2$.

(1)求证: $a_n\in\mathbb{Z}$;

(2)求$a_{2022}$的个位数字.

\section{2023年南京大学强基测试数学试题}


1.平分圆$x^2-2x+y^2-4y=0$的直线不经过第四象限,则斜率的范围是\underline{\hspace{2cm}}.

2.已知$x,y\in [0,1]$,则$x^2+y^2\leqslant 1$且$(x-1)^2+(y-1)^2\leqslant 1$的概率是\underline{\hspace{2cm}}.

 


3.在$1,2,3,4,5$中,有放回地取数字三次,则最小的数字为$2$的概率是\underline{\hspace{2cm}}.

 

4.已知$\frac{\sin ^4\alpha }{\sin ^2\beta }+ \frac {\cos ^4\alpha }{\cos ^2\beta }=1$ , 则$\frac{\sin^4\beta}{\sin^2\alpha}+\frac{\cos^4\beta}{\cos^2\alpha}=$
\underline{\hspace{2cm}}.

 

5.满足不定方程$\frac{1}{x}+\frac{1}{y}=\frac{1}{30}$且$x\leqslant y$的正整数解的组数是\underline{\hspace{2cm}}.

 

6.已知实系数二次方程$ax^2+bx+c=0$的两根为$\alpha$和$\beta$,且满足$\alpha$是虚数, $\frac{\alpha^2}{\beta}$是实数,
则$\frac{\alpha}{\beta}=$\underline{\hspace{2cm}}.

7.已知$a>0,b>0,x+y=c$,则$\sqrt{x^2+a^2}+\sqrt{y^2+b^2}$ 的最小值为\underline{\hspace{2cm}}.

 

8.已知$f(n)=\sum_{k=1}^{n}\left(k^{2}+3k+2,n\right)$,
则$f(100)=$\underline{\hspace{2cm}}.

 

9.已知$\triangle ABC$的三边为$a,b,c$,满足$a^2+b^2+c^2=\alpha,a^2b^2+b^2c^2+c^2a^2=\beta$,则
$\triangle ABC$的面积为\underline{\hspace{2cm}}.

 

10. 记$a_n$表示$n$在三进制下的各数字之和,例如$a_7=3,a_9=1$,记
$$S=\{a_n\mid a_n=5,1\leqslant n\leqslant 2023\}$$
则$\left|S\right|=$\underline{\hspace{2cm}}.


西湖大学2022年本科招生考试数学试题

1.若$19=a_0+a_1\cdot \frac{3}{2}+a_2\cdot \left( \frac{3}{2} \right) ^2+\cdots +a_n\cdot \left( \frac{3}{2} \right) ^n$,其中系数$\{a_i\}$的取值范围是$\{0,1,2\}$,求$a_{0}+ a_{1}+ a_{2}+\cdots+ a_{n}$的值.

 


2.若复数$z$满足$z^3=i$,求所有满足该式的$z$的虚部的乘积.

 

3.二次曲线$C_1:x^2=y+2,C_2:\frac{x^2}4+y^2=1$在同一平面上交于四点,易知该四点共圆,求该圆的半径.

令$$
\frac{3}{4}\left( x^2-y-2 \right) +\left( \frac{x^2}{4}+y^2-1 \right) =0,
$$
故$x^2+y^2-\frac{3}{4}y=\frac{5}{2}$,整理得$x^2+\left( y-\frac{3}{8} \right) ^2=\frac{169}{64}$.


4.当参数$a$变化时,求函数$f(x)=\frac{|x|}a+\frac a2-\sin x$的零点个数的最大值.

 

5.若数集$A,B$满足$A\cap B=\varnothing$, $A\cup B=I$,其中$I$亦是数集,判断下列说法的正误:

A.若$B=\{x+y\mid x,y\in A\}$,则存在$A,B$使得$I=\mathbb{R}$


B.若$B=\{x+y\mid x,y\in A\}$,则存在$A,B$使得$I=\mathbb{N}_+$

C.若 $B=\{xy\mid x,y\in A\}$,则存在 $A,B$使得$I=\mathbb{R}$

D.若$B=\{xy\mid x,y\in A\}$,则存在$A,B$使得$I=\mathbb{N}_+$

 

6.给定一个标准的半球,其高度为$R$.若在$\frac R2$的高度平行于底面将该半球切成两个部分,求两个部分的体积比.

 

7.用一个平面切立方体,求所有的截面可能的形状.

8.空间中两条异面线段$AB,CD$满足$AB=2,CD=1$,且它们呈 $45^\circ$角,求三棱锥$A-BCD$的体积.


棱长为$1$的正方体被平面截得的截面可能为

锐角三角形面积最大值为$\frac{\sqrt{3}}{2}$

六边形面积的最大值为$\frac{3\sqrt{3}}{4}$

A.面积为

【问题串】

1.如果截面是三角形,你认为可以截出几类不同的三角形(按角,边分类)?为什么?

2.如果截面是四边形,你认为可以截出几类不同的四边形?为什么?

3.还能截出哪些多变形?为什么?

4.能否截出直角三角形?为什么?

5.能否截出正五边形?为什么?

6.有没有边数超过6的截面?为什么?

7.是否存在正六边形的截面?为什么?

8.最大面积的三角形截面是哪个?为什么?

9.你还能发现什么?


\section{2015年清华大学数学直博考试真题}

 

1.求极限: $\lim_{n\to\infty}\sum_{k=0}^n\left(1-\frac kn\right)^n$.


2.设 $A\in M_{n\times n}(\mathbb{C})$可逆, $A^m$为对角矩阵, $m\in\mathbb{N}_+$.求证: $A$可对角化.


3.设 $\varphi:\mathbb{R}[x]\to\mathbb{R}[x]$, $f\mapsto f+f'$,求证: $\varphi$可逆.


4.设$G$为$\mathbb{R}^n$中的有界开集, $D$为$G$的紧致子集, $F\in C^1(G,\mathbb{R}^n)$, $F|_D$为单射,并且
$\det F'(x)\neq 0,\forall x\in D$.求证:

(a) $d(D,\partial G)>0$.


(b)存在$0<\varepsilon<d(D,\partial G)$,使得$F|_{D_\varepsilon}$为单射,其中$D_\varepsilon=\{x\in\mathbb{R}^n:d(x,D)<\varepsilon\}$.

5.设$f:B(0,1)\to B(0,1)$为全纯函数, $f(\alpha)=0,\alpha\in B(0,1)$.求证:
$|f(z)|\leqslant\left|\frac{z-\alpha}
{1-\overline{\alpha}z}\right|,\quad \forall z\in B(0,1)$.


\section{2016年清华大学数学直博考试真题(1)}

1. (a)设$D$为$\mathbb{R}^n$中的区域, $f\in C^1(D,\mathbb{R}^n)$.试叙述关于$f$的逆射定理(包括条件和结论).


(b)试应用逆射定理证明不存在$\mathbb{R}^n$到$\mathbb{R}$中的连续可微单射.


2.设
$$\overrightarrow{F}(x,y,z)=\left(\frac{x}
{\left(x^2+2y^2+3z^2\right)^{\frac{3}{2}}},
\frac{y}{\left(x^2+2y^2+3z^2\right)^{\frac{3}{2}}},
\frac{z}{\left(x^2+2y^2+3z^2\right)^{\frac{3}{2}}}\right),$$
其中$(x,y,z)\in\mathbb{R}^3\backslash\{(0,0,0)\}$, $\overrightarrow{n}$为$\mathbb{R}^3$中单位球面$\mathbb{S}^{2}$的单位外法向量场.试求曲面积分:
$$\int_{\mathbb{S}^2}\overrightarrow{F}\cdot
\overrightarrow{n}\mathrm{d}\sigma.$$


3.设$f$为$\mathbb{R}$上周期为$2\pi$的函数, $f(x)=x,\forall x\in (-\pi,\pi]$.


(a)试求$f$的Fourier级数及Fourier级数的和函数,并说明该级数是否在$\mathbb{R}$上一致收敛.


(b)试利用上述Fourier级数及Parseval等式求出级数$\sum_{n=1}^\infty\frac1{n^2}$的值.


4.设$f:B(0,1)\to B(0,1)$为全纯函数, $f(\alpha)=0,\alpha\in B(0,1)$.

(a)求证:
$$|f(z)|\leqslant\left|\frac{z-\alpha}
{1-\overline{\alpha}z}\right|,\quad \forall z\in B(0,1).$$

(b)求上述不等式中等号成立的充要条件.

5.设$A\in M_{n\times n}(\mathbb{C})$, $f_A(x)$为$A$的特征多项式, $g(x)\in\mathbb{C}[x]$, $\deg g(x)=n-1$, $g(x)\mid f(x)$.求$g(A)$所有可能的秩,并说明理由.


6.设$V$为$\mathbb{C}$上$n$维线性空间, $\varphi$为$V$上的幂幺线性变换,即存在 $k\in\mathbb{N}_+$使得$\varphi^k=1_V$,又设$W$为$\varphi-$不变子空间.求证:存在 $\varphi-$不变子空间$W'$,使得 $V=W\oplus W'$.

 

\section{2016年清华大学数学直博考试真题(2)}


1.设定义在$D\subset\mathbb{R}$上的函数$f$在$x_0\in D$处解析,即存在$\delta>0$,使得$f$在$(x_0-\delta,x_0+\delta)$
内可展开为$x-x_0$的幂级数.


(a)求证: $f$在$\left(x_0-\delta,x_0+\delta\right)$内任意点处解析.


(b)若$f$在$(x_0-\delta,x_0+\delta)$内不恒等于零,求证: $f$在$(x_0-\delta,x_0+\delta)$内的零点是孤立的,即
若$x_1\in (x_0-\delta,x_0+\delta)$为$f$的零点,则存在$\varepsilon>0$,使得$(x_1-\varepsilon,x_1+\varepsilon)\subset (x_0-\delta,x_0+\delta)$,并且$f$在$(x_1-\varepsilon,x_1+\varepsilon)$内有且仅有$x_1$一个零点.


2.求由椭球面$\frac{x^2}2+\frac{y^2}6+\frac{z^2}{27}=1$在第一象限的切平面与三个坐标平面所围成的四面体的最小体积.

 

3.设
$$\overrightarrow{F}(x,y,z)=\left(\frac{x}
{\left(x^2+2y^2+3z^2\right)^{\frac{3}{2}}},
\frac{y}{\left(x^2+2y^2+3z^2\right)^{\frac{3}{2}}},
\frac{z}{\left(x^2+2y^2+3z^2\right)^{\frac{3}{2}}}\right),$$
其中$(x,y,z)\in\mathbb{R}^3\backslash\{(0,0,0)\}$, $\overrightarrow{n}$为$\mathbb{R}^3$中单位球面$\mathbb{S}^2$的单位外法向量场.试求曲面积分:
$$\int_{\mathbb{S}^{2}}\overrightarrow{F}\cdot
\overrightarrow{n}\mathrm{d}\sigma.$$

 

4.设 $D$为$\mathbb{R}^n$中的区域, $K$为$D$的紧致子集, $f\in C^1(D,\mathbb{R}^n)$, $f|_K$为单射,并且$\det F'(x)\neq 0$, $\forall x\in D$.求证:存在开集$U,V$,使得 $K\subset U\subset D,f(K)\subset V\subset\mathbb{R}^n$, $f:U\to V$为$C^1$微分同胚.


5.设 $A,B\in M_{n\times n}$ (数域$\mathbb{F}$), $AB-BA=aB,a\in\mathbb{F}$.求证:若$B$不是幂零矩阵,则 $a=0$.

 

6.设$X_1=(1,-2,1)^{\mathrm{T}}$和$X_2=(-1,a,1) ^{\mathrm{T}}\ (a\in\mathbb{R})$分别是$3$阶不可逆实对称矩阵$A$的属于特征值$1$和$-1$的特征向量,试求$A$.

 

7.设$V$为有限维线性空间, $\varphi$为$V$上可对角化的线性变换, $U$为$\varphi-$不变子空间.求证: $\varphi|_U$也可对角化.


\section{2017年清华大学直博生招生考试试题(厦大版)}


数学分析试题


1.设$\{a_n:n\geqslant 1\}$是一个实数列,且满足:
$$a_{n+m}\leqslant a_n+a_m,\quad\forall n,m\in\mathbb{N}.$$


(1)证明:
$$\lim_{n\to\infty}\frac{a_n}n=\inf_{n\geqslant 1}\frac{a_n}n.$$


(2)举例说明该极限可以是$-\infty$.

(3)若序列$\{a_n:n\geqslant 1\}$还满足
$$a_{n+m}\geqslant a_n+a_m-c,\quad\forall n,m\in\mathbb{N}.$$
这里$c>0$是一个常数,证明(1)中的极限必是实数.

2.设$\{a_n:n\geqslant 1\}$是一个实数列,且满足$a_n\geqslant\gamma>2$.

(1)定义
$$\alpha_{n}:=\cfrac{1}{a_{1}-\cfrac{1}{a_{2}
-\cfrac{1}{\cdots-\cfrac{1}{a_{n}}}}}$$
证明$\alpha:=\lim_{n\to\infty}\alpha_n$存在.我们记$\alpha=[a_1,a_2,\cdots]$.

(2)计算$\alpha_\gamma:=[\gamma,\gamma,\cdots]$.

(3)证明$\alpha\in(0,\alpha_\gamma)$.

高等代数试题

1.在复数域$\mathbb{C}$上的线性空间$M_n(\mathbb{C})$内定义一个线性变换$\sigma$如下:
$$\sigma\begin{bmatrix}
a_{11}&a_{11}&\cdots&a_{1,n-1}&a_{1n}\\
a_{21}&a_{22}&\cdots&a_{2,n-1}&a_{2n}\\
\cdots&\cdots&\cdots&\cdots&\cdots\\
a_{n1}&a_{n2}&\cdots&a_{n,n-1}&a_{nn}
\end{bmatrix}
=\begin{bmatrix}
a_{12}&a_{22}&\cdots&a_{1n}&a_{11}\\
a_{22}&a_{23}&\cdots&a_{2n}&a_{21}\\
\cdots&\cdots&\cdots&\cdots&\cdots\\
a_{n2}&a_{n3}&\cdots&a_{nn}&a_{n1}
\end{bmatrix}.$$

(1)求$\sigma$的特征多项式;

(2)证明$\sigma$的矩阵可以相似对角化.

 

2.设$\sigma$为$n$维欧几里德空间$V$的一个线性变换.若任取$V$上的两个向量$\alpha,\beta$总有$\alpha$与$\beta$的
夹角等于$\sigma\alpha$与$\sigma\beta$的夹角,则称$\sigma$为保角变换,我们规定零向量与任何向量正交.证明: $\sigma$为$V$的保角变换当且仅当$\sigma$为一个非零的实数与$V$的一个正交变换的乘积.

 

清华大学数学直博生考试概要

2018年4月12日晚

 

1. $g:\mathbb{R}^{n}\to \mathbb{R}^{n}$一阶连续可微,且Jaccobi矩阵范数有界,即存在$M>0$,使得
$\left( \sum_{i,j=1}^n{\left( \frac{\partial g_i}{\partial x_j} \right) ^2} \right) ^{\frac{1}{2}}<M$.求证:对$f\left ( x\right ) = x+\varepsilon g(x)$,存在充分小的$\varepsilon>0$,使得$f$是可逆映射,且$f,f^{-1}\in C^1( R^n)$.

 

2.方程$x^{5}-x+1=0$的实根,虚根各有多少个?

 

3.设$S$为区域$0\leqslant x,y,z\leqslant 1$的表面,其单位法向量从区域内指向区域外部
求曲面积分$\iint_{S}\left(5x,-2y,-4z\right)\cdot d\overrightarrow{S}$.

4.记曲线$C_a\ (a>0)$为$r_{a}:[0,\pi]\to \mathbb{C},r_{a}(t)=ae^{it}$的参数化曲线(半圆)
求$\lim_{a\to\infty}\int_{C_a}\frac{e^{iz}}{z}\mathrm{d}z.$

5.设$4$阶复方阵$B$满足
$B^n=A=\left( \begin{matrix}
0& 0& 0& 1\\
0& 0& 0& 0\\
0& 0& 0& 0\\
0& 0& 0& 0\\
\end{matrix} \right)$,
求$n$可能是哪些正整数,并举出对应的一个$B$.

 

6.线性空间$V$有线性映射$\sigma,\tau$,使得$\sigma^{2}=\tau^{2}=0$, $\sigma\tau+\tau\sigma=\mathrm{id}_{V}$ (恒同).
求证:

(1) $V=\mathrm{ker}\sigma\oplus \mathrm{ker}\tau$;

(2) $V$的维数为偶数;

(3)求作$V$的一组基,使得$\sigma$和$\tau$在基下矩阵表示为
$\left( \begin{matrix}
0& 0\\
I_n& 0\\
\end{matrix} \right),\left( \begin{matrix}
0& I_n\\
0& 0\\
\end{matrix} \right)$,其中$n=\frac{\dim V}{2}$.

 

7. $f(z)$在圆盘$|z|<1$上解析,且$|f(z)|<1,f(a)=0,|a|<1$.

(1)在$|z|<1$时,求证:
$$
\left| f\left( z \right) \right|\leqslant \left| \frac{z-a}{1-\overline{a}z} \right|.
$$


(2)讨论(1)中取等的充要条件.


\section{清华大学2021年数学直博考试试题}

 

1.三阶实对称矩阵普通计算题.


2.设$\phi$是线性空间$V$上的线性变换,求证:
$\ker\phi=\ker\left(\phi^2\right)
\Leftrightarrow\operatorname{im}\phi
=\operatorname{im}\left(\phi^2\right).$


3.设$A,B$是数域$\mathbb{F}$上的$n$阶方阵, $A,B$都可对角化, $AB=BA$.求证: $A,B$可同时对角化.并举例说明$AB\neq BA$时结论不一定成立.

4.设$\Omega$是$\mathbb{R}^n$的开集, $f\in C^1(\Omega;\mathbb{R}^m)$.求证:

(1) $n>m$时, $f$不可能是单射;

(2) $n<m$时, $f$不可能是满射.

5.设$f,\phi$是周期为$2$的偶函数,并且
$$f(x)=x(2-x),\quad \phi(x)=x,x\in[0,1].$$

(1)求$f,\phi$的Fourier级数;

(2)求证: $f(x)=\sum_{n=0}^ {\infty}\frac1{2^{2n}}\phi(2^nx),x\in\mathbb{R}$.题目有问题,毕竟在$x\in [0,1]$时就不成立.

 

6.设$\overrightarrow{B}(x,y,z)
=\frac{(x,y,z)}{(x^2+y^2+z^2)^{\frac32}},
(x,y,z)\in\mathbb{R}^3\backslash\{(0,0,0)\}$.

(1)求$\mathrm{div} \overrightarrow{B}$;

(2)求证:不存在$\overrightarrow{F}\in C^1\left(\mathbb{R}^3\backslash\{(0,0,0)\};
\mathbb{R}^3\right)$,使得$\mathrm{rot} \overrightarrow{F}=\overrightarrow{B}$.

7.设$\alpha>1$, $I(\alpha)=-\int_0^{\infty}\frac{\ln x}{1+x^\alpha}\mathrm{d}x$.

(1)求$I(\alpha)$;

(2)求证: $I(\alpha)=\sum_{n\in\mathbb{Z}}\frac{(-1)^n}
{(n\alpha-1)^2}$.


复旦大学谢启鸿高等代数每周一题[2022S13]参考解答

%https://www.bilibili.com/read/cv18148110/
%https://www.bilibili.com/read/cv15680975/?spm_id_from=333.999.0.0


已知函数$f(x)=ae^x-x$有$2$个零点$x_1,x_2\ (x_1<x_2)$,函数$g(x)=ae^x+bx+2\ln \frac{x}{a}$,若$g(x_1)=0$,则

A. $b=-2$


B. $x_1<1<x_2$

C. $g(x)$的零点个数可能为$2$

D. 若$g(x)$存在大于$x_2$的零点,则$a\in \left( \frac{2}{e^2},\frac{1}{e} \right)$

首页
知乎知学堂
发现
等你来答

提问

消息

私信
点击打开undefined的主页
数学
清华大学
数学竞赛
丘成桐(数学家)
强基计划
如何评价5月26日的清华大学2024年“丘成桐领军计划数学水平考试”(原TACA)?
2024年4月28日举行。 清华大学2024年 “丘成桐领军计划数学水平考试” 报名通知 (qq.com)显示全部 ​

关注者
27
被浏览
28,974
关注问题​写回答
​邀请回答
​好问题 1
​添加评论
​分享

查看全部 12 个回答
Fiddie
Fiddie​
数学话题下的优秀答主
这次的题目前面与前几年相比略微简单,然而后面的题依然不简单.

转载请注明来源.
2024年5月丘成桐数学科学领军人才培养计划数学水平考试
考试时间:2024年05月26日

以下记 \lfloor x\rfloor 表示不超过 x 的最大整数,\{x\}=x-\lfloor x\rfloor.

1. 令 S=\displaystyle\int_0^{2024}\min\left(\left\{\dfrac{x}{8}\right\},\left\{\dfrac{x}{4}\right\}\right)\mathrm{d}x,则\lfloor S\rfloor= ____________.
【Fiddie解答】 被积函数f(x)=\min\left(\left\{\dfrac{x}{8}\right\},\left\{\dfrac{x}{4}\right\}\right)是周期为8的函数,画出f(x)的图像,知

f(x)=\left\{\begin{aligned} &\dfrac{x}{8}, &&x\in[0,4), \\ &\dfrac{x}{4}-1, &&x\in[4,8). \end{aligned}\right.\\

y=f(x)在x\in[0,8]上的面积为\dfrac{1}{2}\times 4\times\dfrac{1}{2}+\dfrac{1}{2}\times 4\times 1=3,所以

S=253\times 3=759, \qquad \lfloor S\rfloor=\boxed{759}. \\

2. 已知圆锥面x^2+y^2=\dfrac{z^2}{3},记沿该圆锥面从P(-\sqrt{3},3,6)到Q(\sqrt{3},0,3)的曲线长度的最小值为 I,则\lfloor 10I\rfloor= ____________.
【Fiddie解答】 我们考虑圆锥在z=0到z=6之间的部分,画出草图如图所示.为了求曲线 PQ 长度的最小值,只需把这个圆锥展开.圆锥与平面z=6的相交曲线是 x^2+y^2=12.所以圆锥的底面半径是2\sqrt{3},高为6,则母线为4\sqrt{3}.又因为圆锥底面周长是4\sqrt{3}\pi,所以圆锥的展开图是圆心角为\alpha=180^{\circ}的扇形.

我们不妨假设展开图从点 P 切开.如图,作 QQ'\bot 圆锥底面,则可以求得 \angle POQ'=120^{\circ},恰好为 \dfrac{1}{3} 的周角.于是可以确定 Q 在展开图中的位置是半径为 2\sqrt{3} 的半圆上,且使得展开图中 \angle POQ=\dfrac{\alpha}{3}=60^{\circ}.又因为 OP=4\sqrt{3},OQ=2\sqrt{3},所以 \angle PQO 是直角,于是 PQ 的长度为 OP\sin 60^{\circ}=6.因此 I=6,

\lfloor 10I\rfloor = \boxed{60}. \\


3. 令A,B,C,D,E,F是三阶实方阵,且
\begin{pmatrix} I_3 & A & C \\ & I_3 & B \\ & & I_3 \end{pmatrix}^{-1}=\begin{pmatrix} I_3 & D & F \\ & I_3 & E \\ & & I_3 \end{pmatrix}\\ 已知A=\begin{pmatrix} 2 \\ & 3 \\ & & 4 \end{pmatrix},B=\begin{pmatrix} 1& & 2 \\ & 2 \\ 5& &1 \end{pmatrix},且C=A+B-I,则\lfloor |\det F|\rfloor=____________.
【Fiddie解答】 利用初等变换,可以知道

\begin{pmatrix} I_3 & A & C \\ & I_3 & B \\ & & I_3 \end{pmatrix}^{-1}=\begin{pmatrix} I_3 & -A & (A-I)(B-I) \\ & I_3 & -B \\ & & I_3 \end{pmatrix}.\\

所以F=(A-I)(B-I),则\det F=\det(A-I)\det(B-I)=6\times 10=60.因此,\lfloor |\det F|\rfloor = \boxed{60}.

4. 令\displaystyle S=\int_1^{\infty}\dfrac{(\ln x)^5}{x^2}\mathrm{d}x,则\lfloor S\rfloor= ___________.
【Fiddie解答】 设x=\mathrm{e}^t,则

\begin{aligned} S&=\int_0^{\infty}\dfrac{t^5}{\mathrm{e}^{2t}}\mathrm{e}^t\mathrm{d}t \\ &=\int_0^{\infty}t^5\mathrm{e}^{-t}\mathrm{d}t \\ &=\Gamma(6)=5!=120. \end{aligned}\\

所以

\lfloor S\rfloor = \boxed{120}. \\

5. 令\displaystyle S=\sum\limits_{m=1}^{\infty}\sum\limits_{n=1}^{\infty}\dfrac{m^2n}{2^m(n\cdot 2^m+m\cdot 2^n)},则\lfloor 100S\rfloor= _____________.
【Fiddie解答】 首先这个级数是绝对收敛的(只需分母放掉n\cdot 2^m即可).

交换 m,n,得

\begin{aligned} S&=\sum\limits_{m=1}^{\infty}\sum\limits_{n=1}^{\infty}\dfrac{m^2n}{2^m(n\cdot 2^m+m\cdot 2^n)} \\ S&=\sum\limits_{m=1}^{\infty}\sum\limits_{n=1}^{\infty}\dfrac{n^2m}{2^n(m\cdot 2^n+n\cdot 2^m)} \\ \end{aligned}\\

两式相加,通分得

\begin{aligned} 2S&=\sum\limits_{m=1}^{\infty}\sum\limits_{n=1}^{\infty} \dfrac{m^2n\cdot 2^n+n^2m\cdot 2^m}{2^{m+n}(n\cdot 2^m+m\cdot 2^n)} \\ &=\sum\limits_{m=1}^{\infty}\sum\limits_{n=1}^{\infty} \dfrac{mn(m\cdot 2^n+n\cdot 2^m)}{2^{m+n}(n\cdot 2^m+m\cdot 2^n)} \\ &=\sum\limits_{m=1}^{\infty}\sum\limits_{n=1}^{\infty}\dfrac{mn}{2^{m+n}} \\ &=\left(\sum\limits_{m=1}^{\infty}\dfrac{m}{2^m}\right)\left(\sum\limits_{n=1}^{\infty}\dfrac{n}{2^n}\right). \end{aligned}\\

回顾:设f(x)=\sum\limits_{n=1}^{\infty}x^n=\dfrac{x}{1-x},则

f'(x)=\sum\limits_{n=1}^{\infty}nx^{n-1}=\dfrac{1}{(1-x)^2}. \\

令x=\dfrac{1}{2},得

\sum\limits_{n=1}^{\infty}\dfrac{n}{2^n}=\dfrac{1}{2}f'(\dfrac{1}{2})=2. \\

所以2S=4,S=2,\lfloor 100S\rfloor=\boxed{200}.

6. 方程\prod\limits_{k=1}^6(x^5-5x+k)=0有 ______________ 个实根.
【Fiddie解答】 注意到当k=4时,方程 x^5-5x+4=0 有二重根1.于是可画出曲线 y=x^5 与 y=5x-k(k=1,\cdots,6) 的草图如下.

可知对所有 k,在x < 0时有一个交点.当 k=1,2,3 时,有3个交点;当 k=4 时,有2个交点;当 k=5,6 时,有1个交点.故原方程的实根的个数是

3+3+3+2+1+1=\boxed{13}. \\


7. 设 A 是一个三阶方阵,其元素为 1,2,\cdots,9,且满足每行元素从左到右、每列元素从上到下递增.则满足条件的 A 有 _________ 个.
【Fiddie解答】 首先可确定1在左上角,9在右下角.接下来确定2和8,有下面四种情况:

\begin{pmatrix} 1&2& \\ & & \\ &8&9 \end{pmatrix}, \begin{pmatrix} 1&2& \\ & &8\\ & &9 \end{pmatrix}, \begin{pmatrix} 1& & \\ 2& &8\\ & &9 \end{pmatrix}, \begin{pmatrix} 1& & \\ 2& & \\ &8&9 \end{pmatrix}\\

我们可以发现第1、第3种情况是一样的(只是作了个转置),第2、第4种也是一样的(只是作了个转置),于是我们只需考虑前两种情况.

第一种情况:先确定3和7的位置.一共有下面几种:

\begin{pmatrix} 1&2&3\\ & & \\ 7&8&9 \end{pmatrix}, \begin{pmatrix} 1&2&3\\ & &7\\ &8&9 \end{pmatrix}, \begin{pmatrix} 1&2& \\ 3& & \\ 7&8&9 \end{pmatrix}, \begin{pmatrix} 1&2& \\ 3& &7\\ &8&9 \end{pmatrix}\\

然后就能确定4、5、6的位置了,上述四种情况的4、5、6的放置方法数分别为1、2、2、6.因此一共有11种方法.

第二种情况:先确定3和7的位置.一共有下面几种:

\begin{pmatrix} 1&2&3\\ & &8\\ &7&9 \end{pmatrix}, \begin{pmatrix} 1&2&7\\ 3& &8\\ & &9 \end{pmatrix}, \begin{pmatrix} 1&2& \\ 3& &8\\ &7&9 \end{pmatrix}\\

然后就能确定4、5、6的位置了,上述三种情况的4、5、6的放置方法数分别为2、2、6.因此一共有10种方法.

综上,一共有2\times (11+10)=\boxed{42}个满足条件的矩阵.

8. 对矩阵 P,定义 \displaystyle\mathrm{e}^P=\sum\limits_{m=0}^{\infty}\dfrac{1}{m!}P^m.设20阶实矩阵 A 满足 \mathrm{e}^A=I_{20} 且 A 的所有特征值的模长不超过 20,则这样的互不复相似的 A 有 ___________ 个.
【Fiddie解答】 首先设 A=PJP^{-1},于是 \mathrm{e}^A=I_{20} 等价于 \mathrm{e}^J=I_{20}.在复相似意义下,只需考虑 A 的 Jordan 标准型 J.

于是,对于 J 中的每个 n 阶 Jordan 块 J_n(\lambda),有 \mathrm{e}^{J_n(\lambda)} = I_n.

若n\ge 2,则

\mathrm{e}^{J_n(\lambda)}=\mathrm{e}^{\lambda I+J_n(0)} =\mathrm{e}^{\lambda I}\mathrm{e}^{J_n(0)},\\

(这里利用了 \lambda I 和 J_n(0) 可交换)

但是当 n\ge 2 时,\mathrm{e}^{\lambda I}=\mathrm{e}^{\lambda} I,

\mathrm{e}^{J_n(0)}=\begin{pmatrix} 0&1&\dfrac{1}{2!}&\cdots&\dfrac{1}{(n-1)!} \\ 0&0&1&\cdots&\dfrac{1}{(n-2)!} \\ \vdots&\vdots&\vdots& & \vdots \\ 0&0&0&\cdots& 1\\ 0&0&0&\cdots&0 \end{pmatrix}\\

所以此时 \mathrm{e}^{J_n(\lambda)} 不可能等于 n 阶单位矩阵.矛盾.

综上,必有 n=1,即 A 可对角化.

J=\mathrm{diag}(\lambda_1,\cdots,\lambda_{20}). \\

因为

\mathrm{e}^J=I_{20}, \\

所以对 k=1,\cdots,20,有 \mathrm{e}^{\lambda_k}=1,则存在整数 m 使得 \lambda_k=2\pi m\mathrm{i}.而 |\lambda_k| \le 20,所以 |m|\le 3.

由于 A 是实矩阵,所以 A 的复特征值都互为共轭,实特征值只可能为 0.于是可改写

J=\mathrm{diag}(\lambda_1,\overline{\lambda}_1,\cdots,\lambda_{10},\overline{\lambda}_{10}). \\

其中 |\lambda_1| \ge \cdots \ge |\lambda_{10}|.这样,\lambda_1,\cdots,\lambda_{10} 在 \{2\pi m\mathrm{i}|m=0,1,2,3\} 中选取.采取插空法,在排成一列的 13 个空中,选3个位置插入字母 a,b,c,剩下10个位置把 \lambda_1,\cdots,\lambda_{10} 按模长从大到小依次排列,位于 a 之前的取 m=3,位于 a,b 之间的取 m=2,位于 b,c 之间的取 m=1,位于 c 之后的取 m=0.所以,\lambda_k 的位置可由字母 a,b,c 在排列中所处的位置唯一确定.

所以满足条件的方法数是\binom{13}{3}=\boxed{286}.

9. 设有理数加群 (\mathbb{Q},+) ,记所有分母不超过 10 的有理数构成的子集为 G,陪集 G\mathbb{Z} 记为 \overline{G},则 \mathbb{Q}/\mathbb{Z} 包含 \overline{G} 的最小子群的阶为 ___________.
【Fiddie解答】 设 H 是包含 \overline{G} 的最小子群.

由题意,\dfrac{1}{5},\dfrac{1}{7},\dfrac{1}{8},\dfrac{1}{9}\in G,而

\dfrac{1}{35}=\dfrac{3}{7}-\dfrac{2}{5}, \dfrac{1}{72}=\dfrac{1}{8}-\dfrac{1}{9}, \\

所以 \dfrac{1}{35}+\mathbb{Z}\in H,\dfrac{1}{72}+\mathbb{Z}\in H.

根据带余除法,存在 a,b\in\mathbf{Z},使得

35a+72b=1. \\

所以

\dfrac{1}{2520}=\dfrac{35a+72b}{2520} =\dfrac{a}{72}+\dfrac{b}{35},\\

得 \dfrac{1}{2520}+\mathbb{Z}\in H.

另一方面,1,2,\cdots,10 的最小公倍数是 2520,所以找不到大于 2520 的整数 n 使得 \dfrac{1}{n}\in H.于是,H=\dfrac{1}{2520}\mathbb{Z}/\mathbb{Z},|H| = \boxed{2520}.

10. 设 V 是 10 维列向量构成的内积空间.对 u\in V,定义 V 上线性变换 P_u 如下:
P_u:V\to V, \quad x\mapsto x-\dfrac{2\langle x,u\rangle u}{2\langle u,u\rangle}. \\ 给定 v,w\in V,满足 0 < \langle v,w\rangle < \sqrt{\langle v,v\rangle\langle w,w\rangle}.记 Q=P_v\circ P_w,则所有满足 P\circ Q=Q\circ P 的线性变换 P:V\to V 构成的空间的维数是 __________ .
【Fiddie解答】 P_u 是关于向量 u 的反射.

以下考虑简单情形,取 v=e_1=(1,0,0,\cdots,0),w=e_1+e_2=(1,1,0,\cdots,0). 于是

\begin{aligned} P_v(x)&=x-\dfrac{2\langle x,v\rangle}{\langle v,v\rangle}v= x-2x_1e_1, \\ P_w(x)&=x-\dfrac{2\langle x,w\rangle}{\langle w,w\rangle}w= x-(x_1+x_2)(e_1+e_2), \\ \end{aligned}\\

则 P_v 对应的矩阵为

A=\begin{pmatrix} -1 \\ & 1 \\ & & 1 \\ & & & \ddots \\ & & & & 1 \\ & & & && 1 \end{pmatrix}\\

P_w 对应的矩阵为

B=\begin{pmatrix} 0 & -1 \\ -1 & 0 \\ & & 1 \\ & & & \ddots \\ & & & & 1 \\ & & & & & 1 \end{pmatrix}\\

所以 A^2=I,B^2=I.记 Q 对应的矩阵为 C,则

\begin{aligned} P\circ Q=Q\circ P &\Leftrightarrow CAB=ABC \\ & \Leftrightarrow ACA=BCB. \end{aligned}\\

设 C=(c_{ij})_{n\times n},则

ACA=\begin{pmatrix} c_{11} & -c_{12} & -c_{13} & \cdots & -c_{1n} \\ -c_{21} & c_{22} & c_{23} & \cdots & c_{2n} \\ -c_{31} & c_{32} & c_{33} & \cdots & c_{3n} \\ \vdots & \vdots & \vdots & & \vdots \\ -c_{n1} & c_{n2} & c_{n3} & \cdots & c_{nn} \end{pmatrix}\\

BCB=\begin{pmatrix} c_{22} & c_{21} & -c_{23} & \cdots & -c_{2n} \\ c_{12} & c_{11} & -c_{13} & \cdots & -c_{1n} \\ -c_{32} & -c_{31} & c_{33} & \cdots & c_{3n} \\ \vdots & \vdots & \vdots & & \vdots \\ -c_{n2} & -c_{n1} & c_{n3} & \cdots & c_{nn} \end{pmatrix}\\

比较系数可得

c_{11}=c_{22}, \qquad c_{21}=c_{12}, \\

对k=3,4,\cdots,n,c_{1k}=0,c_{2k}=0,c_{k1}=0,c_{k2}=0. 所以一共有

2+4(n-2)=4n-6=34 \\

个线性约束条件.因此所有线性变换 P 构成的空间维数是 10^2-34=\boxed{66}.

11. 10阶矩阵 A 满足每行恰有 5 个 1 和 5 个 0,则使得 A^2+5A 是一个全部元素均为 5 的矩阵的 A 有 __________ 个.
【Fiddie解答】 首先对角元不能为 1,如果 a_{11}=1,则 B=A^2+5A 中,b_{11} \ge 6,矛盾.

下面我们假设 A 的第一行为 (0,1,1,1,1,1,0,0,0,0).由条件,A^2 的第一行是 (5,0,0,0,0,0,5,5,5,5),于是可以唯一确定 A 的第一到第六行如下:

\begin{pmatrix} 0 & 1 & 1 & 1 & 1 & 1 & 0 & 0 & 0 & 0 \\ 1 & 0 & 0 & 0 & 0 & 0 & 1 & 1 & 1 & 1 \\ 1 & 0 & 0 & 0 & 0 & 0 & 1 & 1 & 1 & 1 \\ 1 & 0 & 0 & 0 & 0 & 0 & 1 & 1 & 1 & 1 \\ 1 & 0 & 0 & 0 & 0 & 0 & 1 & 1 & 1 & 1 \\ 1 & 0 & 0 & 0 & 0 & 0 & 1 & 1 & 1 & 1 \\ \ast & \ast & \ast & \ast & \ast & \ast & \ast & \ast & \ast & \ast \\ \vdots & \vdots & \vdots & \vdots & \vdots & \vdots & \vdots & \vdots & \vdots & \vdots \end{pmatrix}\\

又因为 A^2 的第二行到第六行都是 (0,5,5,5,5,5,0,0,0,0),所以可以唯一确定整个矩阵 A 是

\begin{pmatrix} 0 & 1 & 1 & 1 & 1 & 1 & 0 & 0 & 0 & 0 \\ 1 & 0 & 0 & 0 & 0 & 0 & 1 & 1 & 1 & 1 \\ 1 & 0 & 0 & 0 & 0 & 0 & 1 & 1 & 1 & 1 \\ 1 & 0 & 0 & 0 & 0 & 0 & 1 & 1 & 1 & 1 \\ 1 & 0 & 0 & 0 & 0 & 0 & 1 & 1 & 1 & 1 \\ 1 & 0 & 0 & 0 & 0 & 0 & 1 & 1 & 1 & 1 \\ 0 & 1 & 1 & 1 & 1 & 1 & 0 & 0 & 0 & 0 \\ 0 & 1 & 1 & 1 & 1 & 1 & 0 & 0 & 0 & 0 \\ 0 & 1 & 1 & 1 & 1 & 1 & 0 & 0 & 0 & 0 \\ 0 & 1 & 1 & 1 & 1 & 1 & 0 & 0 & 0 & 0 \end{pmatrix}\\

也就是说,只要确定第 1 行的所有元素,就能确定其它行的所有元素.注意到交换 A 的任意两行或两列不会改变 A^2+5A 的值,所以所有方法数是 A 的第一行的 5 个 1 所处非对角位置的总数,一共是 \binom{9}{5}=\boxed{126}.

12. 设 5\times 5 矩阵 A 的元素均取自 \{0,1,\cdots,14\},满足 (\det A,15)=1 的矩阵 A 的个数记为 N,v_2(N) 表示 N 的质因数分解中 2 的次数.则 v_2(N)= _________.
【Fiddie解答】 设 \mathbb{F}_p 是 p 元域 \mathbb{Z}/p\mathbb{Z}(p 是素数),一般线性群定义为

GL_n(\mathbb{F}_p)=\{\text{分量都在}\mathbb{F}_p\text{的}n\text{阶可逆矩阵}\}. \\

在模 15 的意义下考虑矩阵 A 的每个元素.定义映射:

f:\{A|(\det A,15)=1\}\to GL_n(\mathbb{F}_3) \times GL_n(\mathbb{F}_5), \\

如下:对 A 中每个元素 a_{ij},存在唯一的 b_{ij}\in\{0,1,2\},c_{ij}\in\{0,1,2,3,4\},使得 a_{ij}=5b_{ij}+3c_{ij} .于是定义

B=(b_{ij}), \qquad C=(c_{ij}). \\

这样,在模 15 的意义下,有

A=5B+3C. \\

因为 (\det A,15)=1 当且仅当 (\det A,3)=1 且 (\det A,5)=1,当且仅当在模 3 的意义下和模 5 的意义下 A 都是非奇异矩阵.而模 3 的意义下 A=5B,模 5 的意义下 A=3C.所以 (\det A,15)=1 当且仅当 (\det B,3)=1 且 (\det C,5)=1,当且仅当 B\in GL_n(\mathbb{F}_3) 且 C\in GL_n(\mathbb{F}_5).

反之,对 B\in GL_n(\mathbb{F}_3) 且 C\in GL_n(\mathbb{F}_5),取 A=5B+3C.则 (\det A,5)=(\det 3C,5)=1,(\det A, 3)=(\det 5B,3)=1,所以 (\det A,15)=1.

因此 f 是双射,所以

|\{A|(\det A,15)=1\}| =|GL_n(\mathbb{F}_3)|\cdot|GL_n(\mathbb{F}_5)|.\\

下面,熟知如下结论:对素数 p,

|GL_n(\mathbb{F}_p)| = (p^n-1)(p^n-p)\cdots(p^n-p^{n-1}), \\

代入 p=3,p=5,n=5 可知 v_2(|GL_n(\mathbb{F}_3)|)=10, v_2(|GL_n(\mathbb{F}_5)|)=13, 从而 v_2(N) = 10+13=\boxed{23}.

13. 给定 3\times 3 整数元矩阵 M=\begin{pmatrix} 11 & 11 & 20 \\ 10 & 10 & -8 \\ -10 & 2 & 8 \end{pmatrix},已知存在整数元矩阵 P,Q,满足 \det P,\det Q\in\{\pm 1\},且 PMQ 是对角矩阵,则 PMQ 的最大元素的最大值为 _______________.
【Fiddie解答】 由条件 M 与一个对角阵相抵.所以只需作初等行变换或初等列变换(这些变换的行列式都是1)把 M 变成对角矩阵.

\begin{aligned} \begin{pmatrix} 11 & 11 & 20 \\ 10 & 10 & -8 \\ -10 & 2 & 8 \end{pmatrix} &\rightarrow \begin{pmatrix} 11 & 11 & 20 \\ -1 & -1 & -28 \\ 1 & 13 & 28 \end{pmatrix} \\ &\rightarrow \begin{pmatrix} 0 & -132 & 288 \\ 0 & 12 & 0 \\ 1 & 13 & 28 \end{pmatrix} \\ &\rightarrow \begin{pmatrix} 0 & 0 & 288 \\ 0 & 12 & 0 \\ 1 & 13 & 28 \end{pmatrix} \\ &\rightarrow \begin{pmatrix} 1 & \\ & 12 \\ & & 288 \end{pmatrix} \\ \end{aligned}\\

所以 PMQ 的最大元素的最大值是 \boxed{288}.

14. 令 \displaystyle I=\sqrt{\dfrac{2}{\pi}}\int_{-\infty}^{+\infty}\mathrm{e}^{x-10\cosh(2x)}\mathrm{d}x,则 \lfloor |\ln I|\rfloor= __________.
【Fiddie解答】 把 I 的 x 换为 -x 得

\begin{aligned} I&=\sqrt{\dfrac{2}{\pi}}\int_{-\infty}^{+\infty}\mathrm{e}^{x-10\cosh(2x)}\mathrm{d}x \\ I&=\sqrt{\dfrac{2}{\pi}}\int_{-\infty}^{+\infty}\mathrm{e}^{-x-10\cosh(2x)}\mathrm{d}x \\ \end{aligned}\\

两式相加,得

\begin{aligned} I&=\sqrt{\dfrac{2}{\pi}}\int_{-\infty}^{+\infty}\dfrac{\mathrm{e}^{-x}+\mathrm{e}^x}{2}\mathrm{e}^{-10\cosh(2x)}\mathrm{d}x \\ &=\sqrt{\dfrac{2}{\pi}}\int_{-\infty}^{+\infty}\cosh x\cdot \mathrm{e}^{-10(1+2(\sinh x)^2)}\mathrm{d}x \\ &=\sqrt{\dfrac{2}{\pi}}\int_{-\infty}^{+\infty}\mathrm{e}^{-10(1+2(\sinh x)^2)}\mathrm{d}(\sinh x)\\ &=\sqrt{\dfrac{2}{\pi}}\int_{-\infty}^{+\infty}\mathrm{e}^{-10(1+2t^2)}\mathrm{d}t\\ &=\sqrt{\dfrac{2}{\pi}}\cdot \mathrm{e}^{-10} \cdot \dfrac{\sqrt{\pi}}{\sqrt{20}} = \dfrac{\mathrm{e}^{-10}}{\sqrt{10}}. \end{aligned}\\

所以,

\lfloor |\ln I|\rfloor = \left\lfloor 10+\dfrac{1}{2}\ln 10\right\rfloor = \boxed{11}. \\

15. 已知椭球体 \dfrac{x^2}{6}+\dfrac{y^2}{9}+z^2\le 1,记过 (0,0,1) 的平面与该椭球的截面面积最大值为 A,则 \left\lfloor\dfrac{100A^2}{\pi^2}\right\rfloor= ___________.
【Fiddie解答】 点 P(0,0,1) 是椭球z轴方向上的顶点,所以截面不平行于 xOy 平面.设平面方程为

ax+by+(z-1)=0. \\

平面与椭球所截曲面

\Omega:\left\{\begin{aligned} &ax+by=(z-1), \\ &\dfrac{x^2}{6}+\dfrac{y^2}{9}+z^2 \le 1. \end{aligned}\right.\\

记 \Gamma=\partial\Omega,则

\Gamma:\left\{\begin{aligned} &ax+by=(z-1), \\ &\dfrac{x^2}{6}+\dfrac{y^2}{9}+z^2 = 1. \end{aligned}\right.\\

消去 z 可得 \Gamma 在 xOy 平面上的投影曲线是

(a^2+\dfrac{1}{6})x^2+2abxy+(b^2+\dfrac{1}{9})-2ax-2by+1=0. \\

这是一个椭圆,记它的内部为 E.它在正交变换下的不变量有

\begin{aligned} &I_2=\begin{vmatrix} a^2+\dfrac{1}{6} & ab \\ ab & b^2+\dfrac{1}{9}\end{vmatrix}, \\ &I_3=\begin{vmatrix} a^2+\dfrac{1}{6} & ab & -a \\ ab & b^2+\dfrac{1}{9} & -b \\ -a & -b & 1\end{vmatrix}, \\ \end{aligned}\\

于是可以用正交变换化为 \lambda_1(x')^2+\lambda_2(y')^2+\dfrac{I_3}{I_2}=0.\\

\dfrac{(x')^2}{\frac{I_3}{-I_2\lambda_1}}+\dfrac{(y')^2}{\frac{I_3}{-I_2\lambda_2}}=1. \\

其中\lambda_1\lambda_2=I_2.由椭圆的面积公式得这个椭圆的面积是

|E|=\pi\sqrt{\frac{I_3}{-I_2\lambda_1}\cdot \frac{I_3}{-I_2\lambda_2}} = \pi\dfrac{-I_3}{I_2^{3/2}}. \\

平面与椭球所截的平面 \Omega 可写成参数曲面 z=ax+by+1,(x,y)\in E.其面积为

\begin{aligned} S&=\int_{E}\sqrt{1+z_x^2+z_y^2}\mathrm{d}x\mathrm{d}y \\ &=\sqrt{1+a^2+b^2}|E| \\ &=\sqrt{1+a^2+b^2}\cdot\dfrac{\frac{1}{9}a^2+\frac{1}{6}b^2}{(\frac{1}{9}a^2+\frac{1}{6}b^2+\frac{1}{54})^{3/2}}\pi. \end{aligned}\\

所以

\begin{aligned} S&\le\sqrt{1+9(\frac{1}{9}a^2+\frac{1}{6}b^2)}\cdot\dfrac{\frac{1}{9}a^2+\frac{1}{6}b^2}{(\frac{1}{9}a^2+\frac{1}{6}b^2+\frac{1}{54})^{3/2}}\pi. \end{aligned}\\

令 x=\dfrac{1}{9}a^2+\dfrac{1}{6}b^2,取函数

f(x)=\dfrac{x^2(1+9x)}{(x+\frac{1}{54})^3}\pi. \\

\begin{aligned} f'(x)&=\dfrac{(2x+27x^2)(x+\frac{1}{54})-3(x^2+9x^3)}{(x+\frac{1}{54})^4}\pi \\ &=\dfrac{x(-\frac{1}{2}x+\frac{1}{27})}{(x+\frac{1}{54})^4}\pi \end{aligned}\\

所以 f(x) 在 (0,\dfrac{2}{27}) 单调递增,在 (\dfrac{2}{27},+\infty) 单调递减.从而 f(x) 的最大值是 f(\dfrac{2}{27})=\dfrac{32\times 9}{25}\pi.

因此 S\le \sqrt{f(\dfrac{2}{27})},等号成立条件是 a^2=\dfrac{2}{3},b^2=0.于是 A^2=\dfrac{32\times 9}{25}\pi,

\left\lfloor 100\dfrac{A^2}{\pi^2}\right\rfloor = \boxed{1152}. \\

编辑于 2024-05-27 15:17・IP 属地江苏
真诚赞赏,手留余香
赞赏
还没有人赞赏,快来当第一个赞赏的人吧!
​赞同 32​
​添加评论
​分享
​收藏
​喜欢

收起​
查看全部 12 个回答

%https://www.zhihu.com/question/624467932/answer/3510945541


\section{2021年清华大学数学九推考试真题}

 

1.与线性变换有关,疑似错题.

 

2.设$A,B$为数域$\mathbb{F}$上$n$阶方阵.求证:


(a)对$\forall\lambda\in\mathbb{F}$,均成立$\lambda I_n-AB$可逆$\Leftrightarrow\lambda I_n-BA$可逆.


(b)若$r(AB)=r(B)$,则$r(AB^2)=r(B^2)$.


3.设$V$为$\mathbb{C}$上的$n$维线性空间, $\varphi$为$V$上的线性变换,其全体不同特征值为$\lambda_1,\lambda_2,\cdots,\lambda_r$.
求证: $\varphi$可对角化$\Leftrightarrow$存在 $1\leqslant i\leqslant r$,使得 $\mathrm{Im}\left(\varphi-\lambda_{i}I\right)
=\bigoplus_{j\neq i}\mathrm{Ker}\left(\varphi-\lambda_{j}I\right).$


4.设 $f\in C[0,1]$,对任意$[0,1]$上的可微函数$g$均成立$\int_{0}^{1}f(x)g(x)\mathrm{d}x=0$,求证: $f\equiv 0$.


5.求证Green第二公式.


6.与连续函数介值定理有关.


7.设区域$D\subset\mathbb{C}$, $\{f_n\}\subset H(\overline{D})$,存在$\overline{D}$上的复变函数$f$,使得$\{f_n\}$在$\partial D$上一致收敛于$f$.求证:


(a) $\{f_n\}$在$D$中内闭一致收敛于$f\in H(D)$.


(b)若$\{f_n\}$是$D$中的单叶函数列, $f$不是常值函数,则$f$也是$D$中的单叶函数.

 


\section{2023年清华大学直博考试真题}

 

高代:

1.设$A$是实对称三阶阵,每行元表和为$4$, $\alpha_1= (1,-1,0)^{T},\alpha_{2}=(1,-2,1)^{T}$为$(I_{3}-A)x=0$ 的解.求正文阵$Q$,使得$Q^TAQ$为对角阵.


2.设$V$为$n$维线性空间, $\phi,\psi:V\to V$是线性映射, $\phi^2=\phi$, $\mathrm{Ker}\phi,\mathrm{Im} \phi$为$\psi-$不变子空间.求证: $\phi\psi=\psi\phi$.


3.设$P$为实矩阵.求证: $\left( \begin{matrix}
& P^T\\
P& \\
\end{matrix} \right)$
的正交惯性指数相等,且均等于$\mathrm{rank}(P)$.


4.设$A$为$n$阶可逆阵, $S\in \mathbb{N}^+$, $A^s$可对角化,求证$A$可对角化,举例说明$A$可逆条件不能删去.

 

分析:

1.设$\left\{\alpha_{n}\right\}_{n=0}^{\infty}\subseteq \mathbb{R}$, $\alpha_{n}\to\alpha$,求证:

(1) $\sum_{n=0}^{\infty}\alpha_{n}\frac{x^{n}}{n!}e^{-x}$
在$\mathbb{R}$上收敛.

(2)求$\lim_{x\to +\infty}\sum_{n=0}^{\infty}\alpha_{n}\frac{x^{n}}{n!}
e^{-x}$.

(3)问: $\sum_{n=0}^{\infty}\alpha_{n}\frac{x^n}{n!}e^{-x}$是否在$(0,+\infty)$上一致收敛.

 

2.设$f:\mathbb{R}^{n}\to \mathbb{R}$, $f\in C^0(\mathbb{R}^{n})$, $f$在$\mathbb{R}^{n}\backslash \{0\}$可微,且$\lim_{x\to 0}\frac{\partial f}{\partial x_i}|x|=0,\forall i$,求证: $f$在$\mathbb{R}^{n}$上可微.

3.设$f\in C^1(\Omega\subseteq \mathbb{R}^{m+n},\mathbb{R}^{m}),x_0\in \Omega$, $\det Jf(X_{0})\neq 0$,求证: $\exists x_{0}$的开邻域$U\subseteq \Omega$, $C^1$同胚$\varphi:
U\to V$,使得$f_\circ \varphi^{-1}(y_{1},\cdots,y_{m},y_{m+1},
\cdots,y_{m+n})=(y_{1},\cdots,y_{m})$.

 

4.设$D\subseteq \mathbb{R}^{3}$为坐标平面, $x+y=1$, $x^ {2}+z^{2}=1$所围成的在第一象限$(x,y,z>0)$的区域,计算:
$$\int_{\partial D}(2xz\mathrm{d}y\wedge \mathrm{d}z+y\mathrm{d}z\wedge \mathrm{d}x+z^{2}\mathrm{d}x\wedge \mathrm{d}y),$$
其中$\partial D$取外侧.

5.设$\gamma\subseteq C$为简单闭曲线, $f$在$\gamma$围成的无界区域$D$内全纯, $f(+\infty)=\alpha$,且在$\overline{D}$上连续,计算:
$$\frac{1}{2\pi i}\int_{\gamma}\frac{f(\xi)}{\xi-z}\mathrm{d}\xi,$$
其中$z$在$\gamma$围成的有界区域内.

6.设$f:\mathbb{D}\to\mathbb{D}$ (unit disk)全纯, $f(a)=0$,求证: $f$在$D(0,|a|)=\{z:|z|<|a|\}$内无零点.

 

 

\section{2024年5月17日清华大学数学科学系直博笔试试题}

 

1.设$A=\begin{pmatrix}
-3&2&2\\
2&-3&2\\
2&2&-3\end{pmatrix}$,试求矩阵$Q$,使得$Q^\mathrm{T}AQ$为对角矩阵,并求$A$的谱分解.


2.设$AB=BA$,求证$AB^\ast=B^\ast A$,其中$B^\ast$表示$B$的伴随矩阵.


3.设$\mathbb{F}$为数域, $A\in\mathbb{F}^{m\times m},B\in\mathbb{F}^{n\times n}$,并且
$$\phi:\mathbb{F}^{m\times n}\to\mathbb{F}^{m\times n},\quad x\mapsto AXB.$$
求证:若$\lambda,\mu$分别为$A,B$的特征值,则$\lambda\mu$为$\phi$的特征值,并且$\phi$的特征值均有此形式.

4.设$A$为$n$阶矩阵, $r(A)=r$.求证 $I,A,\cdots,A^{r+1}$线性相关.

5.设$f:\mathbb{R}^n\to\mathbb{R}$为局部常值函数,求证$f$为常值函数.

6.设 $B=\left\{(x,y)\in\mathbb{R}^2:x^2+y^2\leqslant 1\right\}$,试找出$u:B\to\mathbb{R},(x,y)\mapsto xy^2$ 的驻点,判断其类型,并求出$u$的最值.

 

7.设$I(\alpha)=\int_{\mathbb{R}^2}
\frac{\sin\left(x^2+y^2\right)}
{\left(x^2+y^2\right)^\alpha}\operatorname{d}x \operatorname{d}y,\alpha\in\mathbb{R}$,试求$\alpha$的范围使得$I(\alpha)$收敛,并求出其收敛时的值.

 

8.设$f,g\in C(\mathbb{R},\mathbb{R})$周期为$2\pi$,并且二者具有相同的Fourier展开式,求证$f\equiv g$.


9.设$f:\mathbb{C}\to\mathbb{C}$为全纯函数,并且$\mathrm{Im} f(z)>0,\forall z\in\mathbb{C}$.求证$f$为常值函数.


10.求$\int_{-\infty}^{+\infty}\frac1{\cosh x}\mathrm{d}x$及$\int_{-\infty}^{+\infty}\frac x{\sinh x}\mathrm{d}x$.

 

16道用数学归纳法证明的不等式


1.用数学归纳法证明下列不等式.

(1). (柯西不等式)设$a_1,a_2,\cdots,a_n$和$b_1,b_2,\cdots,b_n$是两个实数列,则
$$(a_1^2+a_2^2+\cdots+a_n^2)(b_1^2+b_2^2+\cdots+b_n^2)\geqslant
(a_1b_1+a_2b_2+\cdots+a_nb_n)^2.$$


(2) (权方和不等式)
$$\frac{a_1^2}{b_1}+\frac{a_2^2}{b_2}+\cdots+\frac{a_n^2}{b_n}\geqslant
\frac{(a_1+a_2+\cdots+a_n)^2}{b_1+b_2+\cdots+b_n}.$$


(3) (闵科夫斯基不等式)
$$\sqrt{a_1^2+b_1^2}+\sqrt{a_2^2+b_2^2}+\cdots+\sqrt{a_n^2+b_n^2}
\geqslant\sqrt{(a_1+a_2+\cdots+a_n)^2+(b_1+b_2+\cdots+b_n)^2}.$$


(4) (平方平均不等式)
$$(a_1+a_2+\cdots+a_n)^2\leqslant n(a_1^2+a_2^2+\cdots+a_n^2).$$


(5) (琴生不等式)假设$f(x)$时定义在$[a,b]\in\mathbb{R}$上的凸函数,实数$x_1,x_2,\cdots,x_n\in[a,b]$,对
所有非负实数$a_1, a_2, \cdots , a_n\geqslant 0$,
$a_1+ a_2+ \cdots + a_n= 1$,则有
$$a_1f(x_1)+a_2f(x_2)+\cdots+a_nf(x_n)\geqslant f(a_1x_1+a_2x_2+\cdots+a_nx_n).$$

(推论)若对于某种平均运算$M$ (算术,几何,调和等),当$n=2$时有
$M(f(x),f(y))\geqslant f(M(x,y))$,则对一般的$n$,有
$$M(f(x_1),f(x_2),\cdots,f(x_n))\geqslant f(M(x_1,x_2,\cdots,x_n)).$$


2. (1992年APMO)令$a_1,a_2,\cdots$是一个数列,对于所有的$i,j$,满足$a_i+j\leqslant a_i+a_j$.证明:
对于每个正整数$n$,有
$$a_1+\frac{a_2}{2}+\frac{a_3}{3}+\cdots+\frac{a_n}{n}\geqslant a_n.$$


3. (1988年CMO)在有限项的实数列$a_1,a_2,\cdots,a_n$ 中,如果有一段数$a_k,\cdots,a_{k+l-1}$的算术平均数大于$1988$,那么我们把这段数叫做一条“龙”,并把$a_k$ 称为这条龙的“龙头” (如果某一项$a_m>1988$,那么单独这一项也是龙).假定数列中至少存在一条龙,证明:数列中全体可以作为龙头的项的算术平均数也必定大于$1988$.

 

4. (2006年CMO)实数列$\left\{a_n\right\}$满足
$$a_1=\frac{1}{2},a_{k+1}=-a_k+\frac{1}{2-a_k},k=1,2,\cdots $$
证明不等式:
$$\left(\frac n{2(a_1+a_2+\cdots+a_n)}-1\right)^n\leqslant
\left(\frac{a_1+a_2+\cdots+a_n}n\right)^n\cdot
\prod_{i=1}^n\left(\frac1{a_i}-1\right).$$


5. (2009年CTST) 给定整数$n\geqslant 2$,求具有下述性质的最大常数$\lambda(n)$:若实数序列$a_0,a_1,a_2,\cdots,a_n$满足
$$0=a_0\leqslant a_1\leqslant\cdots\leqslant a_n,\quad a_i\geqslant\frac12(a_{i+1}+a_{i-1}),i=1,2,\cdots,n-1,$$
则有
$$\left(\sum_{i=1}^nia_i\right)^2\geqslant
\lambda(n)\sum_{i=1}^na_i^2.$$


6. (2013年CTST)设整数$n\geqslant 2$, $a_1, a_2, \cdots , a_n$, $b_1, b_2, \cdots , b_n$是非负实数.证明:
$$\left(\frac n{n-1}\right)^{n-1}\left(\frac1n\sum_{i=1}^na_i^2\right)
+\left(\frac1n\sum_{i=1}^nb_i\right)^2\geqslant
\prod_{i=1}^n\left(a_i^2+b_i^2\right)^{\frac1n}.$$

 

7.设 $x_k>0\ (k=1,2,\cdots,n),\sum_{k=1}^{n}x_{k}=1$,求证:
$$\prod_{k=1}^n\frac{1+x_k}{x_k}\geqslant
\prod_{k=1}^n\frac{n-x_k}{1-x_k}.$$

 

 

8.证明对任何正整数 $N$, $\sqrt{2\sqrt{3\cdots \sqrt{\left( N-1 \right) \sqrt{N}}}}<3$.

 

9. (牛顿不等式)设$a_1,a_2,\cdots,a_n$为非负实数. $S_k$表示第$k$个初等对称多项式, $d_k=\frac{S_k}{\binom nk}$,
则$$d_{k-1}d_{k+1}\leqslant d_k^2.$$

 


10.设$a_1=1,a_{2}=2,a_{n+1}=\frac{a_{n}a_{n-1}+1}{a_{n-1}},
n=2,3,\cdots$.证明:对任意正整数 $n\geqslant 3$,都有
$a_n>\sqrt{2n}$.

 


11.已知数列$\{a_n\}$满足$a_0=a_1=1$, $a_n=\sqrt{a_{n-1}a_{n-2}+ \frac n2}\ ( n\geqslant 2)$.证明:存在实数$\lambda$,使得
对任意的正整数$n$,有$\lambda n<a_n<\lambda n+1$.

 

12.设$a>0$,证明:对任意正整数$n$,有
$$\frac{1+a^2+a^4+\cdots+a^{2n}}{a+a^3+a^5+\cdots+a^{2n-1}}\geqslant
\frac{n+1}n.$$

 

13.设 $x_1,x_2,\cdots,x_n\in(0,1),n\geqslant 2$,证明:
$$\frac{\sqrt{1-x_1}}{x_1}+\frac{\sqrt{1-x_2}}{x_2}
+\cdots+\frac{\sqrt{1-x_n}}{x_n}<\frac{\sqrt{n-1}}{x_1x_2\cdots x_n}.$$

 

14.设$a_1,a_2,\cdots,a_n$为非负实数,证明:
$$\sum_{k=1}^n\sqrt{a_k+a_{k+1}+\cdots+a_n}\geqslant
\sqrt{a_1+4a_2+9a_3+\cdots+n^2a_n}.$$

 

15.已知正数$x_1,x_2,\cdots,x_n$满足$x_1x_2\cdots x_n=1$,证明:
$$\left(\sqrt2+x_1\right)\left(\sqrt2+x_2\right)\cdots
\left(\sqrt2+x_n\right)\geqslant (\sqrt2+1)^n.$$

 

16.设$a_1,a_2,\cdots,a_n$是正实数,且满足$\sum_{i=1}^na_i=n$,证明:
$$a_1a_2\cdots a_n\cdot\left(\frac1{a_1}+\frac1{a_2}+\cdots+\frac1{a_n}
-n+3\right)\leqslant 3.$$

 


20道用阿贝尔变换证明的不等式

 

阿贝尔分部求和公式

令$S_0=0$, $S_k=\sum_{i=1}^ka_i\ (1\leqslant k\leqslant n$),则
$$\sum_{k=1}^na_kb_k=\sum_{k=1}^{n-1}S_k(b_k-b_{k+1})+b_nS_n.$$

证明:注意到$S_0=0$,则
\begin{align*}
\sum_{k=1}^na_kb_k &=\sum_{k=1}^n(S_k-S_{k-1})b_k
=\sum_{k=1}^nS_kb_k-\sum_{k=1}^{n-1}S_kb_{k+1}\\
&=\sum_{k=1}^{n-1}S_k(b_k-b_{k+1})+S_nb_n,
\end{align*}
从而结论成立.

 


以下为一些可以用阿贝尔变换证明的不等式:


1. (阿贝尔不等式)设 $x_1,x_2,\cdots,x_n$和$y_1\geqslant y_2\geqslant\cdots\geqslant y_n\geqslant 0$是实数,记
$S_{k}=\sum_{i=1}^{k}x_{i},1\leqslant k\leqslant n$,
设$M= \max \{ S_{1}, S_{2}, \cdots , S_{n}\}$, $m= \min \{ S_{1}, S_{2}, \cdots , S_{n}\}$,则
$$my_1\leqslant x_1y_1+x_2y_2+\cdots+x_ny_n\leqslant My_1.$$

 

2. (排序不等式)设$a_1\leqslant a_2\leqslant \cdots \leqslant a_n$, $b_1\leqslant b_2\leqslant \cdots \leqslant b_n$.
$c_1$, $c_2$, $\cdots $, $c_n$为$b_1$, $b_2$, $\cdots$, $b_n$的一个排列,则
$$a_1b_n+a_2b_{n-1}+\cdots+a_nb_1\leqslant a_1c_1+a_2c_2+\cdots+a_nc_n\leqslant a_1b_1+a_2b_2+\cdots+a_nb_n.$$


3. (改进切比雪夫不等式)设$a_1,a_2,\cdots,a_n$和$b_1,b_2,\cdots,b_n$是实数,且
$$a_1\geqslant\frac{a_1+a_2}2\geqslant\cdots\geqslant
\frac{a_1+a_2+\cdots+a_n}{n},$$
$$b_1\geqslant\frac{b_1+b_2}2\geqslant
\cdots\geqslant\frac{b_1+b_2+\cdots+b_n}n.$$
证明:
$$a_1b_1+a_2b_2+\cdots+a_nb_n
\frac{1}{n}(a_1+a_2+\cdots+a_n)(b_1+b_2+\cdots+b_n).$$

 


4. (Karamata不等式)若$a_1\geqslant a_2\geqslant \cdots \geqslant a_n$, $b_1\geqslant b_2\geqslant \cdots \geqslant b_n$, 且$a_1\geqslant b_1$,
$a_1+ a_2\geqslant b_1+ b_2$, $\cdots$, $a_1+ a_2+ \cdots + a_{n- 1}\geqslant b_1+ b_2+ \cdots + b_{n- 1}$,
$a_1+ a_2+ \cdots + a_n= b_1+ b_2+ \cdots + b_n$.
$f$为一个下凸函数,则
$$f(a_1)+f(a_2)+\cdots+f(a_n)\geqslant f(b_1)+f(b_2)+\cdots+f(b_n).$$

若$f$为上凸函数,则不等号反向.

 

5.给定正整数$n$,对任意$1\leqslant k\leqslant n$,正实数$a_1,a_2,\cdots,a_n$满足$a_1+a_2+\cdots+a_k\leqslant k$,证明:
$$\frac{a_1}{1}+\frac{a_2}{2}+\cdots+\frac{a_n}{n}\leqslant
\frac{1}{1}+\frac{1}{2}+\cdots+\frac{1}{n}.$$

 


6. (1978年IMO)己知$a_1,a_2,\cdots,a_n$是两两不同的自然数,证明:
$$a_1+\frac{a_2}{2^2}+\frac{a_3}{3^2}+\cdots+\frac{a_n}{n^2}\geqslant
1+\frac12+\frac13+\cdots+\frac1n.$$

 

7. (1999年APMO)设$a_1,a_2,\cdots$是正实数列,且对所有 $i,j=1,2,\cdots$,满足 $a_{i+j}\leqslant a_i+a_j$,证明:
$$a_1+\frac{a_2}2+\frac{a_3}3+\cdots+\frac{a_n}n\geqslant a_n,n\in\mathbf{N}^+.$$


8.给定实数$a_1,a_2,\cdots,a_{n+1}$,记$M=\max_{1\leqslant k\leqslant n}|a_k-a_{k+1}|$,证明:
$$\left|\frac{1}{n}\cdot\sum_{k=1}^na_k
-\frac{1}{n+1}\cdot\sum_{k=1}^{n+1}a_k\right|\leqslant
\dfrac{M}{2}.$$

 

9.已知$a_1,a_2,\cdots,a_n$均为实数,证明:
$$\sum_{i=1}^na_i^2\geqslant\sum_{i=1}^{n-1}a_ia_{i+1}
+\frac3{2(n+1)^3}\cdot\left(\sum_{i=1}^na_i\right)^2.$$


10. (钟开莱不等式)给定整数$n\geqslant 2$,设$a_1\geqslant a_2\geqslant\cdots\geqslant a_n>0$,且对任意$1\leqslant k\leqslant n$,有
$a_1+a_2+\cdots+a_k\leqslant b_1+b_2+\cdots+b_k$,求证:

(1) $a_{1}^{2}+ a_{2}^{2}+ \cdots + a_{n}^{2}\leqslant b_{1}^{2}+ b_{2}^{2}+ \cdots + b_{n}^{2}$;


(2) $a_1^3+ a_2^3+ \cdots + a_n^3\leqslant a_1b_1^2+ a_2b_2^2+ \cdots + a_nb_n^2$.


11. (1994年USAMO)设$a_1,a_2,\cdots,a_n$是正实数,且对任意正整数 $n$,有$\sum_{i=1}^na_i\geqslant\sqrt{n}$,证明:
$$\sum_{i=1}^na_i^2\geqslant
\frac14\cdot\left(1+\frac12+\cdots+\frac1{n-1}+\frac1n\right).$$

12. (2018年清华飞测)对任意$n\in\mathbf{N}^+$,设正实数$a_1,a_2,\cdots,a_n$满足$a_1+a_2+\cdots+a_n\leqslant n^2$,证
明:
$$\frac{1}{a_1}+\frac{1}{a_2}+\cdots+\frac{1}{a_{n-1}}
+\frac{1}{a_n}>\frac{1}{4}\log_2n.$$


13.设 $x_1\geqslant x_2\geqslant\cdots\geqslant x_n\geqslant x_{n+1}=0$,证明
$$\sqrt{x_1+x_2+\cdots+x_n}\leqslant\sum_{i=1}^n\sqrt{i}
\left(\sqrt{x_i}-\sqrt{x_{i+1}}\right).$$


14.设$a_1,a_2,\cdots,a_n\in\mathbf{R},b_1\leqslant b_2\leqslant\cdots\leqslant b_n$,且
$$a_1^2+a_2^2+\cdots+a_k^2\leqslant b_1^2+b_2^2+\cdots+b_k^2,k=1,2,\cdots,n$$
证明:
$$a_1+a_2+\cdots+a_n\leqslant b_1+b_2+\cdots+b_n.$$


15. 对$1\leqslant i\leqslant n$,令$a_i$与$b_i$为实数,且满足$a_1\geqslant a_2\geqslant\cdots\geqslant a_n\geqslant 0$,
$$b_{1}\geqslant a_{1}\\
b_{1}b_{2}\geqslant a_{1}a_{2}\\
b_{1}b_{2}b_{3}\geqslant a_{1}a_{2}a_{3}\\
\vdots\\
b_{1}b_{2}\cdots b_{n}\geqslant a_{1}a_{2}\cdots a_{n}$$
证明:
$$b_1+b_2+\cdots+b_n\geqslant a_1+a_2+\cdots+a_n.$$


16. (2008年CMO)给定正整数$n$,及实数$x_1\leqslant x_2\leqslant\cdots\leqslant x_n,y_1\geqslant y_2\geqslant\cdots y_n$,满足
$\sum_{i=1}^nix_i=\sum_{i=1}^niy_i$.证明:对任意实数$\alpha$,有
$$\sum_{i=1}^nx_i[i\alpha]\geqslant \sum_{i=1}^ny_i[i\alpha].$$

 

17. (2013年CTST)设$n,k$为给定的大于$1$的整数,非负实数$a_1,a_2,\cdots,a_n;c_1,c_2,\cdots,c_n$
满足: $a_1\geqslant a_2\geqslant \cdots \geqslant a_n$, $a_1+ a_2+ \cdots + a_n= 1$, $c_1+ c_2+ \cdots + c_m\leqslant m^k$,
$m=1,2,\cdots,n$.求最大值:
$$c_1a_1^k+c_2a_2^k+\cdots+c_na_n^k.$$


18. (1987年CTST)已知数列$\{r_n\}$满足: $r_1=2,r_n=r_1r_2\cdots r_{n-1}+1,n\geqslant 2$.自然数$a_1,a_2,\cdots,a_{n}$满足$\frac1{a_{1}}+\frac1{a_{2}}+\cdots+\frac1{a_{n}}<1$.求证:
$$\frac{1}{a_{1}}+\frac{1}{a_{2}}+\cdots+\frac{1}{a_{n}}\leqslant
\frac{1}{r_{1}}+\frac{1}{r_{2}}+\cdots+\frac{1}{r_{n}}.$$

 

19. (2000年RMO)设$- 1< x_1< x_2< \cdots < x_n<1$, $y_1< y_2< \cdots<y_n$,且
$$x_1^{13}+x_2^{13}+\cdots+x_n^{13}=x_1+x_2+\cdots+x_n.$$
证明:
$$x_1^{13}y_1+x_2^{13}y_2+\cdots+x_n^{13}y_n<
x_1y_1+x_2y_2+\cdots+x_ny_n.$$


20. (1994年USAMO)设$a_1,a_2,\cdots$是正实数数列,对所有的$n\geqslant 1$满足条件$\sum_{j=1}^na_j\geqslant\sqrt{n}$.证
明对所有的$n\geqslant 1$,
$$\sum_{j=1}^na_j^2>\frac{1}{4}\left(1+\frac{1}{2}+\cdots
+\frac{1}{n}\right).$$

 


\end{QsNum}

 

 

 

 

 

 

\end{document}


首发于
⑨の高中数学竞赛笔记

写文章
点击打开undefined的主页
⑨のn元不等式笔记(1)
⑨のn元不等式笔记(1)
天才琪露诺
天才琪露诺​
琪露诺的算术教室因为双减倒闭了……
​关注她
正式开始之前先碎碎念几句

联赛前在百度贴吧更新了一段时间的n元不等式笔记,有不少竞赛党发现我那个贴子不见了,实际上是账号遇到了一点问题,于是把所有的贴子都给删了

怎么说,高中学竞赛的时候最擅长的就是不等式,然而联赛时看错了题目暴毙了。。。

虽然说自己已经是大学狗了,很少再去花时间研究这些

不过,毕竟前人栽树后人乘凉嘛,我觉得花一点时间把笔记整理出来分享给大家似乎也挺好的

在整理笔记时也会对一些地方进行补充,同时也会添加一些题目,基本上会覆盖大部分TST难度以内的各种技巧

为了督促读者自己动脑去思考,本笔记大多数题目不会给出完整答案,但是绝对会给出较为详细的分析过程,还有笔者的一些经验与碎碎念

更新频率大概一周至少一更

另外,还是不要脸地请求各位,收藏的时候顺带点个赞嘛Orz

Tips:本节所有下标按模 n 考虑

一、关于 \sum ia_i 的处理
1.待定系数

已知 \sum_{i =1}^{n}ix_i=1 ,求: (1) \sum_{i=1}^nx_i^2+\sum_{1\leq i<j\leq n}x_ix_j 的最小值 (2) \sum_{i=1}^nx_i^2+\sum_{1\leq i< j\leq n}(x_i-x_j) ^2 的最小值
分析:

对于(1),我们有 \left(\sum^n_{i=1}ix_i\right)^2=\left[ \sum_{i=1}^n(i-\alpha)x_i+\alpha\sum_{i=1}^nx_i \right]^2\leq\left[ \sum_{i=1}^n(i-\alpha)^2+\alpha^2 \right]\left[ \sum_{i=1}^nx_i^2+ (\sum_{i=1}^nx_i)^2\right]

=2\left[ \sum_{i=1}^n(i-\alpha)^2+\alpha^2 \right]\left[ \sum_{i=1}^nx_i^2+\sum_{1\leq i<j\leq n}x_ix_j\right]

剩下只需配方求最值即可

对于(2),尝试之后发现无法直接待定系数,我们先猜取等条件。分析 n=2,3 时的情况可以猜测当 x_1,x_2,...,x_n 成等差数列时等号成立,这时我们可以分析出待定的形式。

\sum_{i=1}^nix_i=\sum_{1\leq i <j\leq n}(i-j)\alpha(x_i-x_j)+\sum_{i=1}^n\left[i-\frac{\alpha}{2}(i-1)i+\frac{\alpha}{2}(n-i)(n-i+1)\right]x_i

此时再利用Cauchy不等式即可

2.局部放缩

a_i\in R^+ ,证明: \max_{1\leq k \leq n}{a_k\sum_{i=1}^nia_i}\geq\frac{n+1}{n-1}\sum_{1\leq i<j\leq n}a_ia_j
分析:

自然是从复杂的项入手,右侧的形式有很多处理手段,但左侧是两个烦人的乘积,处理手段较为单一。 \sum_{i=1}^nia_i 不好放缩,故考虑 \max_{1\leq k \leq n}a_k ,最简单的便是取平均放缩,但是为了保证对称性,单一的放缩是不合适的,考虑:

\max_{1\leq k \leq n}a_k\geq\frac{\sum _{m=1}^ia_m}{i},\forall i=1,2,...,n

故有 \max_{1\leq k \leq n}{a_k\sum_{i=1}^nia_i}\geq\sum_{i=1}^na_i(a_1+a_2+\cdot\cdot\cdot+a_i)=\sum^n_{i=1}a_i^2+\sum_{1\leq i <j \leq n}a_ia_j

\geq(\frac{2}{n-1}+1)\sum_{1\leq i <j \leq n}a_ia_j\geq\frac{n+1}{n-1}\sum_{1\leq i<j\leq n}a_ia_j

 

二、奇数n的一个小结论
给定 n\in N^+ 为奇数,则 x_1,x_2,...,x_n 中存在相邻两个数相等或三个数成单调子列
证明:

由\prod_{i=1}^{n}(x_i-x_{i+1})(x_{i+1}-x_{i+2})=\prod_{i=1}^{n}(x_i-x_{i+1})\geq 0 ,可知结论成立

应用一:

设 n\geq3 ,a_1,a_2,...,a_n\in R ,证明: \sum_{i=1}^na_i^2-\sum_{i=1}^na_ia_{i+1}\leq\left[ \frac{n}{2} \right](M-m)^2 其中 M=\max_{1\leq i\leq n}a_i,m=\min_{1\leq i\leq n}a_i
分析:

n 为偶数时结论显然,只需考虑n 为奇数的情况

思考一下此时问题不显然的原因,其实是左边相对于右边来说多了一个平方项,没有办法直接放缩得到结论成立

不过要是有相邻两项相等的话就好了,这一项的差距就不再存在

最坏的情况就是没有相等的项,那其实到这里为止提示就相当明显了,就看选手们有没有足够的积累,利用我们的结论,不妨设单调的三项为 a_1,a_2,a_3

那我们的目标就是分析这三项之间的两个平方式,如果能放缩成一个平方式,这道题就做完了

根据这一目标,考虑: (a_1-a_2)^2+(a_2-a_3)^2\leq(a_1-a_3)^2 ,展开后显然成立

应用二:

给定 n\geq4 ,设 x_1,x_2,...,x_n\in[0,1] ,求 \sum_{i=1}^nx_i|x_i-x_{i+1}| 的最大值
分析:

(小小地吐槽一下)怎么说,我竞赛生涯中最讨厌带取整函数的不等式,其次就是带绝对值的不等式。其实前者还好些,有许多固定的套路,但后者真的是非常恶心。既可出成各种诡异的组合风格不等式,又能出成神必积分不等式。本题属于前者。

首先猜取等是必要的,尤其是这种题目

n 为偶数时: 1,0,1,0,...,1,0 , \max=\frac{n}{2}
n 为奇数时: 1,0,1,0,...,1,0,x , x|x-1|\leq\frac{1}{4} , \max=\frac{n}{2}-\frac{1}{4}
这里注意到本题有个很重要的特征,即变量之间没有约束,在这种情况下归纳法是很有用的

假设 n-1 的情况成立,考虑 n 的情况

当 n 为奇数时,必定存在连续三个数单调(相等的情况显然不是最大值)

(i) x_i\geq x_{i+1} \geq x_{i+2}

x_i|x_i-x_{i+1}|+x_{i+1}|x_{i+1}-x_{i+2}|=x_i(x_i-x_{i+1})+x_{i+1}(x_{i+1}-x_{i+2})

\leq x_i(x_i-x_{i+1})+x_{i}(x_{i+1}-x_{i+2})=x_i|x_i-x_{i+2}|

由归纳假设结论成立

(ii) x_i\leq x_{i+1} \leq x_{i+2}

x_i|x_i-x_{i+1}|+x_{i+1}|x_{i+1}-x_{i+2}|=x_i(x_{i+1}-x_{i})+x_{i+1}(x_{i+2}-x_{i+1})

=x_i(x_{i+2}-x_i)+(x_{i+1}-x_i)(x_{i+2}-x_{i+1})\leq x_i|x_i-x_{i+2}|+\frac{1}{4}(x_{i+2}-x_{i})^2

由归纳假设结论成立

当 n 为偶数时,若存在连续三个数单调则结论成立

反之不妨设x_1\geq x_2\leq x_3\geq...\geq x_n\leq x_1 ,此时绝对值就可以拆开啦

原式化为 L=x_1(x_1-x_2)+x_2(x_3-x_2)+...+x_n(x_1-x_n)

L(x_1) 最大值在边界处取到,即 \max L(x_1)=\max\left\{ L(1),L(x_2),L(x_n) \right\}

当 x_1=x_2 或 x_n 时,化为 n-1 的情形,得证

当 x_1=1 时,对 x_3,x_5,...,x_{n-1} 作同样的操作,得到 x_3=x_5=...=x_{n-1}=1

原式 =\frac{n}{2}-(x_2^2+x_4^2+...+x_n^2)\leq\frac{n}{2}

 

三、待定系数法的非常规应用
例一:

设 n\geq2 , a_0,a_1,...a_n\in R ,且 a_1=a_{n-1}=0 ,求证:对任意 t\in R , \sum_{i=0}^{n-2}\bigg|{a_i-ta_{i+1}-a_{i+2}}\bigg|\geq|a_0|-|a_n|
分析:思路是很明确的,将 a_i-ta_{i+1}-a_{i+2} 视作整体,希望这 n-1 个整体相加减之后消去 a_1,a_2,...,a_{n-1}

待定 \lambda_0,\lambda_1,...,\lambda_{n-2} ,使得 \lambda_0(a_0-ta_1-a_2)+\lambda_1(a_1-ta_2-a_3)+...+\lambda_{n-2}(a_{n-2}-ta_{n-1}-a_n) 中

a_1,a_2,...,a_{n-1} 的系数均为0

得到方程组如下

\begin{align*} \begin{split} \left \{ \begin{array}{ll} -\lambda_0-t\lambda _1+\lambda_2=0\\ -\lambda_1-t\lambda _2+\lambda_3=0\\ \cdot\cdot\cdot\\ -\lambda_{n-4}-t\lambda _{n-3}+\lambda_{n-2}=0 \end{array} \right. \end{split} \end{align*}

递推方程为 s^2-ts-1=0 ,其中 \lambda_i=s^i ,为便于放缩,取绝对值小的根

|\lambda_i|\leq1,\lambda_0=1

\sum_{i=0}^{n-2}\bigg|{a_i-ta_{i+1}-a_{i+2}}\bigg|\geq\sum_{i=0}^{n-2}\bigg|\lambda_i({a_i-ta_{i+1}-a_{i+2}})\bigg|

\geq\bigg|\sum_{i=0}^{n-2}\lambda_i({a_i-ta_{i+1}-a_{i+2}})\bigg|=|\lambda_0a_0-\lambda_{n-2}a_n|\geq|a_0|-|a_n|

例二:

设 n\geq3 ,非零实数 x_1,x_2,...,x_n 满足 \frac{x_1}{x_2}+\frac{x_2}{x_3}+...+\frac{x_n}{x_1}=0 ,求证: \left| x_1x_2+x_2x_3+...+x_nx_1\right|\leq\sum_{i=1}^n|x_i|\left(\max_{1\leq i\leq n}|x_i|-\min_{1\leq i\leq n}|x_i|\right)
分析:

0 项的加减乃常见套路,这里进行如此操作后得到

\left| x_1x_2+x_2x_3+...+x_nx_1\right|

=\left| x_1x_2+x_2x_3+...+x_nx_1-\lambda\left( \frac{x_1}{x_2}+\frac{x_2}{x_3}+...+\frac{x_n}{x_1} \right)\right|

=\bigg|x_1(x_2-\frac{\lambda}{x_2})+x_2(x_3-\frac{\lambda}{x_3})+...+x_n(x_1-\frac{\lambda}{x_1})\bigg|

\leq\bigg|x_1(x_2-\frac{\lambda}{x_2})\bigg|+\bigg|x_2(x_3-\frac{\lambda}{x_3})\bigg|+...+\bigg|x_n(x_1-\frac{\lambda}{x_1})\bigg|

而我们有 \bigg|x_i-\frac{Mm}{x_i}\bigg|\leq M-m ,取 \lambda=Mm 即可得到本题的结论

 

预告:下次更新的内容为Carleman不等式与Hardy不等式,冲!!!


编辑于 2021-10-18 20:53
高中数学
数学竞赛
数学
​赞同 175​
​13 条评论
​分享
​喜欢
​收藏
​申请转载

文章被以下专栏收录
⑨の高中数学竞赛笔记
⑨の高中数学竞赛笔记
周更


首发于
⑨の高中数学竞赛笔记

写文章
点击打开undefined的主页
⑨のn元不等式笔记(2)——可能是最全面透彻的Hardy不等式与Carleman不等式讲解
⑨のn元不等式笔记(2)——可能是最全面透彻的Hardy不等式与Carleman不等式讲解
天才琪露诺
天才琪露诺​
琪露诺的算术教室因为双减倒闭了……
​关注她
先来介绍一下著名的Hardy不等式:

对于不全为零的非负实数 x_i 以及p>1 ,我们有 \sum_{k=1}^{n}\left( {\frac{\sum_{i=1}^{k}{x_i}}{k}} \right)^{p}<\left( \frac{p}{p-1} \right)^p\sum^{n}_{k=1}x_{k}^p\\
对这个不等式简单认识一下,它表明了部分和平均值与幂平均之间的联系,直接证明这个命题相当困难,我们先来看一道简单的问题

p=2 时,我们得到一个常见的命题:

对于不全为零的非负实数 x_i ,我们有 \sum_{k=1}^{n}\left( {\frac{\sum_{i=1}^{k}{x_i}}{k}} \right)^{2}<4\sum^{n}_{k=1}x_{k}^2\\
学过数学竞赛的同学都应该做过这道题目,我们来分析一下

由于题目中有很明显的“和的平方”与“平方和”,这提示我们使用Cauchy不等式

然而并不清楚具体的使用方法,此时就需要先待定系数来一探究竟了

对于左式,待定系数后得到 \left( \sum^k_{i=1}x_i \right)^2 \leq\sum_{i=1}^{k}{\frac{x^2_i}{c_i}}\sum_{i=1}^{k}{c_i}\\

这样我们就有

\sum_{k=1}^{n}\left( {\frac{\sum_{i=1}^{k}{x_i}}{k}} \right)^{2}<\sum^n_{k=1}\lambda_{k}a_k^2\\

其中 \lambda_k=\frac{1}{c_k}\left( \frac{c_1+c_2+\cdot\cdot\cdot+c_k}{k^2}+ \frac{c_1+c_2+\cdot\cdot\cdot+c_{k+1}}{(k+1)^2}+\cdot\cdot\cdot+\frac{c_1+c_2+\cdot\cdot\cdot+c_n}{n^2}\right)

下面的目标就是选取合适的 c_k 使得\lambda_k\leq4 ,为了处理复杂的求和,容易想到取c_k为差分形式,即

c_k=k^\alpha -(k-1)^\alpha\\

但如何选取幂又成了另一个问题,我们尝试粗略的分析

\lambda_k=\frac{1}{k^\alpha -(k-1)^\alpha}\left( k^{\alpha-2}+(k+1)^{\alpha-2}+\cdot\cdot\cdot +n^{\alpha-2}\right)

~ \sim\frac{n^{\alpha-1}-k^{\alpha-1}}{k^\alpha -(k-1)^\alpha}\frac{1}{\alpha-1}\sim\frac{n^{\alpha-1}-k^{\alpha-1}}{k^{\alpha-1}}\frac{1}{\alpha-1}\frac{1}{\alpha}

这里其实是将求和看作积分,将差分看作求导,这种从连续角度分析离散问题的方法在待定常数时非常有用

我们继续考虑上式,为了让 \frac{n^{\alpha-1}-k^{\alpha-1}}{k^{\alpha-1}}\frac{1}{\alpha-1}\frac{1}{\alpha}\leq4

一方面需要保证 \alpha-1<0 ,另一方面需要保证 \frac{1}{\alpha(1-\alpha)}\leq4 ,因此 \alpha=\frac{1}{2}

至此本题的分析结束,证明过程只需验证 \lambda_k\leq4 恒成立即可

回到Hardy不等式,我们对 p 的范围进行延拓(注意这里并不严谨,只是揭示不等式之间的关系)

当 p\rightarrow0 时,我们就得到了Carleman不等式:

对于不全为零的非负实数 x_i,我们有 \sum_{k=1}^{n}{\sqrt[k]{\prod_{i=1}^{k}x_i}}<e\sum_{k=1}^{n}{x_k}\\
这个过程和幂平均不等式导出基本不等式链的过程是类似的,取 x_k=\frac{1}{k} 可知 e 是最佳的,不过也可以进一步改进为 e-\frac{2\pi^2e}{\mathrm l\mathrm nn}+O\left( \frac{1}{\mathrm l\mathrm n^3n} \right) ,当然这与我们要讲的无关,下面我们来证明上述命题

对于上式进行分析,容易想到利用均值不等式建立左式与右式的联系,在系数选取上仍然先考虑待定

\sqrt[k]{\prod_{i=1}^{k}x_i}=\frac{1}{\sqrt[k]{\prod_{i=1}^{k}c_i}}\sqrt[k]{\prod_{i=1}^{k}c_ix_i}\leq\frac{1}{k\sqrt[k]{\prod_{i=1}^{k}c_i}}\sum_{i=1}^{k}{c_ix_i}\\

于是我们有

\sum_{k=1}^{n}{\sqrt[k]{\prod_{i=1}^{k}x_i}}\leq\sum_{i=1}^{n}\left(c_i\sum_{k=i}^n\frac{1}{k\sqrt[k]{\prod_{i=1}^{k}c_i}} \right){x_i}\\

与上述分析异曲同工,为了使系数可以计算,我们凑成可以裂项的形式,一个自然的想法是令

\sqrt[k]{\prod_{i=1}^{k}c_i}=k+1\Leftrightarrow c_k=\frac{(k+1)^k}{k^{k-1}}\\

于是

c_i\sum_{k=i}^n\frac{1}{k\sqrt[k]{\prod_{i=1}^{k}c_i}}=\frac{(i+1)^i}{i^{i-1}}\left(\frac{1}{i}-\frac{1}{n+1} \right)<\left( 1+\frac{1}{i} \right)^i<e\\

这样就完成了本题的证明

下面我们来利用Carleman不等式解决王永喜的一道难题

设 x_i\geq0 (i=1,2,...,n-1) ,有 \sum_{k=1}^{n-1}{x_k^{\frac{6}{5}}}=\frac{2}{n(n-1)} ,记 \sum_{k=1}^{n-1}{x_i}=S , x_0=0 ,证明: \sum_{k=1}^{n-1}{\left( \prod_{l=1}^{k}\left( S-\sum_{i=0}^{l-1}{x_i}\right) \right)^{\frac{2}{k}}}<\frac{2e}{n(n-1)}\\
分析:首先看到条件中莫名其妙的系数(以及wyx的出题风格),大概就能猜到这是一道凑的题目了,首先可以使用一次Carleman不等式凑出系数 e ,并将难以处理的乘积式转化为求和式,后续就仅剩下对不同幂和之间的估计了

\sum_{k=1}^{n-1}\left( \prod_{l=1}^{k}\left( S-\sum_{i=0}^{l-1}{x_i}\right) \right)^{\frac{2}{k}}<e\sum_{l=1}^{n-1}\left( S-\sum_{i=0}^{l-1}{x_i} \right)^2\\

故只需证

\sum_{l=1}^{n-1}\left( S-\sum_{i=0}^{l-1}{x_i} \right)^2\leq\frac{2}{n(n-1)}\\

直接展开处理并使用Holder不等式

LHS=\sum_{k=1}^{n-1}{kx_k^2}+2\sum_{1\leq k<l\leq n-1}{kx_kx_l}<\sum_{k=1}^{n-1}{kx_k^2}+2\sum_{1\leq k<l\leq n-1}{\sqrt{kl} x_kx_l}

=\left( \sum_{k=1}^{n-1}\sqrt{k}x_k \right)^2\leq\left( \left(\sum_{k=1}^{l-1}\left( \sqrt{k} \right)^6 \right) ^{\frac{1}{6}}\right)^2 \left( \left( \sum_{k=1}^{n-1} x_k ^{\frac{6}{5}}\right) ^{\frac{5}{6}} \right)^2=\frac{2}{n(n-1)}

至此本题证毕

不过还没有结束,wyx似乎非常喜欢Carleman不等式,他在某年的中数模拟题上对其作了如下推广:

设 x_i\geq0 (i=1,2,...,n) ,记 M_k=\max_{1\leq i\leq k}\sqrt{ix_i},m_k=\min_{1\leq i\leq k}\sqrt{ix_i} ,求证: \frac{1}{e}\sum_{k=1}^{n}{\sqrt[k]{\prod_{i=1}^{k}x_i}}+\sum_{k=1}^{n}{\frac{(M_k-m_k)^2}{k(k+1)}}<\sum_{k=1}^{n}{x_k}\\
这个加强的方式其实本质上是均值不等式的加强,原本的均值不等式取等条件是所有变量相等,那么有没有可能改进为只让其中部分变量相等呢?显然是可以的,而且通过取等条件就可以看出这是一种加强形式的均值不等式:

\sqrt[n]{\prod_{k=1}^{n}x_i}\leq\frac{2\sqrt{x_1x_2}+x_3+\cdot\cdot\cdot+x_n}{n}=\frac{\sum_{i=1}^{n}{x_i}}{n}-\frac{(\sqrt{x_1}-\sqrt{x_2})^2}{n}\\

基于这种思路,我们对证明Carleman不等式的第一步进行改进,有

\sqrt[k]{\prod_{i=1}^{k}x_i}=\frac{1}{\sqrt[k]{\prod_{i=1}^{k}i}}\sqrt[k]{\prod_{i=1}^{k}ix_i}\leq\frac{1}{k\sqrt[k]{\prod_{i=1}^{k}i}}\left(\sum_{i=1}^{k} {ix_i}-\left( M_k-m_k \right)^2 \right)\\

于是

\sum_{k=1}^{n}\frac{\sqrt[k]{k!}}{k+1}\sqrt[k]{\prod_{i=1}^{k}x_i}+\sum_{k=1}^{n}\frac{\left( M_k-m_k \right)^2}{k(k+1)}\leq\sum_{k=1}^{n}\frac{\sum_{i=1}^{k}{ix_i}}{k(k+1)}\\

RHS=\sum_{k=1}^{n}{kx_k}\sum_{i=k}^{n}{\frac{1}{i(i+1)}}=\sum_{k=1}^{n}{\left( 1-\frac{k}{n+1} \right)x_k<\sum_{k=1}^{n}{x_k}}\\

而 \frac{\sqrt[k]{k!}}{k+1}>\frac{1}{e} 是显然的,故本题证毕

看上去这道题目很简短,原因在于使用均值不等式时我们直接写出了系数,这要归功于我们在证明Carleman不等式时的探索,不过,细心的你一定发现了,为什么这里的系数与我们当初的选择不一样呢?有三点原因:

(1)Carleman不等式的取等条件为 x_k=\frac{1}{k} ,倒不如说这里我们选取的系数是最优的

(2)对于Carleman不等式,我们的目标只是为了凑出可以裂项的形式,而本题的切入点在于 M_k-m_k 的构造中已经暗示了系数 k 的选取,具体题目应该具体分析

(3)系数的选取通常并不唯一,但值得一提的是,它们的阶应该是相同的( \frac{(k+1)^k}{k^{k-1}}\sim ek )

下面回到Hardy不等式,同样对 p 作延拓处理,若 p\rightarrow-1 ,就得到了如下熟知结论:

设 x_i\geq0 (i=1,2,...,n) ,则有: \sum_{k=1}^{n}{\frac{k}{x_1+x_2+\cdot\cdot\cdot+x_k}}<2\sum_{k=1}^{n}{\frac{1}{x_k}}\\
方法类似,我们略去待定系数的过程直接给出证明,由Cauchy不等式

\sum_{i=1}^{k}{ \frac{i^2}{x_i}}\sum_{i=1}^{k}{x_i}\ge\frac{k^2(k+1)^2}{4}\\

于是有

LHS\leq\sum_{k=1}^{n}\frac{4}{k(k+1)^2}\sum_{i=1}^{k}{\frac{i^2}{x_i}=\sum_{i=1}^{n}{\frac{1}{x_i}}\sum_{k=i}^{n}{\frac{4i^2}{k(k+1)^2}}}\\

故只需证 \sum_{k=i}^{n}{\frac{4i^2}{k(k+1)^2}}<2 ,即 \sum_{k=i}^{n}{\frac{1}{k(k+1)^2}}<\frac{1}{2i^2}

对于这种中间命题,首先要检查命题的正确性,从量级的角度考虑,左右确实等价,这样我们再开始证明

自然的想法是将左侧求和(相当于积分)放缩成高一阶的差分(相当于微分),即

\frac{1}{k(k+1)^2}<\frac{1}{2}\left( \frac{1}{k^2}-\frac{1}{(k+1)^2} \right)\\

而上式显然成立,求和之后即得待证结论

事实上,对于这个命题,我们甚至可以给出加权形式:

设 a_k>0,b_k>0 ,证明: \sum_{k=1}^{n}{\frac{a_k(a_1+a_2+\cdot\cdot\cdot+a_k)}{b_1+b_2+\cdot\cdot\cdot+b_k}}<2\sum_{k=1}^{n}{\frac{a^2_k}{b_k}}
这是一道非常困难的题目,对于这个问题,待定系数法显然失效了,原因在于系数的参数化表达导致巨量的计算,而上一讲提到过,对于此类无约束多变量不等式我们可以考虑数学归纳法

但问题在于,直接归纳是不可行的,变量的地位会在n-1\rightarrow n时发生改变

为此,我们使用加强归纳的方法,我们考虑由于归纳递推新引入的末尾项

注意在从 n-1\rightarrow n 时,出现问题的是左端有 \sum a_k、\sum b_k ,而右端只有单独的一项,因此需要引入含有求和的项,另一方面,需要保证变量和不等式的齐次性,需要引入的项必为如下形式

M\frac{(\sum a_k)^2}{\sum b_k}\\

尝试后发现, M=\frac{1}{2} 恰好能使结论成立,下面的工作就是无聊的验证,没有任何技巧

这里,我们给出另一种加强归纳的思路,不同于上述考虑尾项,我们想考虑首项,这要用到一些归纳技巧

n-1\rightarrow n的递推会使尾项单独留下,那我们有没有可能不让n-1\rightarrow n而实现递推呢?其实上面在讲均值不等式的加强形式已经提到了类似的思路,若是将某两项视为一个整体,这样变量的个数就减少了,就可以在总项数不变的前提下使用归纳假设进行证明,这种对变量个数归纳而非对变量维度归纳的技巧是非常强大的

此时若考虑归纳加强命题,则应对右侧(对左侧也一样,无非一加一减)的 \frac{a_1}{b_1} 进行改进,将这一项减小为 m\frac{a_1}{b_1} ,经过后文的尝试后发现 m=\frac{3}{2} 恰可使命题成立,下面来给出完整的证明过程:

我们归纳证明加强后的命题:

设 a_k>0,b_k>0 ,证明: \sum_{k=1}^{n}{\frac{a_k(a_1+a_2+\cdot\cdot\cdot+a_k)}{b_1+b_2+\cdot\cdot\cdot+b_k}}<\frac{3}{2}\frac{a_1}{b_1}+2\sum_{k=2}^{n}{\frac{a^2_k}{b_k}}

假设 n-1 时结论成立,考虑 (a_1+a_2,a_3,...,a_n,b_1+b_2,b_3,...,b_n)

\frac{(a_1+a_2)^2}{b_1+b_2}+\sum_{k=3}^{n}{{\frac{a_k(a_1+a_2+\cdot\cdot\cdot+a_k)}{b_1+b_2+\cdot\cdot\cdot+b_k}}}\leq\frac{3}{2}\frac{(a_1+a_2)^2}{b_1+b_2}+2\sum_{k=2}^{n}{\frac{a_k^2}{b_k}}

考虑 n 时,只需证

2\sum^n_{k=3}\frac{a_k^2}{b_k}+\frac{3}{2}\frac{(a_1+a_2)^2}{b_1+b_2}-\frac{(a_1+a_2)^2}{b_1+b_2}+\frac{a_1^2}{b_1}+\frac{a_2(a_1+a_2)}{b_1+b_2}\leq\frac{3}{2}\frac{a_1^2}{b_1}+2\sum^n_{k=2}\frac{a_k^2}{b_k}

这等价于

\frac{1}{2}\frac{(a_1+a_2)^2}{b_1+b_2}+\frac{a_2(a_1+a_2)}{b_1+b_2}\leq\frac{1}{2}\frac{a_1^2}{b_1}+2\frac{a_2^2}{b_2}\\

由Cauchy不等式立得结论

下面我们来看看一般形式的Hardy不等式如何证明

通过上文对于特殊情况的Hardy不等式的证明可以见得其思路,本质上是构造了一个局部不等式,然后裂项相消得到结论,问题的关键在于局部不等式如何构造

事实上,在上文的证明过程中,可以发现待定的每个系数都小于右侧的系数,这使我们想到,可否对下式进行估计?

\Delta=\left( {\frac{\sum_{i=1}^{k}{x_i}}{k}} \right)^{p}-\left( \frac{p}{p-1} \right)^px_{k}^p\\

答案是肯定的,为此,我们先开方化简上式,考虑

\Delta_1= {\frac{\sum_{i=1}^{k}{x_i}}{k}} -\frac{p}{p-1} x_{k}\\

但是我们要讨论的是 p 次幂( L_p 范数),上面这个东西显然不够看的,所以我们对其乘一项,使整个式子的幂次提升到p

\Delta_2=\left( {\frac{\sum_{i=1}^{k}{x_i}}{k}} \right)^{p}-\frac{p}{p-1}\left( {\frac{\sum_{i=1}^{k}{x_i}}{k}} \right)^{p-1}x_{k}\\

对这一式子进行估计,我们的目标是将其放缩为差分形式

注意到若令 A_k=\sum_{i=1}^{k}{x_i} ,则 x_k=A_k-A_{k-1} ,我们的目的是找到 g(k) 使得下式成立

\Delta_2<g(k)A_k^p-g(k-1)a_{k-1}^p\\

待定 g(k) ,利用前文讲过的技巧,我们来分析 g(k) 的阶,对上式整理后得到

1-\frac{pk}{p-1}(1-t)\leq(g(k)-g(k-1)t^p)k^p\\

其中 t=\frac{A_{k-1}}{A_k}\leq1

设 f(t)=(g(k)-g(k-1)t^p)k^p+\frac{pk}{p-1}(1-t)-1

考虑 t\rightarrow1 ,应有 f\rightarrow0 ,也即

(g(k)-g(k-1))\sim k^{-p}\\

这说明 g'(k)\sim k^{-p} ,于是有 g(k)\sim k^{-p+1}\frac{1}{1-p}

下面只需证明 \Delta_2<\frac{1}{p-1}\left( \frac{A_{k-1}}{(k-1)^{p}} -\frac{A_{k}}{k^{p}}\right) ,这可由对 f(t) 求导得到,或者使用Young不等式:

\Delta_2=\left( \frac{A_k}{k}\right)^p\left( 1-\frac{kp}{p-1} \right)+\frac{(k-1)p}{p-1}\left( \left( \frac{A_k}{k} \right)^p \right)^{\frac{p-1}{p}}\left( \left( \frac{A_{k-1}}{k-1} \right)^p \right)^{\frac{1}{p}}

\leq\left( \frac{A_{k-1}}{k-1} \right)^p+\frac{(k-1)p}{p-1}\left( \frac{p-1}{p}\frac{A_k}{k}+\frac{1}{p}\frac{A_{k-1}}{k-1} \right)=\frac{1}{p-1}\left( \frac{A_{k-1}}{(k-1)^{p}} -\frac{A_{k}}{k^{p}}\right)

最后再求和,得到

\sum_{k=1}^{n}\left( {\frac{\sum_{i=1}^{k}{x_i}}{k}} \right)^{p}-\left( \frac{p}{p-1} \right)^p\sum^{n}_{k=1}x_{k}^p=\sum \Delta_2\leq-\frac{n}{p-1}\left( \frac{A_n}{n} \right)^n<0

于是我们完成了对一般情况下Hardy不等式的证明

最后来看一道习题

设x_i\geq0 (i=1,2,...,n-1) ,证明: \sum_{i,j}^{n}{\frac{x_ix_j}{\max{\left\{ i,j \right\}}}}\leq5\sum^n_{i=1}x^2_i
解答:使用 p=2 时的Hardy不等式,有

LHS=\frac{1}{n}\left( \sum_{i=1}^n x _i \right)^2+\frac{1}{n(n-1)}\left( \sum_{i=1}^{n-1} x _i\right)^2+\frac{1}{(n-1)(n-2)}\left( \sum_{i=1}^{n-2} x _i\right)^2+\cdot\cdot\cdot+\frac{1}{2}x_1^2

\leq\frac{1}{n}\left( \sum_{i=1}^n x _i \right)^2+\left( \frac{\sum_{i=1}^{n-1} x _i}{n-1}\right)^2+\left( \frac{\sum_{i=1}^{n-2} x _i}{n-2}\right)^2+\cdot\cdot\cdot+\left( \frac{x_1}{1} \right)^2

\leq\sum_{i=1}^nx^2_i+4\sum_{i=1}^nx^2_i=5\sum_{i=1}^nx^2_i

评注:事实上本题的最佳系数为 4 ,证明过程与 P=2 时的Hardy不等式类似,留作练习

 

 

终于考完期末了!淑芬竟然拿了100。。。

这个假期就把坑都给填上

发布于 2022-01-06 17:17
数学竞赛
高中数学
不等式
​赞同 192​
​16 条评论
​分享
​喜欢
​收藏
​申请转载

文章被以下专栏收录
⑨の高中数学竞赛笔记
⑨の高中数学竞赛笔记
周更

 

\item
\nandu{5}
\tiyuan{拉格朗日乘数法}
\tigan{拉格朗日乘数法是一种寻找变量受一个或多个条件所限制的多元函数的极值的方法.要找函数$z=f(x,y)$在条件$g(x,y)=0$下的可能极值点,先构造函数$F(x,y)=f(x,y)+\lambda g(x,y)$,
其中$\lambda$为拉格朗日乘数.

$\begin{cases}f_x(x,y)+\lambda g_x(x,y)=0,\\
f_y(x,y)+\lambda g_y(x,y)=0,\\
g(x,y)=0.\end{cases}$
解出$x,y,\lambda$,其中$(x,y)$就是可能的极值点的坐标.这里$f_x(x,y),f_y(x,y)$分别表示函数$f$对变量$x,y$的偏导数,要求出$f_x(x,y)$,只要在求导时将变量$y$看成常数即可,例如:若$f(x,y)=x^2\ln y$,则$f_x(x,y)=2x\ln y,f_y(x,y)=\frac{x^2}{y}$.

\textbf{注:}在以下问题中,极值点可当作最值点.

(1)已知$f(x,y)=x^2y^2$,求$f_x(x,y)$和$f_y(x,y)$;

(2)求$f(x,y)=x+y$在条件$x^2+2y^2+xy=1$下的最大值和最小值;

(3)已知椭圆的面积为$S=\pi ab$,其中$a,b$分别为椭圆的半长轴,半短轴的长.在平面直角坐标系$xOy$中,椭圆$C:x^2+2y^2+2xy=1$的中心为原点,求$C$的面积.
}
\daan{ }
\jiexi{(1)由题意知$f_x(x,y)=2xy^2$, $f_y(x,y)=2x^2y$;

(2) 构造函数$F(x,y)=x+y+\lambda (x^2+2y^2+xy-1)$,
由$\begin{cases}
1+2\lambda x+\lambda y=0,\\
1+4\lambda y+\lambda x=0,\\
x^2+2y^2+xy-1=0.\end{cases}$
解得$x=\frac{3\sqrt{14}}{14},y=\frac{\sqrt{14}}{14}$
或$x=-\frac{3\sqrt{14}}{14},y=-\frac{\sqrt{14}}{14}$.

则当$x=\frac{3\sqrt{14}}{14},y=\frac{\sqrt{14}}{14}$时,
$f(x,y)=x+y$取得最大值$\frac{2\sqrt{14}}{7}$;

当$x=-\frac{3\sqrt{14}}{14},y=-\frac{\sqrt{14}}{14}$时,
$f(x,y)=x+y$取得最大值$-\frac{2\sqrt{14}}{7}$.


(3)因为椭圆$C:x^2+2y^2+2xy=1$的中心为原点,则椭圆上一点$(x,y)$到原点$(0,0)$距离$\sqrt{x^2+y^2}$的最大值为椭圆的半长轴的长$a$,距离的最小值为椭圆的半短轴的长$b$.

构造函数$G(x,y)=x^2+y^2+\lambda (x^2+2y^2+2xy-1)$,
由$\begin{cases}
2x+2\lambda x+2\lambda y=0,\\
2y+4\lambda y+2\lambda x=0,\\
x^2+2y^2+2xy-1=0.\end{cases}$
解得$x^2=1-\frac{2\sqrt{5}}{5},y^2=\frac{1}{2}-\frac{\sqrt{5}}{10}$
或$x^2=1+\frac{2\sqrt{5}}{5},y^2=\frac{1}{2}+\frac{\sqrt{5}}{10}$.

当$x^2=1+\frac{2\sqrt{5}} {5},y^2=\frac{1}{2}+\frac{\sqrt{5}}{10}$时,
所求椭圆的半长轴的长的平方为$a^2=x^2+y^2=\frac{3}{2}+\frac{\sqrt{5}}{2}$;

当$x^2=1-\frac{2\sqrt{5}} {5},y^2=\frac{1}{2}-\frac{\sqrt{5}}{10}$时,
所求椭圆的半短轴的长的平方为$b^2=x^2+y^2=\frac{3}{2}-\frac{\sqrt{5}}{2}$.

因此$C$的面积为$S=\pi ab=\pi \sqrt{\left( \frac{3}{2}-\frac{\sqrt{5}}{2} \right) \left( \frac{3}{2}+\frac{\sqrt{5}}{2} \right)}=\pi$.
}
\end{QsNum}


2023年华中科技大学“启明班”选拔考试(综合部分)


考试日期: 2023年9月2日.

第一部分:优化与建模

一、(20分) 封闭曲线$A$中有一条长为1的弦,弦上一点$P$将其分为两段,长度分别为$x$,曲线$B$围成的面积之差为$S(x)$.由??原理可知, $S(x)$与曲线$A$, $B$形状无关,仅与$P$点位置(即$x$的大小)有关. 求:

(1) $S(x)$的最大值;

以及

(2)当$S(x)$取最大值时, $x$的取值.


二、(20分) 一商店老板的称量天平损坏了,其左臂比右臂长了$10\%\sim 20\%$,一客人前来购
买2 kg的物品,老板以左物右码分别称量了1 kg物品,问:

(1) 最终是老板还是客人的利益损失?

(2) 损失比的取值范围是什么?

第二部分:信息熵

三、(40分) 信息熵(information entropy)是信息论的基本概念,用于描述信息源各可能事件发生的不确定性. 20世纪40年代,香农(C.E.Shannon) 借鉴了热力学的概念,把信息中排除了冗余后的平均信息量称为“信息熵”,并给出了计算信息熵的数学表达式. 信息熵的提出解决了对信息的量化度量问题. 给出信息熵定义式
$$
H(x)=\sum_{i=1}^n-p(x_i)\log_ap(x_i),
$$
其中$p(x_i)>0$,并且$p(x_1)+p(x_2)+\cdots+p(x_n)=1$.当$n=2$时,我们可以把上式改写为
$$
H(p,q)=-p\log_ap-q\log_aq,
$$
其中$p>0$, $q>0$, $p+q=1$.容易看出,当$p=q=0.5$时,上式取极值.

(1)比特(BIT, Binary digit)是计算机专业术语,由英文BIT音译而来.比特是二进制数字中的位,是度量信息量的最小单位.例如,抛硬币时,正面与反面朝上的概率均为$\frac12$,根据信息熵定义式可算出信息量为1 bit,那么此时的底数$a$为多少?

(2)有人提出,根据统计知识,对于相互独立的两个随机变量$X$, $Y$,应有
$$
H(X+Y)=H(X)+H(Y).
$$
假设$X$和$Y$均服从参数为$p$的两点分布,由高中知识我们知道, $Z=X+Y$服从二项分布$B(2,p)$.问: $H(Z)=2H(X)$是否正确?并说明理由.

(3)用Jensen不等式,证明$H(X)$有最大值.提示:当$f(x)$为凸函数时,有Jensen不等式
$$
f(a_1x_1+\cdots+a_nx_n)\leqslant a_1f(x_1)+\cdots+a_nf(x_n),
$$
其中, $\sum_{k=1}^na_k=1$.

(4)现有2023个小球(标号为$1,2,\cdots,2023$)和一个足够大的称量天平,有一个劣质小球比其他小球轻一些,问:至少要称量多少次才能找到这个小球?

第三部分:产量分配

四、(20分) 某公司拟使用$t$万元采购原料,原料的定价分别为$p_1,p_2,\cdots,p_n$,采购原料的总
量为$x_1,x_2,\cdots,x_n$.生产产品的总量为$\min\{q_1x_1,q_2x_2,\cdots,q_nx_n\}$.以尽量扩大生产为前提:

(1) 该公司应以什么策略进行购物?

(2)若预算翻倍,哪一种原料增加购买的数量最多?

2023年南京大学“飞迪计划”二次选拔考试题(数学)

第一题 (15分)

证明:任给7个实数,其中必定存在两个实数$x,y$,满足
$$
0<\frac{x-y}{1+xy}<\frac{\sqrt{3}}3.
$$

第二题(15分)

已知平面上有10个圆,任意两个圆都相交,是否存在直线$l$,与每个圆都有公共点?证明你的结论.


第三题(15分)

已知$a$, $b$, $c$为正实数,且三次方程$x^3-ax^2+bx-c=0$有三个实根,证明:三次方程的三个实根为某三角形三边长的充要条件是: $-a^3+4ab-8c>0$.

第四题(15分)

正整数$n$的“重复表示” 是把这个正整数$n$重复写两遍,比如: 2023的“重复表示”是20232023.

问:是否存在正整数$n$,它的“重复表示”是完全平方数?如果是,请给出尽可能多的这样的正整数$n$;如果不是,请说明理由.

第五题(40分)

定义在区间$I\subset\mathbb{R}$上的函数$f:I\to\mathbb{R}$的Legendre变换是函数
$$
f^\ast(t)=\sup_{x\in I}(tx-f(x)).
$$
证明:

(1)使得$f^\ast(t)\in\mathbb{R}$(即$f^\ast(t)\neq\infty$)的值$t\in\mathbb{R}$构成的集合$I^\ast$可能是空集、单点集或者区间,而如果$I^\ast$是区间,则函数$f^\ast(t)$在$I^\ast$上是凸函数.

(2)若$f$是凸函数,则$I^\ast\neq\varnothing$且当$f^\ast\in C^0(I^\ast)$时,
$$
(f^\ast)^\ast(x)=\sup_{t\in I^\ast}(xt-f^\ast(t))=f(x).
$$
对任意的$x\in I$成立.也就是说,凸函数的Legendre变换是对合变换.(一个变换$\mathcal{F}$是对台变换指的是$\mathcal{F}^2=I$,其中$I$表示恒等变换.)

(3)当$x\in I,t\in I^\ast$时,有如下Young不等式的推广版本:
$$
xt\leq f(x)+f^\ast(t).
$$

(4)当$f$是凸的可微函数时, $f^\ast(t)=tx_t-f(x_t)$,其中$x_t$由方程
$$
t=f'(x)
$$
确定;给出Legendre变换$f^\ast$及其自变量$t$的几何解释.

(5)当$\alpha>1$且$x\geq0$时,函数$f(x)=\frac1px^p$的Legendre变换是函数$f^*(t)=\frac1qt^q$,其中$t\geqslant 0$且$p^{-1}+q^{-1}=1.$ 并证明Young不等式

$$
xt\leqslant\frac1px^p+\frac1qx^q.
$$

(6)函数$f(x)=e^x$的Legendre变换是函数$f^*(t)=t\ln\frac te,t>0$,并且满足不等式
$$
xt\leqslant e^x+t\ln\frac te,\quad \forall x\in\mathbb{R},t>0.
$$

经济学拔尖班考试范围:数学.

考试时间:2023年9月5日9:00 $\sim$ 11:00.

2023年南京大学经济学拔尖班二次选拔考试

第一题 (35分) 对任何$s''>s'$, $f(s')\geqslant 0\Rightarrow f(s'')\geqslant 0$成立时,我们称函数$f:\mathbb{R}\to\mathbb{R}$满足单交性质.

对任何$x''>x'$, $\delta(s)=g(x'',s)-g(x',s)$满足单交性质时,我们称函数簇$\{g(\cdot,s)\}_{s\in S}$ 满足单交差性质.

考虑函数
$$
F\left( v_A,u \right) =\frac{\frac{u}{\sqrt{v_H}}+\frac{1-u}{\sqrt{v_L}}}{\frac{1}{v_A}+\frac{u}{v_H}+\frac{1-u}{v_L}}\left( u\sqrt{v_H}+\left( 1-u \right) \sqrt{v_L} \right),
$$
其中所有参数均为实数.

问:当$v_A>0$, $v_H\geqslant v_L>0$时,函数簇$\{F(\cdot,u)\}_{u\in[0,1]}$是否满足单交差性质?为什么?

第二题 (35分)

考虑二维空间中的函数$d:\mathbb{R}\times\mathbb{R}\to\mathbb{R}$,当该函数对任意选取的$x,y,z$都满足下面的条件(1)-(4)时,我们称$d$为二维欧氏空间中的度量函数.

(1) $d(x,y)\geqslant 0$;

(2) $d(x,y)=d(y,x)$;

(3) $d(x,y)=0$当且仅当$x=y$;

(4) $d(x,z)\leqslant d(x,y)+d(y,z)$.

注: $x,y,z$均为二维空间中的点,例如$x=(x_1,x_2)$,其中$x_1\in\mathbb{R}$,$x_2\in\mathbb{R}$.

考虑以下四种度量函数:
\begin{align*}
d_{1}(x,y) &=|x_1-y_1|+|x_2-y_2|, \\
d_{2}(x,y) &=\sqrt{(x_1-y_1)^2+(x_2-y_2)^2}, \\
d_3(x,y) &=|x_1-y_1|, \\
d_4(x,y) & =\begin{cases}0,&\text{当}x=y,\\
1, &\text{当}x\neq y,
\end{cases}
\end{align*}
问:其中哪些是二维空间中的度量函数?哪些不是?为什么?

第三题 (30分)

考虑以下两个优化问题:

问题(i): $\max_{(x_1,x_2)\in X_1\times X_2}f(x_1,x_2)$;

问:上述两个优化问题的最优解是什么关系? (提示:问题(i)的最优解是问题(i)的最优解吗?问题(ii)的最优解是问题(i)的最优解吗?)


阿诺尔德给5至15岁孩子出的数学题

\begin{center}
\textbf{摘要}
\end{center}

 


这个小册子由作者挑选或者编写的79道数学题, 目的是为了思维文化的发展.大部分题目不需要普通教育外的特殊知识.然而, 其中的一些题对教授也是个挑战.

这本书是针对中学生、大学生、教师和父母, 所有认为思维文化训练是个人发展基本要素的人们.

 

\begin{center}
\textbf{作者序}
\end{center}


2004年春, 一些在巴黎居住的俄罗斯人要求我帮助他们的年轻孩子以传统俄罗斯的思维发展训练.


我深信, 这个文化最早是通过早期独立思考简单但不容易的问题来培养的 (我最推荐问题1、3、13).

我的长期经验表明, 在校学习迟钝的C级学生可以比优秀学生更好地解决这些问题, 因为他们在课堂后面的智力“堪察加”的生存”要求比管理帝国所需要的更多的能力”, 正如费加罗在博马舍的戏中所说的那样.另一方面, A级学生在这些问题上无法弄清楚”什么东西要乘以什么".我甚至注意到, 五岁的孩子比那些被学校训练摧残的学龄儿童更能解决这样的问题, 而相应地, 这些学龄前儿童又比那些忙于死记硬背的大学生们更容易找到解答.(诺贝尔奖或菲尔兹奖得主在解决这些问题上是最糟糕的.)

 

\begin{center}
\textbf{问题}
\end{center}

1.玛莎(Masha)身上的钱买一本字母书差7戈比, 美莎(Misha)身上的钱买这本书差1戈比.她们把身上的钱合起来买这本书还是不够.问这本字母书多少钱? (译者注:戈比是俄罗斯最小的货币单位)

设玛莎身上有$x$元戈比,美莎身上有$y$元戈比,则$x+7=y+1>x+y$,
则$x+7>x+(x+6)$,故$x<1$,故$x=0$.于是$y=6$,这本字母书需要$7$戈比.

 

 

 

2.一个带有软木塞的瓶子售价为1.1美元,而瓶子本身比软木塞高出10美分.问软木塞值多少钱?

 

3.一块砖的重量是一磅加半块砖的重量.问这块砖重多少磅?

 

4.从一桶葡萄酒中取出一勺葡萄酒放入一杯茶(未满)里. 之后, 将等量的一勺玻璃杯混合液体倒入葡萄酒桶中.此时, 每个容器里都有一定量的“外来”液体(玻璃杯里的酒和桶里的茶). 问哪一种外来液体的体积更大:桶里的茶还是玻璃杯中的酒?

 

5.两名老年妇女在黎明时离开, 一名从A到B, 另一名从B到A. 她们(沿着同一条路)相向而行. 她们在中午见面, 但并没有停下来, 并且她们每个人都以以前一样的速度继续前行. 第一位女士在下午4点抵达B, 第二位女士晚上9点抵达A. 问她们是当天黎明几点出发的?

 

6.(在一个美国标准的测验中)一个直角三角形的斜边是10英寸,此斜边上的高是6英寸.求此直角三角形的面积.


过去十年, 美国高中生都能成功的解答这道题.而一些从莫斯科来的俄罗斯学生却不能得到他们美国同伴的答案(30平方英寸).这是为什么呢?

 

7.维克多 (Victor) 的姐妹比他的兄弟多2个.问维克多的父母的女儿比儿子多几个?

 

8.南美洲有一个圆形的湖泊. 每年的6月1日, 一朵王莲花 (Victoria Regia flower) 出现在它的中心. (它的茎从湖底部升起, 它的花瓣像睡莲一样躺在水面上). 每天花的面积加倍, 至7月1日, 它终于覆盖整个湖泊, 然后花瓣落下, 其种子下沉. 问几月几号时, 花的面积是湖泊面积的一半?

 

9.一个农夫必须把一只狼, 一只山羊和一棵白菜运过河. 但是这艘船太小了, 他每次只能带这三个中的一个过河. 问他怎样才能把这三个都运过河去? (狼不能和山羊单独呆在一起, 山羊不能和白菜单独呆在一起.)

 

10.白天, 蜗牛在一根柱子上向上爬了3厘米. 在夜间, 它睡着了, 向下滑了2厘米. 这根柱子有10米高, 一个美味的甜点正在柱子顶端等待蜗牛. 问蜗牛要花多少天才能品尝到甜点?

 

11.一个猎人离开他的帐篷向南走了10公里, 然后向东直行, 走了10公里, 射杀了一头熊, 然后转身向北走了10公里后发现了自己的帐篷.问熊是什么颜色?这是在哪里发生的?

 

12.今天中午十二时发生满潮.(在同一地点)满潮明天几点发生?

 

13.两卷普希金的书, 第一卷和第二卷, 并排放在书架上. 每卷书的页面总厚度(不包含封面和封底)为2厘米, 封面和封底各有2毫米厚. 书虫从第一卷第一页啃到第二卷的最后一页(垂直于页面啃咬).问书虫的轨迹有多长?[ 这个拓扑问题有个令人难以置信的答案 — 4 mm — 这对于院士来说是完全不可能的, 但是一些学龄前儿童可以轻松应对.]

 


作者注:在提出这个问题的时候, 我试图在2000年“物理学进展”杂志百周年期刊的邀请论文中说明数学家和物理学家在研究方法上的差异. 我的成功远远超过了我所想到的目标:我的设计是基于学龄前儿童的经验, 与编辑们的经验不同, 所以编辑们不能解决这个问题. 所以他们为了得到答案4毫米把题目改成了下面的方式:他们改成了“从第一卷的最后一页到第二卷的第一页”, 而不是“从第一卷的第一页到第二卷的最后一页”.

 

这个真实的故事是如此令人难以置信, 所以我要把这个题目写进来:证据是杂志发表的编辑版本.


14.从上面和从前面看, 某个物体(多面体)的形状如下. 画出从侧面看它的形状. (多面体的隐藏边用虚线显示.)

图片
俯视

 

图片
前视

 

15.有多少种方法将数字64分成十个自然数之和, 其中每个自然数不超过12? 只有加数顺序不同的和不能算作不同的和.

 

16.我们有一些质地相同的块(比如, 多米诺骨牌).我们把这些块堆放起来, 使得最上面的块比最底下的块移出$x$长度.问: $x$最大可能是多少?

图片


17. A镇和B镇之间的距离是40公里. 两名骑自行车的人从A和B同时离开相向而行, 一个以10 km /h的速度行驶, 另一个以15 km / h的速度行驶. 一只苍蝇与第一个骑手一起离开A, 以100公里/小时的速度飞向第二个骑手. 苍蝇遇到第二个骑手时, 触碰到他的额头, 然后飞回到第一个骑手, 碰到他的额头, 再返回到第二个骑手, 一直这样下去, 直到两个骑手的额头碰撞并压扁苍蝇. 问:苍蝇一共飞行了多少公里呢?

图片


18.瓦尼亚(Vanya)解决了一个关于两个学龄前(preschool)儿童的问题. 在给定两个孩子年龄乘积的情况下, 瓦尼亚必须找出他们的年龄(这是整数).

瓦尼亚说这个问题不能解决. 老师称赞他说得对, 然后给这个了问题增加了条件:大孩子的名字是佩蒂娅(Petya). 这时瓦尼亚可以马上解决这个问题. 现在请你解答这个题.(译者注:美国的学龄前(preschool)指的是不超过5岁的孩子.)

 


19.整数140\ 359\ 156\ 002\ 848是否能被整数 4\ 206\ 377\ 084 整除?

 

20.一块多米诺骨牌可以覆盖棋盘的两个方格.请用31块多米诺骨牌盖满一个除去左上和右下方格(在同一对角线上)的棋盘.(一个棋盘由$8\times 8 = 64$个方格组成)

图片


21.一只毛毛虫想要从一个立方体房间的地板的左前角爬到另一个角落(天花板的右后角). 请找出沿着房间墙壁的最短路线.

图片


22.你有两个容器:分别为5升和3升. 用它们测量出一升液体, 并将液体留在其中一个容器中.

图片


23.家里有五个脑袋和十四条腿. 问家里有多少人, 多少只狗?

 

24.在三角形$ABC$的边$AB$, $BC$和$CA$的外部构造三个等边三角形. 证明这些等边三角形的中心(在图上用星号标记)构成一个等边三角形.

图片


25.用平面切割立方体得到的截面可能会是什么多边形? 我们能得到五边形吗? 七边形? 正六边形吗?

图片


26.画一条直线穿过一个立方体的中心, 使得从立方体的八个顶点到它的距离的平方和为(a)最大, (b)最小(与其它这样的直线相比).

 

27.一个正圆锥体沿着一条闭合的曲线被一个平面切割. 圆锥体的两个内切球与平面相切, 一个在$A$点, 另一个在$B$点.在横截面上找到一个点$C$, 使距离$CA + CB$之和为

(a)最大, (b)最小.

图片


28.地球表面投射到一个圆柱体上, 这个圆柱体由与经线相切的直线构成, 这些直线与赤道的交点正是直线与经线的切点.这个投影是沿着平行于赤道平面的光线作出的, 并通过连接地球的北极和南极的地球轴线.问:法国的投影面积是否大于或小于法国的面积呢?

图片


29.证明$2^{p-1}$除奇素数$p$的余数为$1$.

 

30. 将一根10厘米长的针随机扔到格子纸上. 纸上相邻线之间的距离也是10厘米. 重复$N$(比如一百万)次. 问:多少次(在百分之几的误差内)针会落下与纸上的一条线相交?

 

人们可以用$N=100$而不是一百万次投掷来进行这个实验. (我十岁的时候做过这个实验.)

 

这个问题的答案是令人惊讶的: $\frac{2}{\pi}N$. 而且, 即使对于长度为$a\cdot 10$ cm的弯曲针, 在$N$次投射中观察到的相交的次数也约为 $\frac{2a}{\pi}N$. 由此得到$\pi\approx\frac{355}{113}\approx\frac{22}{7}$.

图片


31.有些多面体只有三角形面. 例如某些柏拉图立体: (正)四面体(4面), 八面体(8面)和二十面体(20面). 二十面体的面全等, 有12个顶点, 有30个边.对于任何这样的立体图形(具有三角形面的有界凸多面体), 面的个数是否等于顶点个数的两倍再减去四?

图片
四面体

 

图片
八面体

 

图片
二十面体

 

32.还有一个柏拉图立体(总共有5个):十二面体. 它是一个凸多面体, 具有12(正)五边形面, 20个顶点和30条边(其顶点是二十面体的各个面的中心).将五个立方体内接于十二面体内, 其顶点也是十二面体的顶点, 其边是十二面体的面的对角线. (立方体有12条边, 每条边落在十二面体的一个面上).[ 这个构造由开普勒发明, 用来描述他的行星模型.]

图片


33.两个正四面体可以内接于一个立方体中, 使得它们的顶点也是立方体的顶点, 它们的边是立方体面的对角线. 描述这两个四面体的相交部分.正方体体积和这两个四面体相交部分体积的比值是多少?

 

33(之二).给定正方体边上三个点, 构造出通过这三点的平面与正方体的截面.[ 绘制平面与立方体相交的多边形.]

图片


34.四面体有多少个对称? 立方体? 八面体? 二十面体? 十二面体? 图形的对称是这个图形保持长度的变换.这些对称中有多少个是旋转, 有多少个是平面的反射(对上面五种多面体分别作答)?

 

35.有多少种方法可以用六种颜色(1, ..., 6)来涂相似立方体的六个面(每个面一种颜色), 以使得到的彩色立方体中没有两个是相同的(也就是说, 没有两个可以通过旋转变换变成彼此)?

图片


36.有多少种不同的方法来排列$n$个对象? 对于$n=3$, 有6种方式:(1, 2, 3), (1,3,2), (2,1,3), (2,3,1), (3,1,2), ( 3, 2, 1). 如果对象的个数是$n = 4$? $n = 5$? $n = 6$? n=10呢?

 

37.一个立方体有4个主对角线(连接其相对的顶点). 通过旋转立方体能得到这四个主对角线的多少种不同排列?

图片


38.一些整数和的立方减去这些整数的立方和.得到的差总能被3整除吗?

 

39.与38题类似.问一些整数和的5次方减去这些整数的5次方和, 得到的差总能被5整除吗?一些整数和的7次方减去这些整数的7次方和, 得到的差总能被7整除吗?

 

40.计算下式的和(误差不超过正确答案的$1\%$)
$$\frac1{1\cdot 2}+\frac1{2\cdot 3}+\frac1{3\cdot 4}
+\cdots+\frac1{99\cdot 100}.$$


41.如果两个多边形具有相等的面积, 则可以将它们切割成有限个的子多边形, 使得重新排列这些子多边形可以得到第一个和第二个多边形. 证明这个结论.[ 对于空间体而言情况并非如此:立方体和等体积的四面体不能通过这种方式得到!]

图片


42.一张方格纸上的四个格子点是平行四边形的顶点. 事实上, 在这个平行四边形的边上或其内部没有其它格子点. 证明这个平行四边形的面积等于方格纸的一个方块的面积.

图片


43.假设在问题42中, 一个平行四边形内部有$a$个格点, 边上有$b$个格点. 求这个平行四边形的区域.(译者注:平行四边形的顶点不算作边上的点, 见上图.)

 

44.在三维平行六面体中有类似与43题的结果吗?

 

45.斐波那契(“兔”)数是序列($a_1=1$), $1, 2, 3, 5, 8, 13, 21, 34,\cdots$, 对于任何$n = 1,2,\cdots$, $a_{n+2}=a_{n+1}+a_n$.求$a_{100}$和$a_{99}$的最大公约数.

 

46.沿不相交的对角线切割能将凸$n$边形切割成三角形.不同的切割方式的个数称为卡塔兰 (Catalan) 数, 记为$c(n)$. 例如, $c(4)= 2$, $c(5)= 5$, $c(6)= 14$.求$c(10)$ ?

图片


47.有$n$个队伍参加一个锦标赛. 每场比赛之后, 负的队将被淘汰出局.经过$n-1$场比赛之后的胜者将成为锦标赛的冠军.比赛的赛程可用以下符号记录(例如) ( $(a,(b,c)),d$ ). 这个记号表示有4个队伍参加. 首先$b$对阵$c$, 然后赢家对阵$a$, 最后, 第二场比赛的胜者对阵$d$.

如果锦标赛有10支参赛队伍, 问有多少个不同的赛程?

如果2支队伍参赛, 我们只有$(a,b)$这一个赛程.

如果3支队伍参赛, 则可能的赛程是3个:它们是$((a,b),c)$或$((a,c),b)$或$((b,c),a)$.

如果对于4支队伍参赛, 我们有15种可能的赛程:
\begin{align*}
&(((a,b),c),d);\quad(((a,c),b),d);\quad(((a,d),b),c);
\quad(((b,c),a),d);\\
&(((b,d),a),c);\quad(((c,d),a),b);
\quad(((a,b),d),c);\quad(((a,c),d),b);\\
&(((a,d),c),b);\quad(((b,c),d),a);\quad(((b,d),c),a);
\quad(((c,d),b),a);\\&((a,b),(c,d));\quad((a,c),(b,d));
\quad((a,d),(b,c)).
\end{align*}


48.我们用$n-1$条线段连接$n$个点$1,2,\cdots,n$组成一棵树.有多少个不同的树? (即使是$n=5$的情况也很有趣!)

图片
树的个数$= 1$,

图片
树的个数$=3$,

图片
树的个数$=16$.

 

49.数字$\{1, 2,\cdots, n\}$的一个排列称为一条蛇(长度$n$),如果$x_1<x_2>x_3<x_4>\cdots$.例如, $n=2$, 只有$1 < 2$, 蛇的个数$=1$;

$n=3$, $1 < 3 > 2$, $2 < 3 > 1$, 蛇的个数$=2$;

$n=4$, $1 < 3 > 2 < 4$, $1 < 4 > 2 < 3$, $2 < 3 > 1 < 4$, $2 < 4 > 1 < 3$, $3 < 4 > 1 < 2$, 蛇的个数$=5$;
求长度为10的蛇的个数.

 

50.长度为$n$的蛇的个数记为$s_n$.注意到$s_1=1,s_2=1,s_3=2,s_4=5,s_6=61$.
证明正弦函数的泰勒展式:
$$\tan x=1\frac{x^1}{1!}+2\frac{x^3}{3!}+16\frac{x^5}{5!}+\cdots
=\sum_{k=1}^{\infty}
s_{2k-1}\dfrac{x^{2k-1}}{(2k-1)!}.$$


51.求以下级数
$$1+1\frac{x^2}{2!}+5\frac{x^4}{4!}+61\frac{x^6}{6!}+\cdots
=\sum_{k=0}^\infty s_{2k}\frac{x^{2k}}{(2k)!}.$$


52.对$s>1$, 证明下面的等式
$$\prod_{p=2}^\infty\frac1{1-\frac1{p^s}}
=\sum_{n=1}^\infty\frac1{n^s}.$$


(左边对所有素数$p$做乘积, 右边的对所有自然数$n$求和)

 

(译者注:原文的右边级数求和的下标是$n=1$, 而不是$n+1$)

 

53.求以下级数
$$1+\frac{1}{4}+\frac{1}{9}+\cdots
=\sum_{n=1}^{\infty}\frac{1}{n^2}.$$
(证明这个级数等于$\frac{\pi^2}{6}$,或近似等$\frac{3}{2}$)

 

54.求分式$\frac{q}{p}$在最小项意义下的概率.这个概率是这样定义的:在圆盘$p^2+q^2\leqslant R^2$中,我们用$N(R)$来记圆盘中的所有满足坐标$p$和$q$互素的整数点$(p, q)$的个数, 用$M(R)$来记圆盘中所有整点的个数.这个概率等于$\frac{N(R)}{M(R)}$, 当$R$趋向于无穷时的极限. ($M\sim \pi R^2$)

图片


55.在45题中,我们定义了斐波那契数列.求当$n$趋向于无穷时, $\mathrm{a_{n+1}}{a_n}$的极限
$$\frac{a_{n+1}}{a_n}=2,\frac{3}{2},\frac{5}{3},\frac{8}{5},
\frac{13}{8},\frac{21}{13},\frac{34}{21},\cdots.$$
答案是“黄金分割率”, $\frac{\sqrt{5}+1}{2}\approx 1.618$.这个比率正是一张明信片两边的比例.在明信片上减去以此明信片短边为边长的正方形后剩下长方形的两边比率也近似于黄金分割率.黄金比例与正五边形和五角星有什么关系?

 

56.计算下面无限连分数的和.
$$\begin{aligned}
1+\cfrac{1}{2+\cfrac{1}{1+\cfrac{1}{2+\cfrac{1}{1
+\cfrac{1}{1+\cfrac{1}{\ddots}}}}}}
=a_0+\cfrac{1}{a_1+\cfrac{1}{a_2+
\cfrac{1}{a_3+\cfrac{1}{\ddots}}}},
\end{aligned}$$
其中, $a_{2k}=1,a_{2k+1}=2$, 即, 求当$n$趋向于无穷时, 下式的极限
$$
a_0+\cfrac{1}{a_1+\cfrac{1}{a_2+\cfrac{1}{\ddots +\cfrac{1}{a_n}}}}.
$$

57.以多项式形式表示: $y=\cos 3( \arccos x), y=\cos 4(\arccos x), y=\cos n(\arccos x)$, 其中$|x|\leqslant 1$.

 

58.计算$n$次单位根的$k$次方的和.
(译者注:求和 $\zeta_1^k+\zeta_2^k+\cdots+\zeta_n^k$,其中
$\zeta_{i},i=1,\cdots,n$是方程$x^{n}-1=0$的$n$个复数根.)


59.在$(x, y)$平面上, 作出下面参数方程定义的曲线:
$$\{x=\cos2t,y=\sin3t\},\quad \{x=t^3-3t,y=t^4-2t^2\}.$$


60.计算下面的积分(误差不超过$10\%$)
$$\int_0^{2\pi}\sin^{100}x\ \mathrm{d}x.$$


61.计算下面的积分(误差不超过$10\%$)
$$\int_1^{10}x^x\ \mathrm{d}x.$$


62.求单位球面上角度为$(\alpha,\beta,\gamma)$的球面三角形的面积. (球面三角形的边长是大圆;即,过球心的平面与球面的交线.)

 

答案是: $S=\alpha+\beta+\gamma-\pi$ (例如, 假设这个球面三角形的三个内角都是直角, 则$S=\frac{\pi}{2}$即球面面积的$\frac{1}{8}$)

图片


63.一个半径为$r$的圆在半径为$1$的圆内滚动(不滑动).分别对下述情况绘制滚动圆上一个点的整个轨迹(该轨迹称为内摆线): $r=\frac{1}{3},r=\frac{1}{4},r=\frac{p}{q},r=\frac{1}{2}$;

 

图片


64.在一个有$n$个学生的班级里,估计两个同学生日相同的概率.这会是一个大概率事件,还是小概率?

答案:当学生人数大于某个数$n_0$时, 概率是(非常)大的;当学生人数小于$n_0$时,概率是(非常)小的.这题真正问的是$n_0$具体是多少(当概率$p\approx \frac{1}{2}$时) ?

 

65.斯涅尔定律(Snell’s Law)描述的是入射角$\alpha$满足方程 $n(y)\sin\alpha=$常数, 其中$\alpha$是光束与分层介质法线所成的夹角, $n(y)$是层高$y$处的折射率. (如果我们假设光速在真空中为$1$, 量$n$与介质中的光速成反比例.在水中, $n= \frac{4}{3}$).画出在介质“沙漠上的空气”中光线的轨迹,其中折射率$n(y)$在某个高度达到最大值. (见下面的右图)

 

图片

(对于那些理解物体发出光线的轨迹是怎样与它们的像联系的人,这道题的解答可以解释海市蜃楼现象.)

 

66.在一个锐角三角形$ABC$中做出一个周长最小的内接三角形$KLM$ (顶点$K$在$AB$边上, $L$在$BC$边上, $M$在$CA$边上).

图片

提示:非锐角三角形的答案并不像锐角三角形的答案那样完美.

 

67.计算函数$\frac{1}{r}$在以点$(X,Y,Z)$为圆心半径为$R$的球面上的均值, 其中$r$是点$(x, y, z)$到原点的距离, $r^2=x^2+y^2+z^2$.

 

提示:这个问题与牛顿的万有引力定律和电场里的库仑定律有关. 在问题的二维版本中, 给定的函数应该用$\ln r$来代替, 球面用圆来代替.

 

68.利用$2^10=1024\approx 10^3$不难得到$\lg 2\approx 0.3$, 估计近似值$0.3$和$\lg 2$的差,并且计算$\lg 2$ (保留小数点后3位).

 

69.用与68题相同的精度计算$\lg 4,\lg 8,\lg 5,\lg 50,\lg 32,\lg 128,\lg 125$和$\lg 64$.

 

70.利用$7^2\approx 50$计算$\lg 7$的近似值.

 

71.假设已知$\lg 64$和$\lg 7$的值,求$\lg 9,\lg 3,\lg 6,\lg 27$和$\lg 12$.

 

72.利用$\ln(1+x)\approx x$ (其中$\ln$指的是$\log e$), 再利用关系$\lg a=\frac{\ln a}{\ln 10}$和之前计算的$\lg a$ (例如, $a=128/125$, $a=1024/1000$等)的值来计算$\lg e$和$\ln 10$.

花半个小时计算后, 利用第67-71题结果, 我们可以得到任意数的4位对数表, 其中用到了对数乘积公式和以下公式
$$\ln(1+x)\approx x-\frac{x^2}{2}+\frac{x^3}{3}-\frac{x^4}{4}+\cdots $$
(这也是牛顿编制40位对数表的方法!)

作者注:欧拉常数$\mathrm{e}=2.71828\cdots$其严格定义为
$$e=\lim_{n\to\infty}\left(1+\frac{1}{n}\right)^n
=\sum_{n=0}^\infty\frac{1}{n!}.$$
它也可以通过公式$\lim_{x\to0}{\frac{\ln(1+x)}{x}}=1$来定义.


73.考虑2的幂次序列: $1, 2, 4, 8, 16, 32, 64, 128, 256, 512, 1024, 2048, \cdots$ 在前12个数字中, 4个以1开头, 没有以7开头的.证明当$n\to\infty$时每个数都可能成为$2^m,0\leqslant m\leqslant n$, 的首位数, 并且它们出现的频率为$p_1\approx 30\%, p_2\approx 18\%,\cdots,p_9\approx 4\%$.


74.验证3的幂次序列的首位数: $1, 3, 9, 2, 8, 2, 7,\cdots$证明, 每个数在3的幂次序列中出现的频率和它在2的幂次序列出现的频率一致.求出$p_1,\cdots,p_9$的准确公式.

 

提示: $x$的首位数由$\lg x$的小数部分决定.因此, 我们需要考虑$ma$小数部分的序列, 其中$a=\lg 2$.证明这些小数部分在$[0,1]$内均匀分布: $n$个$ma$的小数部分,且$0\leqslant m<n$,且$\lim_{n\to\infty}\frac{k_n(A)}{n}=A$的长度, 其中$A$是一个子区间, $kn(A)$是子区间包含的$n$个小数部分序列落在子区间$A$中的个数.

 

75. 设$g:M\to M$是一个有界域到自身的一对一的光滑映射, $g$保持这个域的面积(体积, 在高维的情形).证明:在$M$中任一点的任意领域$U$中, 对于任意数$N$, 存在一个点$x$和某个整数$T>N$使得$g^Tx\in U$ (回归定理).

 

76.令$M$是一个环面(坐标为$(\alpha(\bmod 2\pi),\beta (\bmod 2\pi))$), $g(\alpha,\beta)=(\alpha+1,\beta+\sqrt{2})$.证明:对于$M$上任一点$x$, 序列$\{g^T(x)\}$, $T=1,2,\cdots$在环面$M$上稠密.

 

77.记号同76题.令$g(\alpha,\beta)=(2\alpha+\beta,\alpha+\beta)\ (\bmod 2\pi)$. 证明环面上存在一个处处稠密的子集, 由$x$的周期点构成(即,对某个整数$T(x)>0$, 有有下面式子成立$g^{T(x)}=x$)

 

78.继续77题中的记号, 证明对于环面上的几乎所有点$x$, 点的序列$\{g^T(x)\}$, $T=1,2,\cdots$在环面上处处稠密(即, 不满足以上性质的点$x$构成的集合测度为$0$).

 

79.在76和78题中, 证明序列$\{g^T(x)\}$, $T=1,2,\cdots$在环面上一致分布:如果一个域$A$包含$n$个点中的$k_n(A)$个点, $T=1, 2, …, n$, 则
$$\lim_{n\to\infty}\frac{k_n(A)}n=\frac{\mathrm{mes}\ A}{\mathrm{mes}\ M}.$$
(例如, 对一个约当可测集$A$, 其测度为$\mathrm{mes}\ A$.)

 

(翻译自阿诺德的书《讲义和问题:给青年数学家的礼物》, V. I. Arnold, Lectures and Problems: A Gift to Young Mathematicians)


设$A,B$是两个非空集合,如果对于集合$A$中的任意一个元素$x$,按照某种确定的对应关系 $f$,在集合$B$中都有唯一确定的元素 $y$和它对应,并且不同的$x$对应不同的$y$;同时$B$中的每一个元素 $y$,都有一个$A$中的元素$x$与它对应,则称 $f:A{\to}B$为从集合$A$到集合$B$的一一对应,并称集合$A$与$B$等势,记作$\overline{\overline{A}}=\overline{\overline{B}}$.若集合$A$与$B$之间不存在一一对应关系,则称$A$与$B$不等势,记作$\overline{\overline{A}}\neq\overline{\overline{B}}$.

例如:对于集合 $A=\mathbf{N}^\ast, B=\left\{2n\left|n\in\mathbf{N}^\ast\right.\right\}$, 存在一一对应关系 $y=2x\ (x\in A,y\in B)$,因此
$\overline{\overline{A}}=\overline{\overline{B}}$.

(1)已知集合$C=\left\{(x,y)\left|x^{2}+y^{2}=1\right.\right\}$, $D=\left\{(x,y)\left|\frac{x^{2}}{4}+\frac{y^{2}}{3}=1\right.\right\}$,
试判断$\overline{\overline{C}}=\overline{\overline{D}}$是否成立?请说明理由;

(2)证明: \ding{172} $\overline{\overline{(0,1)}}
=\overline{\overline{(-\infty,+\infty)}}$;

\ding{173} $\overline{\overline{\mathbf{N}^\ast}}\neq \overline{\overline{\{x\mid x\subseteq\mathbf{N}^\ast\}}}$.

济南三模第2题:集合与简易逻辑

 

济南三模第6题:抽象函数

 


济南三模第7题:双曲线与解三角形

 

济南三模第10题:函数综合

 

济南三模第14题:圆与抛物线

 

 


\begin{QsNum}
\item
\nandu{5}
\tiyuan{济南三模第18题:导数与微积分}
\tigan{拉格朗日乘数法是一种寻找变量受一个或多个条件所限制的多元函数的极值的方法.要找函数$z=f(x,y)$在条件$g(x,y)=0$下的可能极值点,先构造函数$F(x,y)=f(x,y)+\lambda g(x,y)$,
其中$\lambda$为拉格朗日乘数.

$\begin{cases}f_x(x,y)+\lambda g_x(x,y)=0,\\
f_y(x,y)+\lambda g_y(x,y)=0,\\
g(x,y)=0.\end{cases}$
解出$x,y,\lambda$,其中$(x,y)$就是可能的极值点的坐标.这里$f_x(x,y),f_y(x,y)$分别表示函数$f$对变量$x,y$的偏导数,要求出$f_x(x,y)$,只要在求导时将变量$y$看成常数即可,例如:若$f(x,y)=x^2\ln y$,则$f_x(x,y)=2x\ln y,f_y(x,y)=\frac{x^2}{y}$.

(1)已知$f(x,y)=x^2y^2$,求$f_x(x,y)$和$f_y(x,y)$;

(2)求$f(x,y)=x+y$在条件$x^2+2y^2+xy=1$下的最大值和最小值;

(3)已知椭圆的面积为$S=\pi ab$,其中$a,b$分别为椭圆的半长轴,半短轴的长.在平面直角坐标系$xOy$中,椭圆$C:x^2+2y^2+2xy=1$的中心为原点,求$C$的面积.
}
\daan{}
\jiexi{

(2)最大值为$\frac{2\sqrt{14}}{7}$,此时$\left( x,y \right) =\left( \frac{3\sqrt{14}}{14},\frac{\sqrt{14}}{14} \right)$,最小值为$-\frac{2\sqrt{14}}{7}$,此时$\left( x,y \right) =\left( -\frac{3\sqrt{14}}{14},-\frac{\sqrt{14}}{14} \right)$.


(3)由$a=\frac{3+\sqrt{5}}{2},b=\frac{3-\sqrt{5}}{2}$可知
$S=\pi ab=\pi$.
}


\end{QsNum}

济南三模第19题:解析几何(抛物线与椭圆,开放性,充要条件)

 


2022-2023数学强基讨论班

课程内容安排

o 09/08: 简介,自行车轮轨迹问题

o 模块一:圆周率的方方面面

o 09/15: 圆周率的计算史,阿基米德算法与刘徽算法

o 09/22: 连分数:从阿基米德、祖冲之到兰伯特

o 09/29: 欧拉公式:e, π, i 的结合

o 10/13: 整数与π:巴塞尔问题

o 10/20: 概率与π:互素问题、布丰投掷问题

o 模块二:形形色色的数

o 10/27: 素数:素数无穷之欧几里得证明及其推广

o 11/03: 素数:素数无穷之欧拉证明,素数公式

o 11/10: 幂次数,幂和

o 11/17: 高斯整数

o 11/24: 无理数

o 12/01: 实数的结构

o 12/08: 数的演化

o 12/15: 复数

o 03/10: 四元数

o 03/17: 超限数

o 03/24: p进数

o 模块三:千奇百怪的形

o 03/31: 尺规作图

o 04/07: 等周问题

o 04/14: 平铺问题

o 04/21: 堆球问题

o 04/28: 高维几何

o 05/05: 格点计数

o 05/12: 怪异曲线

o 05/19: 图形割补

o 05/26: 椭圆曲线

o 模块三:应用

o 06/02: 密码学

o 06/09: 组合博弈

\end{document}


\item
\nandu{5}
\tiyuan{微分中值定理}
\tigan{ }
\daan{ }
\jiexi{ }


%\section{求周长}


\begin{QsNum}
\item
\nandu{5}
\tiyuan{2023潍坊三模3}
\tigan{已知平面向量$\boldsymbol{a}$与$\boldsymbol{b}$的夹角是$60^\circ$, 且$|\boldsymbol{a}|=2,\boldsymbol{b}=(1,2)$, 则$\boldsymbol{a}\cdot(2\boldsymbol{a}-\boldsymbol{b})=$
\xuanxiang{$8+2\sqrt{5}$}{$4-\sqrt{5}$}{$8-\sqrt{5}$}{$4+2\sqrt{5}$}}%选择题如需要控制题干隐藏需要将选项放在题干内部
\daan{C}
\jiexi{由题意得, $|\boldsymbol{b}|=\sqrt{5},\boldsymbol{a}\cdot(2\boldsymbol{a}-\boldsymbol{b})=2|\boldsymbol{a}| ^2-\boldsymbol{a}\cdot\boldsymbol{b}=8-2\sqrt{5}\cos60^\circ=8-\sqrt{5}$, 故选C.}


\end{QsNum}

标签:right,frac,真题,高考,sqrt,cdots,leqslant,left
From: https://www.cnblogs.com/Eufisky/p/18393707

相关文章